SC

Réussis tes devoirs et examens dès maintenant avec Quizwiz!

Defined benefit.

An unfunded pension liability is generally associated with which type of corporate retirement plan?

collateralized mortgage obligation has been issued and is backed by mortgages on real estate property. The obligations must pay interest in any of the following time intervals, except: A. Monthly B. Quarterly C. Semiannually D. Annually

Annually. Since interest is paid on a monthly basis by the homeowners, the mortgage obligations must be paid no less often than at least semiannually. The payments can be monthly, quarterly, or semiannually, but they cannot be annually.

change is the customers name or address, change in registered representative assigned to the account

Under SEC rules, which of the following events require a broker/dealer to furnish a copy of the account record to a customer?

Merger, Consolidation, Acquisition, Transfer of Assets

Under Sec Rule 145, which of the following events requires a corporation to receive the approval of its stockholders.

Growth

associated with high P/E ratios

Value

associated with low P/E ratios

An official or officials of the issuer.

Who signs an official statement for a new municipal issue?

All members of the underwriting syndicate.

Who signs the agreement among underwriters for a municipal bond issue?

need not be preceded by an options disclosure document (ODD). OCC communications rules do not distinguish between retail and institutional customers. Therefore their communications rules apply to all customers. All communications pieces must be approved by a registered options principal (ROP). If the educational piece makes no recommendations or performance projections it need not be preceded by an options disclosure document (ODD) but it must be accompanied by a notice containing a name and address where the ODD can be obtained.

Under OCC rules regarding options communications with the public, if an educational piece making no projected performance figures or recommendations is distributed to customers it:

A dealer's officer sits on the issuer's board of trustees.

Under MSRB rules, which of the following would indicate a control relationship between a municipal dealer and an issuer?

is good for the day only.

Under NYSE rules, a not-held order:

amount due does not exceed $1,000

Under Regulation T, action by the broker/dealer is NOT required when the:

net proceeds to the issuer.

Under SEC Rule 134, a tombstone advertisement includes all of the following EXCEPT:

certain limit price or less

buy limit

1 day

an open order is good for how long

cost of money in the marketplace

another than the issuers credit, what is another factor in a bonds interest rate

margin call

customers are not entitled to an extension of time and firms can sell securities without contacting the customer first

Rule 144

does not pertain to primary offerings; it affects secondary market transactions in restricted or control securities.

"OTC PINKS"

quotes are not firm and updates may not be made intra day

T-Bills

Trade at a discount to par and are a direct obligation of the US government

systematic

What type of risk are traders in stock index options exposed to

Callable Preferred

What type of stock usually carries the highest dividend rate

The San Francisco Barge & Dredging Corporation has announced in a tombstone ad that it will issue $50 million of 10% convertible subordinated debenture bonds convertible into common stock at $40. The bonds will mature in November 2032 and are being issued at par. The bonds are secured by:

"The full faith and credit" and no specific collateral of the San Francisco Barge & Dredging Corporation. Debenture and subordinated debenture bonds are not backed by any corporate assets and are commonly called "full faith and credit" bonds.

An investor owns 1,000 shares of stock. The corporation declares a 3% stock dividend and an increase in its quarterly dividend of $0.15 to $0.17 per share. If the record date is June 10 and the payable date is June 30 for both the stock dividend and the cash dividend, the investor will receive:

$0.17 per share and 30 shares. First, make a calendar for the days. Then read the question carefully: .17 per share is the amount paid to current shareholders, plus the 3% stock dividend. The stock dividend is shares in the amount of 3% for each share owned -- 3% x 1,000 shares = 30 shares.

An investor owns a $100,000 City of Las Vegas bond. The bond goes up by 10 basis points. What effect does this have to the investor?

$100 decrease. To find this, 10 basis points = .0010. Now multiply by $100,000: 100,000 x .001 = $100. Since the yield (basis) went up, the price must decrease

An investor buys 1,000 shares of stock on Thursday, August 4. The corporation declares a 10% stock dividend to holders of record on August 10, and a quarterly dividend of $0.24 per share to holders of record on August 16. If the payable date is August 31 for both the stock dividend and cash dividend, the investor receives:

$0.24 per share and 100 shares. The cash dividend of .24 per share is paid to holders of record on August 16. The stock dividend of 10%, in the form of additional shares of stock, is paid to holders of record on August 10. Since the investor purchased the stock before the ex-dividend date, the investor receives both the cash dividend and the stock dividend. Here's how it looks on a calendar: MON TUE WED THUR FRI 1 2 3 4t 5 t = trade day s = settlement day 8 9s 10rs 11 12 rs = record stock dividend 15rc 16 17 18 19 rc = record cash Since the settlement date is before both record dates, the investor owned the security in time to get both dividends.

A customer with a long margin account and a credit balance of $2,000 purchases $7,000 worth of marginable stock. What amount must the customer deposit to satisfy Reg T?

$1,500. To satisfy Reg T, the customer needs 50% of the purchase price of $7,000, or $3,500. With a credit balance of $2,000, the investor only has to come up with another $1,500. This is actual cash in the account. There was no market value given and no SMA given, so assume it is real, live cash. In addition, that money can be used to purchase more shares of stock.

An investor wants to purchase a British pound option that has a contract size of $31,250 and a premium of 5.60. What is the actual amount that is paid by the investor?

$1,750. To calculate the actual dollar amount, multiply the contract by .01 times the premium. If this had been the Japanese yen, you would multiply by .0001. In this case, you multiply $31,250 × .01 × 5.60 = 1,750, or $1,750 per contract.

On what amount does a customer pay interest in a balanced margin account with a credit balance of $50,000 and a debit of $60,000?

$10,000. Firms offset the debit balance with the customer's credit balance for the purpose of calculating the interest due on the outstanding debit balance. In this case $60,000 DR - $50,000 CR = $10,000 debi

An investor owns 1,000 shares of a mutual fund. The offering price is $12 per share and the fund charges an 8% sales charge and has a 1% redemption fee. If the investor redeems his shares he would receive:

$10,930. To determine this, first find the net asset value (NAV). The sales charge is 8%, so the bid price, or NAV, is 92% (100% - 8% sales charge) x $12 = $11.04 (the bid + sales charge = ask price). 1,000 shares x $11.04 = $11,040. A 1% redemption fee ($110.40) is assessed, so $11,040 - $110 = $10,930.

An investor has the following margin account balances: Long market value = $80,000, Dr = $30,000 Short market value = $70,000, Cr = $130,000 SMA = $40,000 What is the equity in the investor's account?

$110,000. Remember that the SMA is not equity, and therefore, is not included in the calculation. The best way is to calculate the equity for each side of the account and then add them together. In this case: Long account = $80,000 - $30,000 = $50,000 Short account = $130,000 - $70,000 = $60,000 Then add: $50,000 + $60,000 = $110,000 [Module 10, Margin, Sections 4.1 & 5.0]

On what balance is interest charged if a client has a short margin account with a credit balance of $30,000 and a long margin account with a debit balance of $45,000?

$15,000. When an investor has a short margin account with a credit balance, and a long margin account with a debit balance, the credit balance in the short account offsets the debit balance in the long account. As a result, the investor is charged interest on the debit balance in the long account minus the credit balance in the short account. In this case, the investor has a long account debit balance of $45,000, and a short account credit balance of $30,000; therefore, interest is charged on only $15,000. [Module 10, Margin, Section 3.0]

What is the offer price of a mutual fund that has a net asset value of $14.35 and an 8% sales charge?

$15.60. Use the formula ASK = NAV + SALES CHARGE. It is best to set the formula up as follows: POP = NAV + SALES CHARGE $ = $14.35 + 100% = 92% + 8% Since the sales charge is 8% of the ask price, the NAV must be 92% of the ask price, and $14.35 divided by 92% = $15.598, rounded to $15.60, as the offering, or ask price.

An investor has a margin account with the following: Long market value = $96,000 Dr = $30,000 What is the SMA for the account?

$18,000. To find the SMA, you must calculate the equity in the account. In this case, the amount of the equity is $66,000 ($96,000 - $30,000). Subtract the Reg T requirement of the market value (96,000 x 50% = $48,000) from the equity: = $66,000 - $48,000 = $18,000 The SMA is $18,000. [Module 10, Margin, Section 4.3]

a mutual fund has a net asset value of $17.40 and a sales charge of 6% on a quantity discount. What is the public offering price at that quantity of purchase?

$18.51. We find the ask price by dividing the NAV by 100% minus the sales charge. In this case, $17.40 divided by 94% (100% - 6%) = $18.51. Remember the formula: POP = NAV + SALES CHARGE = $17.40 + 100% = 94% + 6%

A piece of property in a municipal tax zone has an appraised value of $500,000. The assessed value of the property is at $200,000. If the rate for the taxes on the property is at a 10-mil tax rate, what are the annual taxes due on the property?

$2,000. Remember, a 10-mil tax rate is .010, and is applied only to the assessed value, not the appraised value. Since the assessed value is $200,000 with a 10-mil tax rate, the math is easy, but should be done on a calculator to get the decimal in the right place. 200,000 x .010 = 2,000, or $2,000 in taxes.

An investor contributes $27,000 to a nonqualified variable annuity. Over a period of 12 years, it grows to $36,000. When the investor is 65 years old, he decides to withdraw $16,000. If the investor is in the 28% tax bracket, what is his tax liability?

$2,520. Since the investor is 65 years old, he has no 10% penalty. Therefore, he is taxed at 28%, but only on the $9,000 appreciatio

A customer of yours has invested $65,000 into a nonqualified annuity, which is now worth $75,000. Your customer is 52 years old and has decided to build a house that will be his first home owned. He withdraws $21,000 for use in building the home. How much tax will he pay if he is in the 28% tax bracket? A. $2,800 B. $3,800 C. $5,880 D. $7,980

$2,800. Since this is a nonqualified annuity, the investor has already paid ordinary income tax on his contributions, and therefore owes ordinary income tax on only the $10,000 of appreciation. A 10% penalty tax applies for investors under age 59 1/2 to any amount withdrawn on which ordinary income tax is due; in this case, the penalty tax applies only to the $10,000 of appreciation. However, if the money is used for a first-time home purchase, the penalty is waived on the first $10,000. Therefore, the investor only owes ordinary income tax on the $10,000 of appreciation — $10,000 × 28% = $2,800. [Module 8, Retirement Plans, Section 7.1]

A U.S. Treasury bond quotation in the newspaper shows a net change of .8. What is the dollar value of the change?

$2.50. To determine this, just work with the .8 or 8/32. Multiply the fractional equivalent of 8/32 by $10: 8 divided by 32 = .25. .25 x $10 = $2.50. Remember that the difference between two whole quotes for a bond, such as 97 and 98, is from 970 to 980, or $10. Therefore, any fraction shown is a fraction of $10.

An investor is purchasing a large amount of Mary Jane Mutual Fund. Normally the fund has an 8% sales charge, but when purchasing a certain minimum amount, the fund charges only 6%. If the net asset value is $22.40 at the time of the purchase of an amount allowing the 6% sales charge, what is the offering price the customer has to pay per share?

$23.83. Normally a person purchasing this fund pays the 8% sales charge, but with a quantity purchase, the customer is entitled to the lowered sales charge rate. Since the quantity rate is given as 6%, we then find the offering price the same way as with any other sales charge. We set it up as follows: POP = NAV + SALES CHARGE $ ____ = 22.40 + ____ 100% = 94% + 6% 22.40 divided by 94% = the offering price of $23.83.

Sam Smitty wants to redeem 1,000 shares of his Low Ball Mutual Fund. The offer price that is determined after receipt of the customer's letter requesting the redemption is $26.40. The mutual fund has a 4% sales charge. How much will Sam receive upon redemption?

$25,344. When redeeming mutual fund shares, the customer will receive NAV, minus any redemption fee. In this case, there is no redemption fee, so you only need to determine the NAV, which is the offer price minus the sales charge. Since the sales charge is 4%, you know that the NAV is 96% of the offer price: $26.40 × 96% = $25.344 × 1,000 shares = $25,344. [Module 7, Investment Companies, Section 6.1]

A married couple can give how much each year as a tax-free gift to an UGMA/UTMA account for a minor?

$28,000. Remember, each person can give $14,000. Therefore, a married couple can contribute $28,000 each yea

An investor has $30,000 invested in a direct participation program. For the year, the partnership gave a capital distribution of $13,000 and depletion of $14,000. The investor's cost basis for tax purposes at the end of the year is:

$3,000. Both the capital distribution and the depletion are subtracted from the investor's basis. $30,000 - $13,000 = $17,000; $17,000 - $14,000 = $3,000. [Module 17, DPP, Section 1.3]

A 35-year-old investor has contributed $45,000 into a nonqualified pension plan. Three years later, the account has grown to $55,000. The investor withdraws $15,000 from the plan. How much tax will the investor pay if he is in the 28% tax bracket?

$3,800. Since the investor is under age 59 1/2, only the $10,000 appreciation is going to be taxed at the 28%, plus the additional 10% penalty. Remember, the 10% is ONLY on the amount that will be taxed. The non-taxed money is a return of the contribution, which has already been taxed. Penalties are only on taxable money taken prematurely. [Module 8, Retirement Plans, Section 7.1]

A municipality has a new offering of $1 million, $2 million of existing debt, and responsibility for 70% of a $1.5 million county debt. If the population is 10,000, what is the net direct debt per capita?

$300. To calculate the net direct debt, divide municipal debt by population. In this case, 3 million (2 million old and 1 million new) divided by 10,000 population. The net debt includes overlapping debt. [Module 5, Municipal Securities, Section 9.1]

An investor wants to redeem 1,000 shares of G.T. Global Mutual Fund. The current offering price is $33. The fund charges a 4% sales charge and a 1% redemption fee. What is the approximate amount the investor will receive?

$31,363. To redeem, you must use the NAV (net asset value), not the offer. To calculate the NAV in this case, first subtract the sales charge and multiply by the current offering price: 96% (100% - 4%) x 33 = 31.68. Next, multiply by the redemption fee: 31.68 x 99 (100 - 1% redemption) = 31.363. Finally, multiply by the number of shares to be redeemed: 31.363 x 1,000 shares = $31,363. [Module 7, Investment Companies, Section 6.1]

An older customer of yours purchased 400 shares of Dow Chemical, Inc. at $33 per share. After five years, the customer gives the stock to his son and daughter-in-law as a Christmas gift when the stock is selling for $48 per share. The son eventually sells the stock for $58 per share. What is the son's cost basis of the stock when he sells it?

$33 per share. The gift was made prior to the shareholder's death, so the son and daughter-in-law must take the cost basis of the original shareholder. If the shares were given upon the customer's death from the estate, the shares would have a cost basis of the value on the date of death. [Module 17, Taxation, Section 6.8]

A collateralized mortgage obligation has been issued and is backed by mortgages on real estate property. The obligations must pay interest in any of the following time intervals, except: A. Monthly B. Quarterly C. Semiannually D. Annually

(false statement): Annually. Since interest is paid on a monthly basis by the homeowners, the mortgage obligations must be paid no less often than at least semiannually. The payments can be monthly, quarterly, or semiannually, but not annually. [Module 3, Government Securities, Section 9.2]

RPL, Inc. has a 12% $50 million convertible bond outstanding. The bond is convertible at $40 and has an antidilution feature. If RPL's board of directors declares a 10% stock dividend payable November 1, what will the conversion price be after the dividend date?

$36.36. The first step is to calculate the conversion ratio before the dividend: NUMBER OF SHARES x SHARE PRICE = BOND PRICE ?? x $40 = 1000 ?? = 1,000 ÷ 40 = 25 shares per bond Before the dividend date, the bond would be converted into 25 shares. Now, a 10% increase in shares is 27.5 shares per bond. Determine the parity price with the new shares: 27.5 x ?? = 1,000 Then, divide 1,000 by 27.5 to arrive at 36.36, the new conversion price. [Module 2, Corporate Debt Securities, Section 10.5]

An investor opens a margin account by depositing $3,000 cash. One week later the investor tells his RR to buy 500 shares of a stock that is selling for $30 per share. How much must the customer deposit for the transaction?

$4,500. 500 shares x $30 = $15,000. Reg T is 50% of the total cost, or $7,500, but $3,000 is already in the account. Therefore, the investor must bring in the remainder: $7,500 - $3,000 = $4,500

Due to financial problems, an SIPC trustee has been appointed to handle Rippem Securities' customers. At the time the trustee is appointed, an investor has $1 million in securities and a $600,000 debit balance in his account. The SIPC will cover his account for:

$400,000. The SIPC (Securities Investor Protection Corporation) only covers stock that is fully paid. Since there is a $600,000 debit balance, the customer only owns $400,000 of the securities. [Module 15, Federal & State Regulations, Section 8.0]

Due to financial problems, an SIPC trustee has been appointed to handle Rippem Securities' customers. At the time the trustee is appointed, an investor has $1 million in securities and a $600,000 debit balance in his account. The SIPC will cover his account for:

$400,000. The SIPC (Securities Investor Protection Corporation) only covers stock that is fully paid. Since there is a $600,000 debit balance, the customer only owns $400,000 of the securities. [Module 15, Federal & State Regulations, Section 8.0]

An investor has the following in his margin account: LMV $20,000, Dr $8,000 SMV $ 30,000, Cr 48,000 What is the amount of SMA in the account as shown by its present position?

$5,000. The SMA is the amount of equity in excess of the Regulation T of the market on each account. The long account: $20,000 = $8,000 + $12,000 Reg T = $10,000 ($20,000 x 50%); SMA = $12,000 - $10,000 = $2,000 SMA The short account: $30,000 = $48,000 - $18,000. Reg T = $15,000 ($30,000 x 50%); SMA = $18,000 - $15,000 = $3,000 SMA = $2,000 + $3,000 = $5,000.

A company has an outstanding convertible bond with an antidilution feature and a conversion price of $60. If the company declares a 20% stock dividend, what would the conversion price be?

$50. This is a 20% stock dividend, which is like a 6-for-5 stock split (120 for the old 100 shares, or 120/100 = 6/5). Multiply the price by the reciprocal of the split ratio -- the new price will be 5/6 of the old price, or $60 x 5/6 = $50. [Module 2, Corporate Debt Securities, Section 6.3]

An investor has the following account: Long MV $80,000 Dr $26,000 SMA $10,000 Short MV $16,000 Cr $22,000 What is the amount of equity in the account?

$60,000. The equity in the long account is $80,000 - $26,000 = $54,000; the equity in the short account is $22,000 - $16,000 = $6,000. Do not bother with the special memorandum account (SMA). Therefore, $54,000 + $6,000 = $60,000

A mutual fund has a net asset value (NAV) of $14.20, a sales charge of 8%, and a 1% redemption fee. If an investor wants to redeem 500 shares of this fund, how much money would he receive?

$7,029. Don't forget the redemption fee. 500 shares x the net asset value (or bid price) of $14.20 = $7,100 less the 1% redemption fee ($71) = $7,029. The sales charge has nothing to do with this problem because you are already given the net asset value.

The OEX closes on Wednesday, January 24 at 246.26. The close was down .70 from the previous day's close of 246.96. In terms of options, what does this represent on the OEX?

$70. The OEX (Standard & Poor's 100 Index) is an index option; however, it still has the same multiplier as an equity option -- 100. In this case, .70 x 100 = 70. [Module 9, Options, Section 1.5]

A registered rep contacts one of his clients about a bond. The bond is a $10M EGG 6% 20-year bond selling for $900. The customer desires the bond, which the rep purchases. The client ends up paying $8,700 for the bond, $250 in accrued interest, and $70 dollars in commissions. What is the cost basis of the bond to the investor?

$8,770. Cost basis is the amount the bonds cost. You add the commissions, but not the accrued interest. The accrued that is paid will be deducted from the interest shown on the bondholder's 1099. The buyer of the bond receives six months of interest at the next interest payment, and the interest paid in the transaction is a deduction from that interest that is reported to the IRS as paid to the bondholder. This reduces the tax on that interest received. Upon sale of the bond, the investor will realize a gain or loss by finding the difference between the purchase price PLUS the commission and the sale price less the commission (unless the bond matures and no commission is paid). The difference is the gain or loss on which the investor will pay taxes

A customer is long 500 shares of ABC Corporation stock. He is concerned about the market going against his position, so he enters an order to sell ABC at 82.50 STOP, 82.25 LIMIT. At which of the following price points will the order execute if ABC trades are the following? 82.53, 82.45, 82.30, 82.20, 82.35, 82.60

$82.30. In this case, it is the very next trade after the order is elected, but had the next trade been anything under 82.25, it would not have been executed until a price of 82.25 was met. Since this is a sell stop limit order, first it has to be elected at 82.50 or lower — thus at 82.45 — and then it executes on the next trade of 82.25 or higher — in this case, 82.30

A customer's margin account has the following: Market Value: $1,100 Dr: 0 Equity: $1,100 The customer makes a new purchase of 100 shares of a $20 stock. What must she deposit to make the purchase (Reg T is 50%)?

$900. Since the investor already has $1,100 in the account, she only has to bring the equity up to $2000 as a minimum until the market value of the purchase exceeds $4,000. The difference between $1,100 and $2,000 is $900. Remember that if the customer is borrowing money at all, a minimum of $2,000 must be deposited. If the stock is less than $2,000, then the full value of the stock has to be deposited. [Module 10, Margin, Section 8.0]

On an order using SuperDOT, all of the following statements are true, except: A. Any size order can be transmitted. B. Odd-lot orders or round-lot orders can be transmitted. C. The registered representative gets the confirmation more quickly. D. The order goes directly to the specialist.

(false statement): Any size order can be transmitted. This is not true. Super DOT can only handle orders of up to 100,000 shares. [Module 12, OTC, Section 9.1]

An investor has a margin account with the following: Short market value = $70,000 Cr = $120,000 At what point in market value will the investor have a maintenance margin call?

$92,308. To find the point where there will be a maintenance margin call, you can either calculate the answer or use a short cut. The equation for the margin maintenance call of a short account is: Cr divided by 130% In this case, $120,000 divided by 130% = $92,308. The short cut is to select the answer that lies between the short market value and the credit balance amount. For this question, $92,308 is the only number between $70,000 and $120,000. Remember for short accounts, if the market goes down, the account increases in value. If the market value increases, the broker/dealer would monitor the account's value to make sure the market value does not exceed the amount of the credit balance (Cr). So $92,308 is the only logical number to pick.

The quote on a $1,000,000 government bond trade is 93.1. How much did the buyer pay? A. $930,315 B. $930,312.50 C. $931,000 D. $933,125

$930,312.50. Remember that government bonds are in 1/32, so this 1/32 = .03125. Write down the 93._ _ _ _ _ _ and fill in with 93.03125. Next, multiply $1,000,000 x 93.03125% = $930,312.50. [Module 3, Government Securities, Section 4.0]

The total takedown is 1/2% and 1/8% additional takedown. What is the discount to a syndicate member on $100,000 of bonds?

$99,500. To calculate this question, the additional takedown (1/8 %) is extra information. The discount to the syndicate member is the reoffering yield or POP, less the total takedown. The total takedown is 1/2%, or .5%; 5% × 1,000 = $5 per bond. Therefore the discount to the syndicate is $1,000 - $5 = $995 per bond. The question is asking about 100 bonds, so $995 × 100 = $99,500.

The Federal Reserve Board begins to purchase Treasury securities. This affects all of the following, except: A. Increasing interest rates B. Decreasing interest rates on stock exchange collateral C. Increasing the money supply D. Increasing outstanding bond prices

(false statement). Increasing interest rates. When the Federal Reserve Board is purchasing, it is buying more Treasury bills and notes in the weekly auctions than are maturing. This puts more money into the economy; thus interest rates decrease -- not increase. Since the Fed is buying, prices of debt issues rise. Remember the teeter-totter diagram.

Which of the following is not true regarding Eurodollar bonds? A. The bonds do not trade on the exchanges. B. The bonds are quoted in U.S. dollars out of the country. C. The bonds are issued by U.S. companies and sold to Europeans in U.S. dollars. D. At the issuer's discretion, the principal can be paid in European or U.S. currencies.

(false statement): At the issuer's discretion, the principal can be paid in European or U.S. currencies. Eurodollar bonds cannot be paid off in any currency. It is a bond paid for with U.S. dollars and will be redeemed in U.S. dollars only. The other choices are all true statements about Eurodollar bonds. **Know them.

Open-end investment companies

All of the following securities trade in the over-the-counter market EXCEPT:

A corporation has $20 billion in market capitalization. The corporation has issued over $1 billion in securities in the last year. The corporation is a "well-known seasoned issuer" (WKSI), and is planning a new issue of $20 million in equity securities. Which of the following is not a true statement regarding the issuance of the new equity securities? A. The securities can be sold if the SEC receives notification prior to 9 a.m. on the day the issue will be sold. B. The underwriter must file an S3 with the SEC prior to any sales taking place. C. No discussion of the issue is allowed for the 30 days prior to the SEC having received the S3. D. The issuer prior to the offering being sold to investors must pay a fee based on the amount of shares to be sold.

(false statement). The issuer prior to the offering being sold to investors must pay a fee based on the amount of shares to be sold. This is the opposite of what is true in a new rule for "seasoned" issuers. They are known as WKSIs -- well-known seasoned issuers -- and are allowed to do a type of shelf offering where they pay for the shares that are issued as they go along. No communications are allowed to investors for the 30 days prior to the offering being sent to the SEC, yet the offering can be sold on the day that the SEC receives the registration statement. However, the issuer must have issued at least $1 billion in equity securities in the previous year.

All of the following statements are true in the sale of restricted securities, except: A. Current financial information must be made available to prospective purchasers. B. The sale must conform to the provisions of SEC Rule 144. C. A firm can only act as an agent in the transaction. D. A firm can only act as a dealer in the transaction.

(false statement): A firm can only act as a dealer in the transaction. When transacting in restricted securities, firms are obligated to get the best price available for their customers; therefore, in such cases they must act as an agent — they cannot act as a dealer. Current financial information must be made available to prospective purchasers, and the sale must conform to the provisions of SEC Rule 144.

A new issue, California Rooftops, Inc., is being offered by MBH Securities, Inc. in the state of California. The issue is being issued under Rule 147. All of the following statements are true of the offering, except: A. At least 80% of the revenues of the company come from the state. B. Over 80% of the proceeds will be used in the state of California. C. A minimum of 80% of the purchasers must be California residents. D. Over 80% of the assets of California Rooftops, Inc. are in the state of California.

(false statement): A minimum of 80% of the purchasers must be California residents. This is not true -- 100% of purchasers must be from the state of the issuer. In addition, the investors may not sell their shares to nonresidents for nine months after the close of the offering. At least 80% of the revenues must come from the state, 80% of the assets must be located in the state, and 80% of the proceeds will be used in the state. However, the purchasers must all be from that state.

All of the following municipal bonds are quoted in yield to maturity for the bond, not in yield to call for the bond, except: A. A discounted bond callable at par B. A discounted bond callable at a premium C. A premium bond callable at par D. A premium bond callable at a greater premium

(false statement): A premium bond callable at par. Since the lower of the yield to call or the yield to maturity must be quoted when bond prices are quoted, the only time a bond is quoted on the yield to call (YTC) is when the call or redemption price is lower than the purchase price. Of the four answer choices, three of them (the two discounted bonds and the premium bond callable at a greater premium) are quoted on the yield to maturity (YTM) because the YTM is less than the YTC. A premium bond callable at par has a yield to call that is lower than the YTM, so the yield to call must be used when quoting this bond.

All of the following statements regarding a letter of intent are true, except: A. A letter of intent is good for 13 months. B. A shareholder's privilege to redeem shares is suspended during the time the letter of intent is in effect. C. A letter of intent may be backdated for up to 90 days. D. When shares are purchased under a letter of intent, it is customary for the fund to hold some shares in escrow in the event the additional payment is not made, and an additional sales charge is required.

(false statement): A shareholder's privilege to redeem shares is suspended during the time the letter of intent is in effect. This is not true -- the shareholder can redeem at any time. A letter of intent may be backdated for up to 90 days. When shares are purchased under a letter of intent, it is customary for the fund to hold some shares in escrow in the event the additional payment is not made, and an additional sales charge is required.

In the OTC market, a broker/dealer executing customer trades can do all of the following, except: A. Act as an agent and charge a commission B. Act as a principal and charge a markup C. Act as an agent for both sides of a trade and charge a commission D. Act as an agent for both sides of a trade and charge a markup

(false statement): Act as an agent for both sides and charge a markup. In the OTC market, a broker/dealer executing customer trades can perform all of the functions listed except act as an agent for both sides and charge a markup. This is a question on semantics. A markup occurs in a principal transaction, and a commission occurs in an agency transaction. Keep this in mind. [Module 14, FINRA, Section 1.1]

Below are four statements that refer to both a traditional IRA and a variable annuity. All of the statements are true, except: A. All dividends and interest received by both accounts accumulate tax-deferred. B. All capital gains distributions received by both accounts accumulate tax-deferred. C. All payments during a systematic withdrawal program are taxed as ordinary income above any nonqualified contributions. D. All distributions prior to a systematic withdrawal are taxed as ordinary income and penalized if the person is under 59 1/2.

(false statement): All distributions prior to a systematic withdrawal are taxed as ordinary income and penalized if the person is under 59 1/2. For an IRA, contributions are made with after-tax dollars, but then are deducted from taxes. Therefore, the entire amount contributed to an IRA is presumed to be fully qualified, and fully taxable when distributed. For VAs, unless otherwise stated, it is presumed that contributions are made with after-tax dollars, and are nonqualified for tax deductions. Therefore, VA distributions are only taxed and penalized (if applicable) on the appreciation. Thus, the statement that all distributions are taxed as ordinary income is not true for VAs. For both IRAs and VAs, only qualified contributions are taxed and penalized if under the age of 59 1/2. For both IRAs and VAs, dividends, interest, and capital gains accumulate tax-deferred. Although IRAs typically contain only qualified contributions, it is possible to have nonqualified contributions if there is an excess contribution that didn't qualify for a tax deduction in the year it was contributed. [Module 8, Retirement Plans, Sections 7.1 & 12.0]

All of the following transactions by a municipal broker/dealer are considered principal transactions, except: A. Buying 1,000 bonds to fill an institution's order for 2,000 bonds B. Selling bonds out of inventory to a retail customer C. Selling bonds out of inventory to a municipal broker's broker D. Buying bonds into inventory from a broker's broker

(false statement): Buying 1,000 bonds to fill an institution's order for 2,000 bonds. A broker/dealer acts in a principal capacity (as a principal) when it buys for or sells from its own account (buying into inventory or selling out of inventory). When a firm buys for a client -- in this case buys to fill an institution's account -- it is acting as a broker. Such a trade could also be considered a simultaneous transaction, because the firm is not taking on any risk by buying the 1,000 bonds. The firm acts as a broker, buying for the institution.

The funded debt of a corporation includes all of the following, except: A. Commercial paper B. Convertible bonds C. Mortgage bonds D. Subordinated debentures

(false statement): Commercial paper. Commercial paper is short-term and is not part of the debt that "funds," or starts up, a company. Funded debt is used in the business capitalization of the company; commercial paper is for short-term use of money.

All of the following are true regarding an ADR, except: A. A bank in the foreign country holds the shares of the stock. B. Each receipt issued is for only one share of the stock. C. Holders of ADRs have a right to declared dividends. D. Dividends are declared in the foreign currency, but paid in U.S. dollars.

(false statement): Each receipt issued is for only one share of the stock. This is not true -- each ADR represents a specified number of shares of the foreign stock. The ADRs are issued by a bank that holds the shares of the foreign stock. The dividends are declared in the foreign currency. The bank that is holding the shares converts the dividend to U.S. dollars and sends it to the ADR holders. ADR holders are entitled to the dividends, but cannot vote in corporate elections and may not subscribe to any rights offerings.

The securities of all of the following trade in the capital market, except: A. Fannie Mae B. Bank for Cooperatives C. Federal Land Bank D. Federal Reserve Bank

(false statement): Federal Reserve Bank. The Federal Reserve Bank does not issue securities. The capital market represents the issuance of stock or bonds. The Federal Reserve Bank holds government money and bank money -- it does not issue securities

A registered rep works for a broker/dealer that charges one fee to its customers. The fee covers the commissions for trading the securities as well as investment advice. All of the following are true regarding the account, except: A. The fee is a percentage of the assets that the customer has with the firm. B. The registered rep must be registered with the state administrator and have passed an appropriate exam. C. If the customer resides in one state and the broker/dealer firm is in another state, the broker/dealer does not have to be registered in the state of the customer. D. The customer must be given a brochure explaining the fees and the benefits the customer will receive.

(false statement): If the customer resides in one state and the broker/dealer firm is in another state, the broker/dealer does not have to be registered in the state of the customer. This is not true -- both the firm and the registered rep MUST be registered in the state where the customer lives for the advice and trades to take place. Investment advisers must pass a qualifying exam as determined by the state administrator.

All of the following are functions of the Federal Reserve Board (FRB), except: A. It sets the federal funds. B. It sets the discount rate. C. It makes loans to commercial banks. D. It acts as fiscal agent for the U.S. government.

(false statement): It acts as fiscal agent for the U.S. government. The Fed is not the agent that sells the Treasury issues at the auction; the Department of the Treasury carries that responsibility. The Treasury actually sets the time and dates for the auctions and determines the winners of the auction. The Fed sets the rates that the Treasury publishes as the maximum rate acceptable at each auction. The Fed does set the range of the federal funds rate, the rate that banks use when borrowing and lending to each other. The Fed also sets the discount rate, the rate at which the banks borrow from the Fed. Banks set the prime rate for their customers.

All of the following are true of commercial paper, except: A. It is marketable. B. It is issued by a corporation. C. It trades at a discount. D. It is registered.

(false statement): It is registered. - Commercial paper is not registered. -It is issued for less than 270 days, and any security that is issued for less than 270 days does not need to be registered with the SEC. Registered in the question means "registered with the SEC" -- it does not mean "in registered form."

All of the following statements regarding the federal funds rate are true, except: A. It is usually lower than the discount rate. B. It is a leading indicator of short-term interest rates. C. Other short-term interest rates fluctuate in relation to the federal funds rate. D. It usually fluctuates in response to other short-term rates.

(false statement): It usually fluctuates in response to other short-term rates. Federal funds rates do not fluctuate in response to short-term interest rate changes. The federal funds rate is the rate that banks charge each other for overnight loans. Short-term interest rates react to the federal funds rate. The federal funds rate is not a leading indicator of all interest rates, but it is an indicator of short-term interest rates and a reaction to the desires of the Fed at the present time. In addition, it is lower than the discount rate.

All the following are needed on the comparison of a stock trade sent to the contra B/D, except: A. Name of the underlying company or its symbol B. Number of shares C. Price per share D. Name of the customer if placed for a customer

(false statement): Name of the customer if placed for a customer. In comparisons sent broker to broker, the name of the customer is not required. The stock symbol, bond CUSIP, name of the company, number of shares, and the price at which the trade was executed are required.

Open-end investment companies.

All of the following securities trade in the over-the-counter market EXCEPT:

A new investor to the firm has called to purchase 500 shares of ASP Limited, Inc. All of the following are needed to open a cash account for the customer, except: A. New account form with the customer's signature B. The customer's age C. The customer's employer D. The customer's name and address

(false statement): New account form with the customer's signature. The customer's signature is not needed. The principal's signature is required on all accounts. The registered rep's signature is needed only if the question states the purpose of the account is to purchase municipal securities. The customer's signature is only required on a margin agreement, options agreement, and/or a numbered account. All the other choices are needed, including the customer's age. [Module 11, Customer Accounts, Section 5.1]

All of the following statements are true concerning collateralized mortgage obligations (CMOs), except: I. WACs and WAMs are calculated separately for each tranche, and the final maturity date must be shown. II. The U.S. government guarantees yield, principal, market value, and maturity. III. The CMO has the same rating as the underlying portfolio of securities. IV. CMOs are not backed by mortgages; they are mortgage-backed securities and may not be compared with other products; prepayment may affect estimated yield.

(false statement): The U.S. government guarantees yield, principal, market value, and maturity. This is not true. In fact, the government can only guarantee the principal if the CMO is backed by GNMA securities. If there are other issuers of the loans, the principal is not backed.

When money is easy and the interest rate curves are normal, all of the following statements are true, except: A. The prime rate is above the discount rate. B. The federal funds rate is above the prime rate. C. The prime rate is above the broker's loan rate. D. The broker's loan rate is above the discount rate.

(false statement): The federal funds rate is above the prime rate. This is not true. Indeed, the opposite of this is true. Today, the federal funds rate is the lowest rate of all the rates given, although it has not always been this way. Remember that the broker's loan rate, known as the call rate, is for purchasing stock. All rates, except the federal funds rate, are higher than the discount. Therefore, if the fed funds rate is below the discount rate, then it must be below the prime rate as well

All of the following statements are true when the city of San Rafael refunds their existing 9 1/2% term bond, except: A. The underlying security changes B. There would be a defeasing of the interest C. The marketability of the issue increases D. The pre-refunded funds will be held in escrow

(false statement): The underlying security changes. This is not true. In fact, the underlying security remains the same. All the other statements are true. [Module 5, Municipal Securities, Section 8.7]

All of the following statements are true about variable annuities and mutual funds, except: A. Mutual funds and variable annuities are regulated under the Investment Company Act of 1940. B. Variable annuity companies retain dividends, but the owner of the variable annuity must pay taxes on the dividends each year. C. Both mutual funds and variable annuities are considered securities. D. The payout of both mutual funds and variable annuities depends on the performance of the securities owned in the portfolio.

(false statement): Variable annuity companies retain dividends, but the owner of the variable annuity must pay taxes on the dividends each year. Variable annuities do retain dividends but the owner of the variable annuity does not have to pay taxes on the dividends each year. All investment income and capital gains are tax-deferred until withdrawn. When they are eventually withdrawn, these funds are considered ordinary income. Mutual funds and variable annuities are regulated under the Investment Company Act of 1940. Both mutual funds and variable annuities are considered securities. The payout of both mutual funds and variable annuities does depend on the performance of the securities owned in the portfolio.

All of the following statements are true about variable annuities and mutual funds, except: A. Mutual funds and variable annuities are regulated under the Investment Company Act of 1940. B. Variable annuity companies retain dividends, but the owner of the variable annuity must pay taxes on the dividends each year. C. Both mutual funds and variable annuities are considered securities. D. The payout of both mutual funds and variable annuities depends on the performance of the securities owned in the portfolio.

(false statement): Variable annuity companies retain dividends, but the owner of the variable annuity must pay taxes on the dividends each year. Variable annuities do retain dividends but the owner of the variable annuity does not have to pay taxes on the dividends each year. All investment income and capital gains are tax-deferred until withdrawn. When they are eventually withdrawn, these funds are considered ordinary income. Mutual funds and variable annuities are regulated under the Investment Company Act of 1940. Both mutual funds and variable annuities are considered securities. The payout of both mutual funds and variable annuities does depend on the performance of the securities owned in the portfolio.

All of the following are true about the purchase of SPDR exchange-traded funds, except: A. They are a unit investment trust. B. They can be purchased on margin as a new issue. C. When purchased as a new issue there is a sales charge. D. They trade on the NYSE.

(false statement): When purchased as a new issue there is a sales charge. There is never a sales charge with an ETF, whether as a new issue or as a trade in the secondary market. All shares, new and secondary, trade on the NYSE and have a commission charge. Even the new issues have a commission. Since the shares are traded on an exchange, they can be purchased on margin, and are the only securities that can have new issues purchased on margin. SPDRS are open-end UITs

most Treasury bill issues are callable

All of the following statements regarding Treasury bills are correct EXCEPT

Which of the following are true regarding the dividends and capital gains generated by the securities in the separate account of a variable annuity? I. Capital gains are taxed as capital gains at the time the annuitant takes the distribution. II. Capital gains are taxed as ordinary income at the time the annuitant takes the distribution. III. Dividends are taxed as ordinary income in the year they are realized. IV. Dividends are taxed as ordinary income in the year the annuitant takes the distribution.

-Capital gains are taxed as ordinary income at the time the annuitant takes the distribution -dividends are taxed as ordinary income in the year the annuitant takes the distribution. A variable annuity defers taxes on the income and capital gains until the payout period. At that time, all distributions are considered ordinary income.

On January 5, Swampland Building Corp. declared a cash dividend of $0.35, payable to stockholders of record on Friday, January 28. The dividend is payable on February 28. A customer who owns 800 shares and purchases 200 additional shares of Swampland on Friday, January 21 would receive:

0.35 per share on 1,000 shares. The customer will receive the dividend of $0.35 on the 800 shares owned. The additional shares are owned prior to the record date for the cash dividend, so the customer receives $0.35 per share of stock owned (1,000 shares).

Samantha intends to invest $40,000 in Tiptop Mutual Fund. The fund charges a 7% sales charge for minimal purchases. Today the fund determined that the public offering price is $24.78 and the NAV is $23.05. The sales charge for quantity purchases at $40,000 is 5%. How many shares of Tiptop Mutual Fund will Samantha receive if she invests in time to get today's price?

1,648.6 shares. To find this, you must first deduct the sales charge and then divide by the NAV. Remember, for a quantity purchase, the new public offering price (POP) must be found, because the POP is based on the standard sales charge. You cannot just deduct 2% (the difference between the standard sales charge and the discounted sales charge for quantity purchases) from the POP to arrive at the new POP. You can either divide the NAV by the reciprocal of the sales charge to get the new POP, or you can deduct the reduced sales charge from the amount of the purchase and divide by the NAV. In this case: $40,000 x 5% = $2,000 Then, $40,000 - $2, 000 = $38,000 Finally, $38,000 divided by $23.05 = 1648.59, or approximately 1648.6 shares

PNDA, Inc. follows the statutory method in voting for its board of directors. If a shareholder owns 100 shares of stock, and elects four new directors out of six people running, the shareholder can vote:

100 votes for each of the four directors the shareholder wishes. Statutory voting allows one vote per share per director of the shareholder's choice. The shareholder can vote the total number of votes equal to the number of shares owned for each of the directors being elected. The shareholder cannot combine the number of votes allotted for each director and cast the total number of votes to one candidate, which is allowed with cumulative voting.

A woman is 28 years old and has six months in cash for living expenses. She has invested previously and, because of her age, she does not mind some risk in her investments. She does not need income, as her salary is sufficient. Of the following examples, which investing program would you recommend? A. 100% stock, no bonds B. 50% stock, 50% bonds C. 25% stock, 75% bonds D. No stock, 100% bonds

100% stock, no bonds. Since no income is needed, skip the bonds. In actuality, there should always be some percentage in debt securities, but since the answers with bonds are so high, and income is not needed, the best answer of those given does not include bonds. [Module 18, Scenario-Based & Financial Statement Questions, Section 3.0]

On Friday, April 8, an investor buys 10 U.S. government bonds with an 8.5% coupon. The bonds pay interest on January 1 and July 1. For how many days of accrued interest does the investor have to pay?

100. To calculate this: 4/8 Trade date + 3 (Regular way is 1, but add 3 days for the weekend) 4/11 Settlement date - 1/1 The last interest date 3/ 10 or 3 months and 10 days = 3 x 30 = 90 days + 10 days = 100 days Since we assumed each month had 30 days, we must add 1 day for each month that has 31 days, which in this case is January and March; we must also subtract 2 days for February, which only has 28 days (don't account for leap years). Therefore, 100 days + 1 + 1 - 2 = 100 + 2 - 2 = 100 days of accrued interest.

A customer purchases a corporate bond on Tuesday, August 24. If the interest payments are May and November 15, the accrued interest will be paid for:

102 days. To solve, remember the shortcut. 8/24 trade date +3 to settlement (5 if a weekend is included) 8/27 settlement date - 5/15 last interest date 3/12 the number of months (at 30 days) plus extra days. So, 3 x 30 = 90 + 12 = 102. Of course, the only answer with a 2 at the end (the same 2 as in 3/12) is 102, so that must be the answer.

On Thursday, July 17, an investor buys 100 U.S. government bonds with a 7% coupon. The bonds pay interest on April 1 and October 1. For how many days of accrued interest does the investor have to pay?

108 days. To find this do the following: 7/17 Trade date + 1 7/18 Settlement date - 4/ 1 The last interest date 3/17 or 3 months and 17 days = 3 x 30 = 90 days + 17 days = 107 days Since we assumed each month had 30 days, we must add 1 day for each month that has 31 days, which in this case is only May: 107 days + 1 = 108 days of accrued interest.

On May 20, Smith-Corona Corp. declares a $0.22 cash dividend to shareholders of record on Monday, June 15 and a 12% stock dividend to holders of record June 18, both payable July 15. An investor buys 100 shares of Smith Corp. on June 11 in a regular way transaction. How many shares will the investor receive?

112 shares. You must draw a calendar. Since Monday, June 15 is the record day for the cash dividend, the person buying on Thursday, June 11 does not get the cash dividend because she settles on Tuesday, June 16, a day that is too late to get the dividend. She does get the stock dividend with the record day of Thursday, June 18, as she is the owner on the record date for the stock dividend. [Module 14, FINRA, Section 5.1]

In a short margin account, an investor has a market value of $40,000 and a credit balance of $80,000. What is the minimum maintenance of the account (Reg T 50%)?

12,000. The minimum maintenance in a short account is 30% multiplied by the market value. In this case, $40,000 x 30% = $12,000

IBM finished trading at 138 on an uptick. The next day, the stock is going ex-dividend at $0.58. An investor believes that the stock is about to turn around and drop in price. What is the lowest price the investor can sell short at the opening price? A. 137.41 B. 137.42 C. 137.43 D. 138

137.41. The investor can sell short at any price. Since the question asked for the "lowest price," pick the lowest price. The stock is reduced by the dividend, so at 137.42, the trade would be considered a zero-plus tick. However, the trade does not have to be on an uptick or on a zero-plus tick. Any price can execute a short sale. When asked for the lowest price, choose the answer with the lowest number. [Module 13, NYSE, Section 3.17]

An investor buys 10 Microsoft Inc. 8% convertible bonds for 102.5 on Tuesday, July 18. If the interest payments are on March 1 and September 1, for how many days would the buyer owe the seller accrued interest?

140 days. Calculate this the following way: Trade date 7/18 7/21 + 3 - 3/ 1 Settlement date 7/21 4/20 or 4 months and 20 days 4 months x 30 plus 20 days = 140 days

When does a customer have to sign the options account agreement? A. Before a trade can be entered B. Before the options principal signs the new account form C. 15 days before opening the account D. 15 days after the account is opened

15 days after the account is opened. This is the rule, not the actual way most broker/dealers work. If the customer is a major holder of stock held by the firm, this might happen, but most of the time the agreement must be signed before the account is opened. Don't confuse the rule with the real world. [Module 9, Options, Section 13.0]

Which of the following indicates the maximum suggested percentage of a customer's investment portfolio that should be devoted to options trading activity? A. 5% to 10% B. 15% to 20% C. 20% to 30% D. 100%

15% to 20%. This is a conservative suggested maximum amount. Some investors have 100% of their portfolios in options, while others (the majority) only have from 0% to 10% of their portfolio in options. It really depends on the investor's comfort level. However, 15% to 20% is in the OCC disclosure document, and this is the number FINRA wants to see in the exam. [Module 9, Options, Section 13.0]

An investor has a portfolio that is composed of securities similar to an index. The value of the portfolio is $568,000. The investor has compared the portfolio to the index and estimates the portfolio to have a beta of 1.5 compared to the index. If the market decreases 10%, how much would you expect the portfolio to decrease?

15%. In this question, the relative movement of the index is being compared to a portfolio. Beta is an estimate that approximates the movement of a portfolio of securities in relation to an index of securities. Since the market changes 10%, and the beta is 1.5, you can estimate that the reaction by the investment may be 1.5 x the effect on the index: 10% x 1.5 = 15%. [Module 9, Options, Section 18.4]

An insider owning 20,000 shares of EGG Corp. unregistered stock has filed a Form 144 notice of offering. The weekly volume of trading in EGG on all exchanges is: Week ending March 11 ..................20,000 March 18 ..................17,000 March 25 ..................17,000 April 1 ..?................18,000 April 8 ......................19,000 EGG has 1.84 million shares of stock outstanding. On April 12, the insider would like to sell a portion of his unregistered stock. What is the maximum amount of stock he can sell under Rule 144?

18,400 shares. To calculate this number, there are two steps. First take 1% of 1.84 million (the outstanding shares) = 18,400. Second, add up the volume of shares for March 18th through April 8th (71,000) and divide by 4 weeks = 17,750. Since 18,400 is the larger amount, it is the amount that can be sold.

You have a customer who purchases $50 M of General Motors 6% debenture bonds. The bonds are convertible at $25 per share. The company has been doing very well lately and the stock has risen to $40 per share. Your customer wants to convert the bond into stock because the bonds will be callable next year, and he is concerned the value may drop. How many shares will he get for his bonds?

2,000 shares. Remember that the conversion price is set at the time the bonds were issued, so they are still convertible at $25 per share. This question is not looking for the parity price, just the amount of stock shares the investor will receive. Based on $25 per share: Sh x $$$$ = B ?? x $25 = 1,000 40 shares per bond x 50 bonds ($50,000 in bonds) = 2,000 shares.

The debt statement of a municipal revenue issue provides the following information: Revenues $15,000,000 Interest $2,000,000 Principal payments $1,000,000 Operating expenses $9,000,000 Under a net revenues pledge, the debt service coverage ratio is:

2-to-1. Debt service coverage is just as it says -- how much is available to pay the debt service. However, you have to be careful. If the question asks for a "net revenues pledge," subtract the expenses from the revenues first. If the question asks for a "gross revenues pledge," do not subtract the expenses at all. Since this is a "net revenues pledge," take the revenues minus operating expenses and divide by the combined principal and interest. In this case, $15,000,000 - $9,000,000 = $6,000,000 divided by 3,000,000 = 2, or 2-to-1.

For the fiscal year 2010-2011, ABC Inc. had total revenues of $11 million. During the year, the company had expenses of $4.9 million and a debt service of $2.8 million. What is the debt service coverage for ABC Inc.?

2.18 to 1. For every dollar of debt service, the amount of principal and interest that is due; there is $2.17 of income to pay each dollar of the bonds. To calculate this number, divide the net revenues by the debt service. Remember: The debt service coverage is the amount the debt service is covered. In this case: $11 million - $4.9 million = $6.1 million; then $6.1 million divided by $2.8 million = 2.18, or 2.18 to 1.

Mobil Oil is paying a quarterly dividend of $0.57. If the present price of Mobil Oil Co. is $102.50, what is the yield from Mobil Oil for the stockholders?

2.22%. Don't forget that the dividend shown is a quarterly dividend, or $0.57 x 4 quarters = $2.28. Now, yield is always the dividend or interest divided by the going market price. In this case, 2.28 divided by 102.50 = .0222, or 2.22%.

A mutual fund has a bid price of $15 and an ask price of $15.65. The fund has distributed a capital gain of $2.05 and a dividend of $0.38 during the past year. The yield for the fund was:

2.43%. In computing yield on a current basis, never combine dividends and capital gains. You just divide the yearly dividend by the ask price. Remember -- yield is calculated at a point in time, and it changes as the value of the shares change and as distribution changes. In this case, you just divide the $0.38 by $15.65 to get 2.43%.

Chevron Oil, Inc. calls 1 million shares of convertible preferred stock for redemption at $40 plus an accumulated dividend of $0.18. Each share of convertible preferred stock can be converted into 1/2 share of common stock and is selling at $30. The common stock has two million shares outstanding with earnings at $4.50 per share, and it is selling at 81.72. If all the called convertible preferred stock shares were converted to common, how many shares of common stock would be outstanding?

2.5 million. There are already 2 million shares of common stock outstanding. With the conversion of 1 million shares to common stock at the conversion ratio of one share of convertible preferred stock for one-half share of common stock, the 1 million shares of convertible preferred convert to 500,000 shares of common. Add the 500,000 new shares of common to the existing 2 million to arrive at 2.5 million shares of common, if all convertible preferred stock shares are converted. When reading exam questions, take care to analyze the specific question and not be distracted by information that is not relevant.

An investor enters an order to buy 300 shares of DAL 24.25 stop limit in the middle of the day. After her order is entered, trades occurred as shown: DAL 3s 24.20 . 3s 24.25 . 5s 24.40 . 4s 24.55 . 2s 24.60 . 7s 24.35 . 3s 24.15 At what price will the order be executed?

24.15. This is a buy stop limit order -- a "frown" drawing. The market has to go up to 24.25 or higher to be elected, and then back to 24.25 or lower. It is elected at 24.25, and then the order is executed at 24.15. [Module 13, NYSE, Section 3.6]

Podunk, Inc. has earnings of $4 per share, the preferred dividend is $1, and the common dividend is $1. What is the dividend payout ratio?

25%. Do as the question asks. How much dividend is paid out of the earnings? Only use the common dividend because the preferred dividend is always fixed. In this case, $1 divided by $4 = 25%

A cash transaction of a U.S. government bond at noon must settle at: A. 5:00 p.m. that day B. 2:30 p.m. the next day C. 5:00 p.m. the next day D. 2:30 p.m. that day

2:30 p.m. that day. All cash transactions settle that day at 2:30 p.m., unless the trade is after 2:00 p.m. In that case, they have a half-hour to bring in the money or security. [Module 14, FINRA, Section 5.0]

You have a client with the following amounts in his long margin account: MV DR SMA $120,000 $70,000 $0 He calls and tells you to do the following trades: Buy 300 shares of EGG at $60 for his account Sell 600 shares of UAL at $40 from his account How much money can the investor have sent to him?

3,000. This is a restricted account. This can be seen where the debit balance is more than 50% of the market value; thus, the equity is less than 50% of the market value. Therefore, the customer can take 50% of the sale proceeds. In addition to the sale of $24,000 (600 x $40) worth of stock, the investor has purchased $18,000 (300 x $60) worth of stock. The investor can always take 50% of the net trade, even when restricted. If there had been SMA, the investor could have taken more. In this case, sell $24,000, buy $18,000, and the difference is $6,000 x 50% = $3,000. The investor can take that amount. If the investor does not take the $3,000, the amount will be applied to the debit balance and the SMA will be increased by $3,000. [Module 10, Margin, Section 4.2]

On a particular day, Pigeon Hole Investment Company closes with an NAV of $12.65 and an offer price of $13.75 with an 8% sales charge for a minimum amount of purchase. The fund offers a sales breakpoint of 6% for purchases of $40,000 - $60,000. An investor tells you to purchase $50,000 of Pigeon Hole. You enter the order prior to the close of business that day. The investor receives approximately how many shares of the fund? $40,000 - $60,000 @ 6%

3,715 shares. This is a quantity purchase, so the public offering price is not used. Instead, a new offering price based on the amount of the purchase must be used. In this case, the $50,000 qualifies for a 6% sales charge. You can find the new offer price using the following formula and dividing that offer into $50,000. POP = NAV + SALES CHARGE (DOLLARS) $ = $12.65 + (PERCENT) 100% = 94% + 6% We can determine that the ask price is 13.457, rounded to 13.46. Then, $50,000 ÷ 13.46 = about 3,715 shares. An easier way, however, is to take out the sales charge and divide by the NAV of 12.65: 100% - 6% = 94%; $50,000 x 94% ÷ $12.65 = 3715.415 shares

The following, appearing on Network A, indicates: BRK 3s/s 12,264

30 shares traded. The "s/s" represents 10 unit shares; therefore, 3 x 10 = 30. [Module 13, NYSE, Section 2.3]

The dollar supply of new bonds to be issued in a month would be found in which of the following? A. Bond Buyer B. Blue List C. 30-Day Visible Supply D. Municipal Bond Buyer's Financial Advisers Service

30-Day Visible Supply. The 30-Day Visible Supply is found in the daily Bond Buyer, but only on Mondays. It is a list of what issues are coming out in the next 30 days as well as the lead underwriters associated with the issues. [Module 6, Underwriting, Section 5.3]

Which of the following debt securities is the most volatile in price in a rising market? A. A negotiable certificate of deposit B. A six-month Treasury bill C. A five-year Treasury note D. A 30-year Treasury bond

30-year Treasury bond. Always pick the longest time to maturity for price volatility and pick the shortest term to maturity for interest-rate risk. Since there are three short-term and one long-term security, the odd-man-out method of selection makes the answer easy -- choose the long-term bond. [Module 4, Economics & the Markets, Section 2.3]

During trading one day, an investor enters an order to sell short 500 shares of EPI 38.50 stop limit. After her order is entered, trades occur as shown: EPI 3s 38.60 . 5s 38.40 . 4s 38.30 . 2s 38.35 . 7s 38.45 . 5s 38.50 . 3s 38.55 At what price will the order be elected?

38.40. This is a sell stop-limit order -- a "smile" drawing. The price has to drop to 38.50 or lower to be elected, then back to 38.50 or higher to be executed. The question states that the order is to be elected; therefore, the first trade below 38.50 is 38.40 where the order was elected. Do not worry about the sell short aspect because the order is not yet elected and a short sale can be executed on an up or downtick. In this case, the order would have sold at 38.50, the limit price.

A mutual fund NAV is calculated to be $16 at the end of the day. The offer price for a minimal purchase is $17.39 with an 8% sales charge. A customer calls early in the day to make a purchase of $70,000 worth of the fund that day. The customer receives approximately how many shares? $50,000 - $75,000 @ 5%

4,157 shares. Since this is a quantity purchase, the investor receives the breakpoint discount. The breakpoint discount is represented by the sales charge of 5%, not the 8% sales charge for a minimal purchase. In this case, the $70,000 qualifies for a 5% sales charge. You can find the new offer price using the following formula and divide that offer price into the $70,000. POP = NAV + SALES CHARGE (DOLLARS) $ = $16 + (PERCENT) 100% = 95% + 5% 16 divided by .95 = 16.84 per share. $70,000 divided by 16.84 = 4,156.7, rounded to 4,157 shares. An easier and more accurate way to calculate the number of shares is to deduct the sales charge from the total purchase and then divide by the NAV since the sales charge has been paid. Remember 100% - 5% = 95%; $70,000 x 95% divided by $16 = 4156.25.

A mutual fund pays $0.30 in dividends and $0.75 in capital gains during the year. The offer (ask) price at the end of the year is $6.50. What is the current yield on this fund?

4.6%. You must remember that yield is calculated at a point in time, and it changes as the value of the shares change and as distributions change. In this case, what you get is the dividend and what you pay is the ask price. Do not consider the capital gain. We always use the dividend and the ask price. Then just do the division: $0.30 divided by $6.50 = 4.6%.

An investor writes a 40 call for 4 after previously purchasing a 30 call for 11. At what point is the profit from the 30 call going to be offset dollar for dollar by the loss from the 40 call

40 1.The question is asking when one call is going to be offset dollar for dollar by the other call. For calls, it is always the highest strike price because the investor will be exercised at that price. For puts, it is always the lower strike price for the same reason. [Module 9, Options, Section 3.1 & Tutorial 9-5, Slide 83]

A 53-year-old woman has contacted you to open an account with $100,000. She is not retired but will be retiring in the next 10 years. She wants some income, but at the same time is looking for appreciation for her retirement. She also has just gone through a divorce, and therefore, has a large amount of legal expenses. How would you allocate her investments? A. 100% equity and zero-percent bonds B. 50% equity and 50% bonds C. 65% equity, 25% bonds, and 10% cash D. 40% equity, 40% bonds, and 20% cash

40% equity, 40% bonds, and 20% cash. This person needs cash for her divorce expenses; therefore, at least 20% cash should cover the large amount of legal expenses. In addition, she will be receiving money periodically from the bonds, so no more than 20% is needed. Since she is getting near retirement, she cannot have a large amount of equity in her portfolio, but she should have some for future years. Plus, she is retiring in 10 years, and that is long enough to not lose on the equity in the long run. She is looking for income as the first parameter, which is why we are stating the 40% in bonds, yet she wants appreciation, so 40% in stock. Therefore, 40%, 40%, 20% is the best of the answers given. [Module 18, Scenario-Based & Financial Statement Questions, Section 3.0]

What does the following transaction on the tape indicate? EGG pf Sld 40s/s 27 1/2.

400 shares of preferred stock were traded but reported out of sequence. The "pf" indicates preferred stock. The "s/s" indicates that this was in units of 10; therefore, 40 x 10 = 400 shares. The "Sld" means the transaction was reported out of sequence. This would never appear on the actual ticker tape, but you could see it on the test, as they are testing you on the s/s representing the 10 unit shares

hedge a long stock position with a short put.

An option investor might do all of the following EXCEPT:

Which of the following trades on the consolidated tape shows a zero uptick? A. 46.40. 46.40. 46.25. 46.15. 46.15 B. 46.40. 46.25. 46.15. 46.25. 46.25 C. 46.15. 46.25. 46.40. 46.25. 46.25 D. 46.50. 46.60. 46.50. 46.50. 46.40

46.40. 46.25. 46.15. 46.25. 46.25. This trade is an uptick. Remember, an uptick is when the previous trade is lower; therefore, this trade is higher. A zero tick is when the trades are at the same price. In the second choice, the uptick is from 46.15 to 46.25 and then the zero tick is the next trade, which makes it a zero uptick

Your customers are a couple in their late 50s. They have enough cash for five months of living expenses. They want to invest with you, but they do not want to lose their purchasing power. Which of the following would be the best investment strategy for them? A. 100% stock and 0% bonds B. 50% stock and 50% bonds C. 10% stock and 90% bonds D. 0% stock and 100% bonds

50% stock and 50% bonds. The key words in this scenario are purchasing power. This indicates that they want appreciation from their investment to keep up with inflation. They have not reached retirement age -- so they do not need the income from their investment at this time -- but they can't afford to invest entirely in stocks. Of the four choices, 50-50 would be the best investment strategy. If they were closer to retirement age, they could invest 10% in stock and 90% in bonds. If they were young, all of the investment could be in stock.

EZ Sleep Beds, Inc. uses the cumulative voting method of electing directors. In a race where eight candidates are vying for five spots on the board, how many votes could a shareholder who owns 100 shares of stock vote?

500 total votes among the five new directors to be elected in any manner chosen by the stockholder. Cumulative voting allows the shareholders to vote in any manner they choose with the number of shares they have times the number of directors being elected to the board. In this case, 100 shares x five directors allow this shareholder to cast 500 votes (100 votes x 5 open director positions). These votes can be cast in any manner the shareholder wishes.

Which of the following single individuals with an adjusted gross income of $40,000 may not make a tax-deductible contribution to an IRA? A. A self-employed doctor who already has a self-employed defined contribution plan B. An employee of an accounting firm who is already covered under a company pension plan and has a part-time job doing income tax returns C. An employee of a corporation who is not covered under her company's pension plan D. A 55-year old person who has retirement income of $35,000 and has interest and dividend income of $5,000

55-year old person who has retirement income of $35,000 and has interest and dividend income of $5,000. While this person has income of $40,000, none of that income is earned income — a person must have earned income to make a tax-deductible contribution to an IRA. All of the other persons have earned income, and can make a tax-deductible contribution to their IRAs.

The maximum amount a 54-year-old individual can contribute yearly to an IRA is:

6,500 or 100% of their yearly income, whichever is less. The maximum amount an individual who is 50 years old or older can contribute yearly to an IRA is $6,500 or 100% of their yearly earned income, whichever is less. If the person were under 50 years of age, the answer would be $5,500 or 100%, whichever is less. The contribution can always be made, but it may not always be tax-deductable. If the person has a retirement plan, his or her income would have to be given to know whether the contribution is tax-deductible or not. Always remember not to include passive income in yearly earned income calculations.

An engineer covered under a pension plan retires. She plans to do some part-time consulting work. The engineer receives her retirement benefit in a lump-sum payment. To avoid paying taxes on the lump-sum payment, she would have to transfer (roll over) the money into an IRA within:

60 days. To avoid paying taxes on the lump-sum payment, she would have to roll over the money into an IRA within 60 days. The rollover (change in trustees) must be completed within 60 days of the funds being received. She does not need to transfer all of the money, but she will be taxed on the withdrawn amount

a closing purchase

An option writer liquidates a position by purchasing an option. This order must be marked as:

A 9% Treasury bond with interest payment dates of June and December 1 is purchased on Monday, August 12. How many days worth of accrued interest must the buyer pay the seller?

73 days. Since this is a government bond, two things can trip you up. First, it settles in one business day -- there is no weekend, so it is just one calendar day. Then, because some months have more than 30 days, you must add one day for every month that has 31 days (it is assumed that each month has 30 days). Since July has 31 days, add one day when you subtract the interest date from settlement date: 8/12 trade date + 1 regular way, no weekend 8/13 settlement date - 6/1 last interest date 2/12 (months and days, assuming a 30-day month) = 60 + 12 = 72 + 1 = 73

A 9% U.S. Treasury note is selling at 102.4. What is its current yield?

8.81%. To calculate this, first find the price of the bond. Write down the dollar quote with line, period, line, line: 102_ ._ __ and then fill in with the decimal equivalent of 4/32 or .125, which becomes 1021.25. Then to find current yield, divide the nominal yield by the price of the bond. Nominal yield is 9%, or $90 per bond. Therefore, 90 divided by 1021.25 = .08812, or 8.81%.

An investor buys a 90 put for 14 and later sells an 80 put for 7. At what point is the profit from the 90 put going to be offset dollar for dollar by the loss from the 80 put?

80 1.These are puts and the investor has purchased the put at 90. The investor would like the price of the stock to drop below 90, because for every point the stock drops, the investor makes more money. However, the investor has also sold an 80 put, which means that if the stock falls below 80, the investor could be exercised. When the price of the stock drops down to 80, the investor stops gaining on the 90 put, because the investor's 80 put is exercised at this point. The maximum gain is at 80. The best way to look at this is if no premiums were involved. If the stock is at 85, the investor could make 5 on the 90 put. At 81, the investor could make 9. However, if the price of the stock drops to 75, the investor can make 15 on the 90 put, but will lose 5 on the 80 put when exercised. The investor would stop making additional money when the price of the stock reaches 80. Ignore the breakeven point.

A Nasdaq market maker shows quotes and sizes for a stock at 45.30 - .70 12 x 14. A trader who wants to buy 800 shares for a customer would purchase how many shares at which of the following prices? A. 100 shares at 45.70 B. 800 shares at 45.30 C. 800 shares at 45.70 D. 1400 shares at 45.70

800 shares at 45.70. When buying from a Nasdaq market maker, traders can get all of the shares they want, if the market maker has the size showing. They always buy at the high price, called the off

A Nasdaq market maker shows quotes and sizes for a stock at 45.30 - .70 12 x 14. A trader who wants to buy 800 shares for a customer would purchase how many shares at which of the following prices?

800 shares at 45.70. When buying from a Nasdaq market maker, traders can get all of the shares they want, if the market maker has the size showing. They always buy at the high price, called the offer

An investor purchases $20,000 of a 9% government bond that matures on April 15, 2028. The customer buys the bond on Monday, July 9, 2012 in a regular way transaction. For how many days of accrued interest must the buyer pay the seller?

86 days. Remember that this is a government bond and that actual days are counted. Using the formula below, the 31-day months (in this case, May) require that an additional day be added to the calculation. (Two days would have had to have been subtracted from the calculation if February were part of this calculation.) Also remember that the interest dates are the maturity date and the date six months prior. In this case, 4/15 is the maturity, so the other interest is paid on 10/15. Here is the calculation: 7/9 trade date + 1 regular way, no weekend 7/10 settlement date - 4/15 the previous settlement date = 2/25 2/25 indicates 2 months and 25 days, which equals 85 days. Finally, add another day for May: 85 + 1 = 86 As a shortcut, you can use the last digits of the trade date and settlement dates to confirm your choice: Take 9 from the trade date; add 1 to arrive at 10. Then subtract 5 (the last digit of the settlement date) from 10. Add 1 to adjust for the month of May. You now have 6. Select the only answer ending in 6.

A corporation has issued a bond with a 12% coupon that is convertible into common stock at $40. The stock is presently selling for $35. At what price would the bond be selling at a premium?

89 This is a parity question. Use the formula, and find the parity price: NUMBER OF SHARES x SHARE PRICE = BOND PRICE 1. ?? x $40 = 1,000 25 x 40 = 1,000 2. 25 x $35 = ?? 25 x $35 = $875 per bond Since a bond quote of 87.5 is the same as 875, the only number greater than this is 89. Remember that premium and discount in convertible bonds are to parity, not par. Also, when asked at which price the bond is selling at premium, ALWAYS choose the highest price given. There cannot be two answers with prices above parity.

85 dollars. a single point is equal two 100 dollars there .85 is 85 dollars

A call option premium increases in price by .85. What is the dollar amount of that increase?

85.

A call option premium increases in price by .85. What is the dollar amount of that increase?

issuer pays above par to redeem the bonds early.

A call premium is best described as the amount the:

under no circumstances.

A customer requests information on a new mutual fund and asks her registered representative to circle the important information in the prospectus and information he thinks will be of special interest to her. This is permitted:

Systematic.

A portfolio manager using index options is trying to reduce which of the following types of risks?

they are always directly backed by the federal government.

All of the following statements regarding government and agency securities are true EXCEPT:

All of the following would be a good recommendation for a person who desires to preserve capital and generate a stream of income, except: A. 20-year U.S. government bond B. AA-rated 10-year municipal bond C. A 15-year zero-coupon corporate bond D. A money market fund

A 15-year zero-coupon bond. The investor wants to preserve capital first, and then produce a stream of income second. The money market fund will only generate a small stream of income; however, the capital will be there. The zero-coupon bond has no income, even though the corporation could be solid and the investment is safe. Of the choices, the zero-coupon bond has the least chance of income and the least chance of preservation of capital. [Module 18, Scenario-Based & Financial Statement Questions, Sections 2.1 - 2.4]

Your new customer is about to retire. She has just inherited $30,000 and wants to supplement her retirement income. She receives Social Security benefits of $4,000 per month. Which of the following investments would you recommend to her? A. A 25-year high-yield corporate bond B. A 20-year AA-rated municipal bond C. A short-term money market investment D. A long-term Treasury receipt

A 25-year high-yield corporate bond. Your customer wants to supplement her income. Her current income appears adequate, and the corporate bond would not involve too much risk. The amount of income from Social Security may reduce the tax advantages of the municipal bond. The Treasury receipt will not provide any income but will pay interest upon maturity. Short-term money market investments generally pay less interest than bonds.

Which of the following single individuals with an adjusted gross income of $40,000 may not make a tax-deductible contribution to an IRA? A. A self-employed doctor who already has a self-employed defined contribution plan B. An employee of an accounting firm who is already covered under a company pension plan and has a part-time job doing income tax returns C. An employee of a corporation who is not covered under her company's pension plan D. A 55-year old person who has retirement income of $35,000 and has interest and dividend income of $5,000

A 55-year old person who has retirement income of $35,000 and has interest and dividend income of $5,000. While this person has income of $40,000, none of that income is earned income — a person must have earned income to make a tax-deductible contribution to an IRA. All of the other persons have earned income, and can make a tax-deductible contribution to their IRAs.

A customer of yours calls and says that a friend of his has been purchasing collateralized mortgage obligations, which he calls CMOs. Your customer asks, "What is a CMO?" You tell him which of the following? A. A CMO is a security that is part of a loan for mortgages on real estate property and the mortgages are backed by the Public Housing Authority. B. A CMO is a security that is part of a loan for mortgages on real estate property and the mortgages are backed by the homes of the people who borrow the money. C. A CMO is a security that is part of a loan for mortgages on real estate property and the mortgages are backed by the underlying issuer. D. A CMO is a security that is a direct obligation of the U.S. government.

A CMO is a security that is part of a loan for mortgages on real estate property and the mortgages are backed by the homes of the people who borrow the money. The actual CMO securities are backed by the issuer; however, that issuer (usually a broker/dealer) has purchased GNMA or FNMA mortgage securities from which the issuer has made the CMO. These mortgage securities are backed by the real estate property for which the loan is made, and thus are an obligation of the people who have taken the loan. The backing is like a sponge in that the investor in the CMO has purchased securities that are backed by the B/D issuer; the B/D issuer is backed by the GNMA/FNMA purchased by that B/D; the GNMA/FNMA is backed by the bank or other entity giving the loan, which is backed by the mortgages underlying the loan

DNR Order

An order that indicates there should be no adjustment for a dividend payment is:

A customer calls and says that a friend of his has been purchasing collateralized mortgage obligations, which his friend calls CMOs. Your customer asks you, "What is a CMO?" You tell him which of the following? A. A CMO is a security that is part of a loan for mortgages on real estate property and the Public Housing Authority backs the mortgages. B. A CMO is a security that is part of a loan for mortgages on real estate property and the mortgages are backed by the homes of the people who borrowed the money. C. A CMO is a security that is part of a loan for mortgages on real estate property and the underlying issuer backs the mortgages. D. A CMO is a security that is a direct obligation of the U.S. government.

A CMO is a security that is part of a loan for mortgages on real estate property and the mortgages are backed by the homes of the people who borrowed the money. CMOs are not guaranteed unless they are issued by GNMA, and they are not a direct obligation of the U.S. government. They are an obligation of the issuer, but they are ultimately backed by the mortgages, and thus by real property. [Module 3, Government Securities, Section 8.1]

may trade for his own account

A Designated Primary Market Maker on the floor of the CBOE

for the purpose of resale at a later date.

A FINRA member broker/dealer trading in shares of an open-end investment company may NOT buy shares of the fund:

Which of the following statements is true of a TAC (Targeted Amortization Class) mortgage obligation? A. A TAC is a just like a PAC, except it has a higher degree of prepayment risk. B. A TAC is a just like a PAC, except it has a higher degree of extension risk. C. A TAC is a just like a PAC, except it has a lower degree of prepayment risk. D. A TAC is a just like a PAC, except it has a lower degree of extension risk.

A TAC is a just like a PAC, except that it has a higher degree of extension risk. The difference between a PAC and a TAC is that PACs have prepayment and extension companion tranches, while TACs only have prepayment companion tranches. Therefore, TACs have a higher degree of extension risk because there are no companion tranches for late payment.

tax credit on the investor's U.S. tax return.

A U.S. citizen owns stock in a Canadian company and receives dividends. The Canadian government withholds 15% of the dividends as a tax. As a result, the investor reports a:

balance sheet

A bona fide regular customer, upon request, may inspect a broker/dealer firm's

140% of the debit balance in a customer's margin account.

A broker/dealer can rehypothecate (repledge) up to:

Commingling

A broker/dealer has put customer's securities in its own proprietary accounts. This violation of the industry is known as:

The excess of securities above 140% of the debt balance

A broker/dealer must segregate what percentage in a customer's margin account

7

A broker/dealer selling open-end investment company shares will be required to return its entire selling concession if the principal underwriter has to repurchase those shares from a customer within how many business days of trade date?

7.

A broker/dealer selling open-end investment company shares will be required to return its entire selling concession if the principal underwriter has to repurchase those shares from a customer within how many business days of trade date?

within 7 business days of purchase.

A broker/dealer with a sales agreement with a mutual fund must return all concessions to the underwriter if a customer redeems his shares:

at or above the stop price

A buy stop is triggered where

What causes the price of PAC CMO to change?

A change in the interest rates. Since the price is tied to the yield of the CMOs, the price has to change with movement in interest rates. The life expectancy of the CMO only changes as interest rates go up and down, causing more or fewer people to pay off mortgages, and thus changes in the prepayment schedule. Life expectancy may have some bearing, but the main change in value is due to interest rate changes and the yields. [Module 3, Government Securities, Sections 4.0 & 8.1]

What causes the price of a PAC CMO to change? A. The underlying value of the CMO B. A change in the interest rates C. A change in the average life expectancy D. A change in prepayments of the underlying mortgages

A change in the interest rates. Since the price is tied to the yield of the CMOs, the price has to change with movement in interest rates. The life expectancy of the CMO only changes as interest rates go up and down; interest rates affect whether or not homeowners pay off their mortgages early, which in turn affects the prepayment schedule. Life expectancy may have some bearing, but the main change in value is due to interest rate changes and the yields. [Module 3, Government Securities, Sections 9.0 - 9.2]

A trader is considering a particular stock. Which of the following could cause a change in alpha of that particular stock? A. A change in the earnings of the company B. A change in the management of the company C. A change in stock market sentiment D. A change in the support or resistance levels of the company

A change in the management of the company. Alpha is the change that occurs due to a change in management of the company, direction of the company, or any other aspect that changes the company outlook. All of the others affect the beta of the stock, as they are all part of the market.

the disbursement of principal and interest payments must be approved semiannually by the state public service commission.

A city has issued bonds to construct a new sewage treatment facility. If the bonds are not backed by the full taxing authority of the city, all of the following statements about the bond issue are true EXCEPT:

TANs.

A city's day-to-day operational expenses may be met by the issuance of:

It loses its original position.

A client asks his broker to enter a day order to buy 100 shares of COW at 36. Later in the day, he changes the order to a GTC order at the same price and for the same number of shares. What happens to the position of the order on the order book?

On July 1, 2010, an investor purchases $25,000 worth of XYZ, Inc. zero-coupon bonds maturing on August 1, 2020. At the time of purchase, the bonds were quoted at 50. One year later, the investor sells the bonds when they are quoted at 60. How is the investor taxed on the appreciation?

A combination of ordinary income and capital gain. The bond is a zero-coupon bond, so part of the appreciation is the interest. This interest income is taxed as ordinary income. Since these are 10-year bonds (2010 to 2020), the $500-per-bond discount (quote of 50) is accreted over the 10 years, or $50 per year. One year later, the bond's accreted value is $550 per bond. Since the bond is sold at 60, or $600 per bond, there is $50 per bond appreciation that becomes a capital gain. Therefore, the investor has a combination of ordinary income and a capital gain.

The offering may not be advertised

A company is offering a private placement with the intent of selling shares to nonaccredited investors up to the 35 allowed for in Regulation D. Which of the following is TRUE?

has had $100,000 in undistributed profits since its inception

A company's balance sheet dated December 31 shows retained earnings of $100,000. You can deduce from this information that the company:

on or before the settlement date.

A confirmation of each customer trade must be given or sent:

Which of the following is considered funded debt? A. A Treasury note B. A corporate bond C. Commercial paper D. A municipal bond anticipation note

A corporate bond. Funded debt is any long-term debt issued by a corporation or a municipality. The bonds issued by the U.S. government really are funded debt, but are usually considered Treasury debt. Treasury notes also appear to be funded debt, since the maturities are for more than a year. However, any debt issued by the U.S. government is not included in the definition of funded debt. Municipal notes, however, are short-term borrowing, usually in anticipation of some other funds coming in -- the municipal bond anticipation note is a short-term note while the bond is being issued. Commercial paper is short term, and although many corporations are funding debt this way, it is not "funded debt"; rather, it is short-term debt that matures in less than one year and is not part of the long-term liabilities of the corporation. Funded debt is generally considered any debt longer than one year.

rights offering.

A corporate offering of 200,000 additional shares to existing stockholders may be made through a:

trust.

A corporate profit-sharing plan must be set up under a(n):

What is a REIT? A. An investment company B. A general partnership C. A limited partnership D. A corporation

A corporation. A REIT is a special type of corporation with special tax treatment. REIT stands for real estate investment trust. It is a publicly held corporation that passes 90% of its net investment income through to the investors. Seventy-five percent of the income must be derived from real estate-related sources. Shares in a REIT are traded similarly to stock

obtain approval from FINRA

A customer asks her registered representative to exercise discretion over her account. To do so, the representative must do each of the following EXCEPT

accreted and is not taxed.

A customer buys a new issue municipal bond at a discount. If held to maturity, the amount of the discount is:

70,000.

A customer buys a real estate limited partnership interest by contributing $20,000 and signing a nonrecourse note for $50,000. The customer's beginning basis is:

YTC is higher than YTM.

A customer buys an 8% bond on an 8.20 basis. If the bond is callable in 5 years at par and matures in 10 years, which of the following statements is TRUE?

All of the following require prior written discretionary authorization, except: A. A customer sends in $50,000 and tells his registered rep to buy whatever she thinks is a good buy at this time. B. A customer calls and tells his registered rep to buy as many shares as she thinks are adequate of Qualcom, Inc. in the next day or so. C. A customer calls and tells his registered rep to buy as many shares as she thinks are reasonable of an East Coast utility company. D. A customer calls and tells his registered rep to buy 300 shares of Tri-com, Inc. at whatever price she feels is reasonable.

A customer calls and tells his registered rep to buy 300 shares of Tri-com, Inc. at whatever price she feels is reasonable. Think "time and price" as the only criteria that a representative can act on without written permission. For this question, the only answer that allows the representative to use her discretion is the 300 shares of Tri-com, Inc. All of the other answer choices involve discretion as to the amount of a security or the security itself, which requires prior written permission. Discretionary orders require that the representative have prior written discretionary authority to make the trade.

partially a tax-free return of capital and partially taxable.

A customer has a nonqualified variable annuity. Once the contract is annuitized, monthly payments to the customer are:

10 business days.

A customer has filed a serious complaint against your firm and is threatening to take the firm to court. When informed that he has signed a predispute arbitration agreement, he demands to see a copy of it. How long does your firm have to supply the customer with a copy of the signed agreement upon receipt of his request?

Dividend treatment

A customer is considering adding a real estate investment trust (REIT) to their portfolio. They list all of the following as "plusses" or advantages. You correct your customer and point out that one of them is not an advantage of investing in REITs. Which of the following is NOT an advantage of investing in REITs?

common stock

A customer must present a signed representation letter stating that he is not a restricted purchaser prior to buying a new issue of:

Which of the following best describes the definition of a customer's limit order to sell? A. A customer order to buy stock when the market price reaches or falls below a support level B. A customer order to sell stock when the market price reaches or moves above a resistance level C. A customer order to sell stock when the market price reaches or falls below a support level D. A customer order to sell stock when the market price reaches a support level but does not fall below the support level

A customer order to sell stock when the market price reaches or moves above a resistance level A limit order to sell means the customer wants to sell the stock when the price is increasing and reaches a specified amount. A resistance level is a term used in technical analysis to indicate the stock may not rise above that value, but will drop below it instead. In this case, the investor wants to maximize her gain and sell before the price drops. An order to sell with a stop is used when the market is decreasing and the investor wants to limit possible losses if the stock drops below a certain value. The answer choice that includes a buy order is not relative, since the question asks about a limit order to sell. The answer choice describing when the market reaches or falls below the support level would indicate a low point for the price of a stock. The support level indicates that the stock will rise from that level.

general obligation bond.

A customer purchased a full faith and credit bond. This bond would be known as a:

YTM.

A customer purchases New York State GOs at a discount in the secondary market. The bonds mature in 10 years and are callable in 5 years at par. Under MSRB rules, the customer's confirmation will show

short-term government bonds

A customer pursuing income using a defensive investment strategy while avoiding volatility would be most interested in:

average annual net assets.

A mutual fund's expense ratio is found by dividing its expenses by its:

Class A shares. For initial purchases, breakpoints are only available if the customer purchases Class A shares, which are sold with a front-end load deducted from the initial investment. A substantial purchase can often reduce the sales charge to zero. Class-B and Class-C shares are sold with annual 12b-1 fees as well as a contingent deferred sales charge. Class D shares are sold with a level sales load plus a redemption fee.

A customer seeks a significant long-term investment in the Ajax fund, a growth-oriented mutual fund. To take advantage of breakpoints applicable to large investments, the customer should purchase:

The settlement

A customer should receive a current option disclosure document before or at the date of

Buy stop at 14.25.

A customer sold 100 shares of QRS short when the stock was trading at 19. If QRS is now trading at 14, and he wants to protect his gain, which of the following orders should he place?

place the order as instructed.

A customer with no other mutual fund investments wishes to invest $47,000 in the XYZ Technology Fund. If the Class A shares are eligible for a breakpoint sales charge discount at the $50,000 investment level, the action least appropriate for an agent is to:

Yield to maturity

A customer's confirmation for a municipal bond callable at par and quoted higher than the nominal yield would show:

$5 per $1,000 of par.

A dealer that quotes a concession of ½ to another dealer means:

A subscriber to Nasdaq Workstation II, Level 3 is which of the following? A. A firm that is only able to see the bid and ask of stocks on Nasdaq B. A firm that is only able to see trades as they occur on Nasdaq, but not enter any prices C. A firm that is only able to see all bid and ask prices and the inside price of Nasdaq stocks D. A firm that can change a quote for a stock that is shown on Nasdaq

A firm that can change a quote for a stock that is shown on Nasdaq. Level 3 is used by Nasdaq market makers. Market makers can also see all bid and ask prices. Level 1 access only allows a representative to see the best bid and ask. Level 2 can see all bid and ask prices. Level 3 can see the prices and change their firm's quotes.

An investor writes an in-the-money OEX option for a $500 premium. What is the investor's margin call? A. The market value, less the amount it is in the money B. The amount it is in the money, plus $500 C. A fixed percentage of the underlying value D. A fixed amount based on the premium

A fixed percentage of the underlying value. The margin for uncovered calls is based on the market value of the underlying option -- in this case, the index (a number). Therefore, the margin call equals the percentage times the number times the multiplier. The margin requirement adds the premium to it; however, since the premium is received, there is no concern for the investor until the index changes and the premiums change. Then it will affect the writer. [Module 9, Options, Section 10.0]

private equity

A fund is investing capital raised from both institutional and private investors in a nonpublic company to assist the company with an anticipated acquisition. This type of investment would be known as

a prime account

A hedge fund has contracted with your broker/dealer to handle all of its clearing functions and provide all back office support functions while it is executing transactions through numerous other broker/dealers whom your broker dealer will have agreements with. This type of account is known as

A registered rep has designed a Web site that lists her qualifications, the firm with whom she is employed, and her association with a broker/dealer that is a member of FINRA. Which of the following is a requirement on the Web site? A. A hyperlink to FINRA's home site B. FINRA's logo C. A hyperlink to a third party's Web site D. The amount of commissions for trades

A hyperlink to FINRA's home site. The name of FINRA is not required, nor is the logo; however, if either is on the site and is used to enhance the qualifications or other aspects of the rep's ability, then a hyperlink to the home site of FINRA is required. Commission charges can be shown, but are not a requirement. Third-party Web sites may be offered but are also not required.

capital losses can be used to offset capital gains only

A married couple who files jointly has a $5,000 long-term capital loss with no offsetting capital gains. Regarding the tax treatment of this loss, all of the following statements are true EXCEPT:

with the customers' written permission.

A member firm may commingle the securities of two or more customers:

Which of the following does not require the approval of a firm's general securities principal? A. A memo to all registered representatives in the office marked "For Internal Use Only" B. A letter to a customer recommending a purchase of a particular security C. A letter to 10 customers recommending that they buy the same stock D. A letter from a registered rep to all her customers

A memo to all registered representatives in the office marked, "For Internal Use Only." Internal memos are not required to be approved by a general securities principal before they are used. A registered principal must approve all letters, advertisements, and other communication to customers.

One of the Federal Reserve Board's methods for controlling the money supply in the United States is changing the reserve requirement. As the economy tightens, the Federal Reserve Board recommends that banks keep more than the normal requirement in bank reserves. If the Federal Reserve Board decreases the reserve requirement, the effect on the economy will be which of the following? A. A long-range effect on the economy B. A straight-line effect on the economy C. A minimal effect on the economy D. A multiplier effect on the economy

A multiplier effect on the economy. Because most money put into banks as savings is used at least seven times over, any increase or decrease in reserve requirements affects the amount of money in circulation

Yield to the 30-year maturity

A municipal bond is offered at a discount. It has a 30-year maturity and is callable in 20 years at par. It is callable in 5 years at a premium and is puttable in 10 years at par. Which of the following yields would be quoted on this basis?

A bond at a premium callable at par A bond at a premium callable at par

A municipal bond subject to a refunding call must be quoted at yield-to-call in which of the following instances?

What characteristic of a municipal bond is most appealing for a customer interested in liquidity? A. A municipal bond with a maturity of two years or less B. An AAA-rated bond that is callable at a premium to par C. A revenue bond selling at a premium to par D. An AA-rated bond selling at a discount to par

A municipal bond with a maturity of two years or less. When the question asks for liquidity, always choose short term. Why would investors want liquidity? So they aren't locked into a long-term debt issue if interest rates are low. Investors wanting liquidity would choose short-term securities. The ratings and types of bonds have nothing to do with it. [Module 5, Municipal Securities, Sections 2.1 & 2.2]

an employee of a broker/dealer engaged in municipal security representative activities other than retail sales or who solicits municipal securities business for the broker/dealer

A municipal finance professional (MFP) is

on a competitive basis.

A municipal issuer publishes an official notice of sale to indicate that the offering will be made:

bond contract.

A municipal revenue issue's flow of funds statement is contained in the:

bond is likely to trade at a discount in the secondary market when it is puttable.

All of the following statements regarding a 6% municipal bond that is puttable at par are true EXCEPT the

A money market mutual fund invests in all of the following, except: A. A non-negotiable certificate of deposit B. A banker's acceptance C. Commercial paper D. A repurchase agreement

A non-negotiable certificate of deposit. A mutual fund does not invest in non-negotiable CDs. Money market funds invest in negotiable CDs, which are very short-term (14 to 30 days), but not non-negotiable CDs. Non-negotiable CDs are long-term investments that may charge surrender fees for early withdrawal. Investment companies invest in commercial paper, banker's acceptances, and repurchase agreements, especially if the fund is a money market fund.

Under SEC Rule 15c 2-6, which of the following is considered a designated security? A. A nonlisted stock selling for over $5 per share B. A listed stock selling for under $5 per share C. A nonlisted stock selling for under $5 per share D. An OTCBB-traded stock selling for under $10 per share

A nonlisted stock selling for under $5 per share. Don't let the rule number scare you -- what the question is referring to is "designated securities." The statement "Under SEC Rule 15c 2-6" could be left out of the question, and the answer would not change. Ignore references to rules, just look to see what is being asked. In this case, it is "designated securities" which are nonlisted stock selling for less than $5 per share. That is the answer, so don't fret about the rule number. Just remember designated securities are cheap stock.

The city of Phoenix, Arizona has published an advertisement stating that it is willing to accept bids for the sale of $20,000,000 of sewer bonds. The advertisement is called:

A notice of sale. When the advertisement is from the issuer, it is a notice of sale. When it is from the underwriters, it is a tombstone. The official statement and indenture are informational pieces, not advertisements.

A company has issued a 12.5% debenture bond convertible at $40 that carries an anti-dilution clause. The company declares a 10% stock dividend. At the distribution of the dividend, what would the debenture holders receive?

A notice saying the conversion price has been reduced to $36.36. There are two different ways to do this problem: An easy way and a more difficult way. The easy way uses logic: Since there is a stock dividend, you know the price will be reduced, not raised. The correct answer (A) is the only one possible since debenture holders do not get a certificate of stock for any shares (they are not stockholders), nor any money (for the same reason) other than their interest. The other way to solve this is using the conversion ratio. Remember, this takes the usual two steps to find parity. The first step finds the conversion ratio of 25 shares per bond: Shares x Share price = Bond price _____ x 40 = 1,000 25 x 40 = 1,000, now increase the shares 10% to 27.5 shares per bond The second step finds the new conversion price for the same $1,000 bond. 27.50 x ___ = 1,000 27.5 x 36.36 = 1,000

Under the investment Advisers Act of 1940, which of the following persons must be registered with the SEC as an investment adviser? A. A person who gives advice to his clients on which stocks to buy and what is best for the individual's portfolio B. A person who charges a fee for executing transactions for a customer after recommending the securities C. A person who has many clients for whom he executes transactions for a fee and occasionally gives clients advice on the best securities for their portfolio D. A person who gives advice to a retirement plan of a corporation for a fee but has an associate broker/dealer execute the trades

A person who gives advice to a retirement plan of a corporation for a fee but has an associate broker/dealer execute the trades. Under the 1940 Act, a person who gives advice to retirement plans, investment companies, and other large investors and charges a fee for the advice must register with the SEC. Whether the person executes the transactions or not has no bearing on being an investment adviser. Individuals who give advice but do not charge a fee for the advice, need not register with either the SEC or the state in which they are based. If the individual does charge a fee to clients and executes the transactions as well, he may have to register with the SEC if the amount managed for a fee is large enough. The individual who advises clients occasionally must register with the state in which the office is located, and usually in the state of the customer as well, but not with the SEC.

providing a limited power of attorney giving discretionary powers. A discretionary account always requires prior written authorization from the customer in the form of a limited power of attorney (trading authorization).

A person wishing to grant a registered representative the right to make investment decisions for his account does so by:

A corporation has a retirement plan for its employees. The corporation makes the contribution for the employees, who are 100% vested after three years. The retirement plan comes under the guidelines of ERISA, and an investment manager makes the investments. Can the ERISA covered retirement plan invest in options in a margin account? A. Any plan can always invest in options in a margin account. B. A plan may never invest in options in a margin account. C. The retirement plan can invest in options in a margin account with a vote of the employees in the account. D. The retirement plan can invest in options in a margin account, but only for specific companies.

A plan may never invest in options in a margin account. If the options have to be in a margin account, then the plan is writing uncovered options, which is not allowed. This is because there is only limited amount of money that can be contributed, and writing uncovered options has high risk and a great loss of capital. Options can be used in a retirement account, but only through the purchase of puts for protection or writing covered calls for income to the plan on the stock that is held. Voting has no bearing in this case. [Module 8, Retirement Plans, Section 9.0]

Thursday of that week.

A primary dealer has its bid on Treasury bills filled at the weekly auction. Settlement between the dealer and the Treasury will be:

An increase to retained earnings.

A profitable company distributes 70% of its earnings in the form of cash dividends. What is the effect on the balance sheet of the 30% earnings NOT distributed?

Which of the following corporations would most likely issue the most in debt securities? A. A textile company B. A public utility company C. An automobile company D. A chemical company

A public utility company. Public utility companies are more apt to issue bonds than stock. The bonds eventually mature, and they will have sufficient income to pay them off. If they issue stock, they now have more owners to share the profits, which they don't want. By borrowing money, they can raise rates for a reason, and then when the bonds are paid off, the rates are already high and there is no need to raise them

net asset value next computed after the fund receives the order

A purchase or redemption order for investment company shares must be executed at a price based on the:

his initial registration date

A registered person leaves the securities industry and 18 months later reassociates with another member firm. FINRA requires that this person's cycle for determining the dates for the regulatory element portion of continuing education be based on:

Any closed-end fund

A registered representative (RR) has just explained to a customer that to purchase a particular security the customer would pay the asking price plus a commission, not a sales charge. Which of the following is the RR is speaking of?

Random walk

A registered representative is explaining a particular market theory that maintains that the direction of a single stock or any general market is unpredictable. Which theory is he speaking of?

The corporation's charter, account resolution and by laws & the names of natural persons authorized to trade the account

A registered representative is opening both cash and margin accounts for a corporation. Which of the following documents will he need?

Direct participation program.

A registered representative must obtain written verification of an investor's net worth for which of the following investments?

moderate

A registered representative sits down with a new customer to complete the customer account form. During this time the customer expresses being comfortable with some risk to his initial investment in exchange for potentially higher returns. After the registered representative explains that the willingness to accept some risk may allow the account to keep pace with inflation but that it also means the account could lose value, the customer acknowledges that he understands. This customer's risk tolerance would best be defined as

2 years.

A registered representative who leaves the industry must requalify by examination to return to the industry if he is unaffiliated with a broker/dealer for more than:

remain unchanged.

A registered representative would recommend a customer establish a short straddle on T-bonds when interest rates are expected to:

the cost of goods purchased by consumers increased, on average, .3% for the month.

A report by the federal government stating the Consumer Price Index (CPI) has increased by .3% indicates:

commercial banks and federal reserve board

A repurchase agreement is usually initiated by which of the following?

commercial banks and the federal reserve board

A repurchase agreement is usually initiated by which of the following?

It is not subject to federal income tax.

A resident of New York City purchases an Albany, New York general obligation bond and receives $600 of interest from that bond during the year. How is that $600 taxed?

taking a position.

A securities firm that holds stock rather than sells the stock is:

The value of the securities at the time the charitable donation is made.

A security you purchased several years ago has appreciated substantially in value. You have decided to donate the investment to your favorite charity. What will the cost basis be to the charity that receives your gift?

rising and is at or above the limit price

A sell limit order is executed when a stock is:

A stock has been declining steadily over a long period of time with little volume. Suddenly there is a sharp decline on very heavy volume. A technical analyst would say the stock has experienced:

A selling climax. A technical analyst would say that the stock experienced a selling climax. This is the last step before a stock is expected to begin appreciating. The selling has just climaxed, the odd-lotters are getting out, and the big boys are ready and starting to move in. This is a bullish sign that the stock price will rise. [

the symbol "ss" on the consolidated tape.

A short sale of stock directed to an exchange must observe all of the following EXCEPT:

Buy stops.

A significant increase in which of the following types of orders may cause a bull market to accelerate?

the dividend must be declared by the board of directors.

A similarity between common and preferred stock is:

Which of the following is considered the safest investment among the choices presented? A. A three-month municipal RAN B. A six-month Treasury bill C. A triple-A rated corporate bond with 30 years to maturity D. A U.S. Treasury bond with 20 years to maturity

A six-month Treasury bill. To determine this answer by process of elimination, select those investments that are backed by the U.S. government. Of those, select the investment with the shortest time to maturity. Of course, shorter-maturity fixed income securities generally have lower yields. In this case, the T-bill is the shortest term for the two government securities given. Do not confuse municipal RANs (revenue anticipation notes) with government securities.

Whether the growth development company is traded or nontraded

A small retail investor wants to invest in a growth development company. Which of the following should be of the most concern regarding suitability?

accept a not-held order

A specialist (designated market maker) is permitted to do all of the following EXCEPT:

not held and good for a month orders

A specialist (designated market maker) must refuse:

Which of the following best describes a Reg A offering? A. A $2 million stock offering of securities in California only B. A private placement to no more than 35 individuals C. An offering of $7 million of corporate bonds D. A stock offering of $5 million or less

A stock offering of $5 million or less. Of the alternatives provided, this best describes a Regulation A offering. [Module 15, Federal & State Regulations, Section 2.4]

A registered rep sends a customer a bond appraisal for his holdings. The appraisal can be said to be which of the following?

A subject quote by the firm and the price the firm thinks the customer can get for the bond. Bonds are unique and difficult to appraise. The prices of bonds are also negotiated in the over-the-counter market, so the actual value of the bonds may not be determined until the bonds are sold. Of course, keep in mind that daily fluctuations in interest rates also affect the value of bonds. Therefore, the price quoted by the appraiser is always a subject quote.

corporate common and preferred stock.

A successful chain of retail stores in the maximum corporate tax bracket may exclude from taxation 70% of income earned on investments in:

Dow Theory

A technical market theory that long-term trends in the stocket market may be confirmed by analyzing the movements of Dow Jones industrial average and dow jones transportation average

A broker/dealer is a subscriber to the Nasdaq Workstation II. One of the screens at the firm is showing five different market makers and their quotes. The screen also shows the inside price, the high for the day, and the low for the day. None of the quotes can be changed from this location. Which of the following persons at the firm would be looking at this screen? A. The market maker for the stock B. A registered rep at her desk C. A trader at the order desk D. The principal of the office

A trader at the order desk. The trader gets to see all of the quotes, but cannot make any changes. If changes could have been made, it would be the market maker. Both the principal of the office (if she/he has a screen) and the registered reps who have screens do not get to see all the market makers' quotes and sizes. They only see the last trade, inside market, open, high and low for the day, and the close and volume. [Module 12, OTC, Section 6.2]

A variable annuity pays out:

A variable amount according to the value of the separate account. A variable annuity pays out just as the name indicates -- a variable amount. This amount is based on the value of the separate account and the value of the annuity unit in the separate account at the time of each payment. Fixed annuities pay out fixed amounts. The mortality rate has nothing to do with the variable amount paid out each month to the annuitant.

The Bond Buyer 20 Bond Index is? A. A weekly list of bonds in MuniFacts B. A daily list of 20 general obligation bonds C. A weekly list of 20 general obligation bonds D. Published monthly by the Bond Buyer

A weekly list of 20 general obligation bonds. The Bond Buyer 20 General Obligation Bond Index is determined on Thursday and published on Friday. There are three general obligation indexes: The 20-bond index, the 11-bond index, and the 40-bond index, all with bonds of 20 years to maturity. [Module 6, Underwriting, Section 5.3]

it is quoted as a percentage of par.

All of the following statements regarding commercial paper are correct EXCEPT:

purchases may not be made through the exercise of options

All of the following statements regarding corporate insiders are true EXCEPT

the general partner's signature grants the limited partners power of attorney to conduct the partnership's affairs.

All of the following statements are true with respect to a limited partnership subscription agreement EXCEPT:

corporate bondholders must amortize premium bonds annually. If an investor purchases a corporate bond at a premium, the investor has the option of whether or not to amortize the premium. Amortization reduces both reported interest income and cost basis.

All of the following statements about the taxation of municipal and corporate bonds are true EXCEPT:

they are most commonly offered with debentures to make the offering more attractive.

All of the following statements describe stock rights EXCEPT:

Why would an ADR be issued?

ADRs are used to facilitate trading of foreign securities on U.S. exchanges. ADRs are receipts issued in lieu of certificates of foreign stock. The stock is actually deposited in correspondent branches of U.S. banks in the foreign country, and then the receipts are traded in the United States. One receipt represents a specific number of shares of the underlying stock. These receipts aid in the transferring of ownership of the stock and thus "facilitate trading of foreign securities

A variable annuity provides: A. Participation in a managed portfolio of securities B. Tax-deferred income C. The assumption of risk by the annuity holder D. All of the above

A. Participation in a managed portfolio of securities B. Tax-deferred income C. The assumption of risk by the annuity holder A variable annuity provides participation in a managed portfolio of securities, tax-deferred income, and the assumption of risk by the annuity holder. The insurance company manages the separate account portion or invests these funds in other established mutual funds. In a variable annuity, the risk is borne by the annuity holder

100%. The corporate dividend exclusion permits a corporation receiving dividends from another corporation to exclude 70% of those payments. Therefore, the corporation will pay tax only on the remaining 30%. This exclusion applies only to dividends, not interest.

ABC Corporation owns stock in XYZ Corporation. What percentage of dividends paid by XYZ to ABC is taxable to ABC?

current yield per share has increased.

ABC's stock has paid a regular dividend every quarter for the last several years. If the price of the stock has remained the same over the past year, but the dividend amount per share has increased, it may be concluded that ABC's:

GNMAs are considered to be the riskiest of the agency issues

All of the following statements regarding Government National Mortgage Association (GNMA) pass-through securities are true EXCEPT

increases aggregate demand.

According to Keynesian economic theory, an economy's health can be ensured if the government:

provide quotes (while participating in a joint account) that indicate more than one market for the securities that are the subject of the account.

According to MSRB rules, a municipal securities dealer participating in a secondary market joint account may NOT:

A municipal securities dealer

According to MSRB rules, a separately identifiable division of a bank engaging in the sale and /or underwriting of securities issued by state and local governments is acting as a(n):

make market in NYSE listed securities from an off-floor location and make a one sided market

According to the multi-tiered market making system of NYSE Euronext, a Supplemental Liquidity Provider may:

Accumulation units are:

Accounting measures used to determine an individual's share in the separate account. Accumulation units are the units that are purchased at the time the investor is buying the annuity. These units are the value of the annuity at any time until the investor wishes to withdraw the money, at which time the accumulation units become annuity units. The annuity units determine the payout an investor will receive and are determined using the mortality tables, the payments desired, and the value in the account. Mutual funds have nothing to do with annuities.

actual days elapsed.

Accrued interest for U.S. government bonds is computed on the basis of:

Keynesian.

Active government manipulation of the economy through tax and budget policies is referred to as:

price of a market order

All of the following information must be on an order ticket EXCEPT:

A corporation that has no earnings in a particular year is obligated to pay the interest on all of the following, except: A. Equipment trust certificates B. Convertible subordinated debentures C. Collateral trust bonds D. Adjustment bonds

Adjustment bonds. Adjustment bonds are also called "income" bonds. A corporation issuing adjustment bonds is in financial trouble and only pays the interest if sufficient income is available to do so. Generally, adjustment bonds are used to replace previously issued bonds. When a corporation has financial difficulties and is unable to pay the interest associated with the existing bonds, the corporation requests that the current bondholders accept adjustment bonds, which offer the best chance that the investor's principal will eventually be repaid. The alternative is that the corporation could go bankrupt and be unable to pay back the principal of the bonds.

A technical indicator that measures the strength and breadth of the market by comparing the number of stocks that increase and the number of stocks that decrease is called the:

Advance-decline theory. The comparison of the number of stocks that increase to the number that decrease is the advance-decline theory. It charts the number of stocks that increase in price (advancing issues) over the number of stocks that decrease in price (declining issues). The advance-decline ratio is used to evaluate the market's performance and the breadth of stock price movements for the general stock market over a period of time. When the ratio is close to 1, the market may be flat. When the ratio is close to 0, the market may be declining; if the ratio is higher than 1, the market may be in a bullish trend.

3 years.

After a mutual fund's tenth year, performance statistics must show results for each of the following periods EXCEPT:

only allow the customer closing transactions

After being approved for options trading, a customer establishes several long option positions but does not return the signed option agreement within the required time period. The broker/dealer may:

When are contributions in a self-employed defined contribution plan vested for the employees?

After one year of employment. Contributions to a self-employed defined contribution plan for employees are vested after one year of service, no matter when the contributions are made. The employer must start contributing after one year of service, but if the contribution is made before the first year is up, the employer can hold off the vesting until one year of service is completed.

price of a market order.

All of the following information must be on an order ticket EXCEPT:

variable annuities.

All of the following investment strategies offer either fully or partially tax-deductible contributions to individuals who meet eligibility requirements EXCEPT:

Federal Home Loan Mortgage Corporation

All of the following issue securities under the Federal Farm Credit System EXCEPT:

not-held.

All of the following kinds of orders may be turned over to the specialist (designated market maker) for execution EXCEPT:

market orders.

All of the following orders could be placed on the specialist (designated market maker) order display book EXCEPT:

Debentures

All of the following securities are exempt from the Trust Indenture Act of 1939 EXCEPT:

are entered in cash and expire of the third Friday of the expiration month

All US exchange listed foreign currency options

completion of the options agreement.

All of the following actions must be completed prior to a customer entering his first option trade EXCEPT:

new issue primary market securities

All of the following are TRACE eligible and require reporting EXCEPT

an unidentified nominal quote

All of the following are allowable municipal dealer quotes EXCEPT

each note is secured by a single specified asset

All of the following are characteristics of exchange-traded notes (ETNs) EXCEPT

it protects against losses produced by fluctuations in the market

All of the following are characteristics of the Securities Investor Protection Corporation (SIPC) EXCEPT:

active secondary trading.

All of the following are characteristics of unlisted options EXCEPT:

TOD accounts.

All of the following are fiduciary accounts EXCEPT:

premium in a stock option.

All of the following are fixed option contract terms EXCEPT the:

setting initial margin requirements on exempt securities

All of the following are functions of the Federal Reserve EXCEPT:

setting initial margin requirements on exempt securities.

All of the following are functions of the Federal Reserve EXCEPT:

REITs.

All of the following are redeemable securities EXCEPT:

the amount of markdown or markup on a principal transaction.

All of the following are required by the MSRB on customer confirmations EXCEPT:

it provides a measurement of a range that the market may move in any given day

All of the following are true concerning a market's beta EXCEPT:

it provides a measurement of a range that the market may move in any given day. The beta is a benchmark and does not indicate anything about market movement as a whole. It only measures the movement of a particular security or portfolio as compared to the movement of the entire market.

All of the following are true concerning a market's beta EXCEPT:

it trades with accrued interest.

All of the following are true of negotiable commercial paper EXCEPT:

that it is carried as an asset on the balance sheet.

All of the following are true of stockholders' equity EXCEPT:

even though a fail to deliver has occurred and is still outstanding, FINRA mandates that the seller still be paid

All of the following are true regarding a fail to deliver EXCEPT

employees may use accumulated funds as collateral for a bank loan.

All of the following are true regarding nonqualified deferred compensation plans EXCEPT:

Sallie Mae. Sallie Mae is the Student Loan Marketing Association, which purchases student loans and packages them for the secondary market. The FNMA, GNMA, and FHLMC sell mortgage-backed securities.

All of the following are used to back collateralized mortgage obligations EXCEPT:

customer's marital status.

All of the following are used to determine the suitability of recommendations made to a municipal bond customer EXCEPT the:

the bonds are normally backed by the full faith and credit of the municipality.

All of the following characteristics regarding industrial development bonds (IDBs) are true EXCEPT:

retail communications that make a financial or investment recommendation

All of the following communications are exempt from filing with FINRA EXCEPT

the customer's ability to pay.

All of the following considerations apply to the 5% markup policy EXCEPT

security sold on an exchange at the market price buyers and sellers have established

All of the following could be a redeemable security under the Investment Company Act of 1940 EXCEPT a(n):

POP.

All of the following describe the price of a no-load mutual fund EXCEPT:

the dated date on a municipal bond that has been outstanding for two years.

All of the following information must be disclosed on a municipal bond confirmation of sale EXCEPT:

Your answer, no debt limitation is set by the issuing municipality ., was incorrect. The correct answer was: the maturity of the revenue bond will usually exceed the useful life of the facility being built.

All of the following statements regarding municipal revenue bonds are true EXCEPT:

a riskless transaction is not generally covered by the 5% markup policy

All of the following statements regarding the 5% markup policy are true EXCEPT:

appreciation of CMV.

All of the following will affect SMA in a short account EXCEPT:

stock dividends

All of the following would affect a customer's equity balance EXCEPT:

stock dividends.

All of the following would affect a customer's equity balance EXCEPT:

collection ratio. The collection ratio shows the percentage of property taxes that are actually collected. This would be relevant in evaluating GO bonds, which are backed by the taxing authority of the issuer. Revenue bonds, however, are backed by user fees, not taxes.

All of the following would be considered when evaluating a municipal revenue bond's creditworthiness EXCEPT:

preferred stock.

All of the following would be found in the money market EXCEPT:

principal repayment on recourse debt.

All of the following would flow through as a loss to limited partners EXCEPT:

An investor purchases $50,000 of an ABC, Inc. 10% bond that matures on September 1, 2019. The investor pays $900 per bond. If he holds the bonds until maturity, how will the appreciation be taxed?

All ordinary income. All gain of bonds purchased at a discount are treated as ordinary income. The IRS feels that the discount is to help make up for the low interest rate; therefore, the discount is like interest and taxed as interest. The taxpayer can claim part of it each year or can claim it all at maturity. Either way, it is taxed as interest income.

Rating agencies look at the net debt and the net direct debt of a municipality to determine the ratings of the municipality's bond issues. Which of the following will the rating agencies use to determine net debt?

All outstanding debt, plus any new debt issued, and any overlapping debt incumbent on the municipality divided by the municipal population. This is net debt. You must include overlapping debt to the municipality for the answer to be correct. Of course, the rating agencies never divide by the state population; the agencies are only concerned with that particular municipality for the ratings. If this had been net direct debt, then the overlapping debt would not be included. [Module 5, Municipal Securities, Sections 9.1 - 9.4]

whether the member firm has any position in the security must be disclosed

All research reports issued by a member firm must disclose certain information. Regarding those disclosures, all of the following statements are true EXCEPT

three years and for the first two years be in a readily accessible location

All retail communications should be retained by the member firm for at least a period of

Dealer

An over the counter market is considered what type of market?

You are informed that your most active trading customer has had her account restricted. Which of the following is true? A. She must deposit enough funds to satisfy Reg T. B. All future trades must be transacted as in a cash account. C. All sales will require the broker/dealer to withhold 50% of the proceeds. D. No trading will be allowed in the account.

All sales will require the broker/dealer to withhold 50% of the proceeds. Many people in firms refer to a frozen account as a restricted account. This is not true. A frozen account is when no trade can be made unless the money or securities are on deposit first. A restricted account is an account where the equity is less than the debt balance, and if any stock is sold, then the broker/dealer keeps 50% of the proceeds from the sale. If the account is not restricted, and the customer has SMA, she could keep more than 50% of the proceeds of the sale. It is a formality, but the account is still live and active. [Module 10, Margin, Section 4.2]

underwriting firms.

Alpha Securities is the managing underwriter for a new issue of 1 million shares of ABC common on a firm-commitment basis. If part of the ABC issue remains unsold and results in a loss, the loss will be divided proportionately among the:

All-or-none.

Alpha is the managing underwriter for a new issue of 1 million shares of ABC common. While Alpha has agreed to sell as much stock as possible in the market, ABC will cancel the offering if any portion of the stock remains unsold. This arrangement is known as what type of underwriting?

Under the Securities Act of 1933, which of the following must be registered with the SEC? A. An ADR trading OTC B. State of Alaska bonds C. Eurodollar bonds trading OTC D. U.S. Treasury securities

An ADR trading OTC. All the other securities are exempt from registration. U.S. Treasury securities are always exempt, as are state (including the state of Alaska) and city bonds. Eurodollar bonds are also exempt. The ADR represents shares of stock of a foreign company, and thus must be registered like any other company's stock.

Who of the following can take a short position in the stock of EGG, Inc.? A. An EGG officer who has 5% of EGG's outstanding shares B. An EGG employee who has 2% of EGG's outstanding shares C. An EGG stockholder who has 5% of EGG's outstanding shares D. The EGG comptroller who has no EGG stock

An EGG stockholder who has 5% of EGG's outstanding shares. The other three are employees of the company, and could have inside information. Although the investor is a minority stockholder, she is the only person in the choices who does not work for the company, and thus, the only person who can sell the stock short. [Module 15, Federal & State Regulations, Section 2.5]

$1.5 million.

An Eastern account underwriting of $100 million in municipal bonds is established. ALFA Securities agrees to underwrite 10% of the issue and sells out its allotment of $10 million. However, some of the other firms participating in the deal are not as successful, and $15 million of the bonds remain unsold. What is ALFA Securities' financial obligation?

Which of the following persons could have a Keogh plan for retirement? A. An OB/GYN doctor in a partnership with three other doctors B. A government employee who is retired C. A school teacher who coaches in the after school program D. A state worker who manages the use of state automobiles

An OB/GYN doctor in a partnership with three other doctors. Even though this is a partnership (it doesn't specify a limited partnership), then all income and costs flow through to the doctor. She is self-employed, and therefore can have a Keogh. All of the others are not self-employed, and therefore cannot have a Keogh. [Module 8, Retirement Plans, Section 8.4]

subject

An OTC quote that must be reconfirmed with the OTC trading room before a broker/dealer takes action is:

4 times a year.

An affiliate or insider holding unregistered shares can sell under Rule 144:

Selling securities from a minor's custodial account without the custodian's consent but with the beneficial owner's consent.

An agent taking which of the following actions would be committing a violation?

An investor purchases an 80 interest-rate call option for a premium of 3 when interest rates on 7.5%, 30-year, U.S. government bonds are trading at 92.21, to yield 8.15%. One month later, when interest rates increase to 8.45%, the investor exercises. What does the investor receive?

An amount of money equal to the difference between 80 and 84.5 x a factor of 100. Interest-rate options only settle in money x the factor of 100. The amount paid is the difference between the strike price and the interest rate on the date of exercise x 100. [Module 9, Options, Section 16.0]

cash flow

An analyst comparing revenues with expenses is most likely analyzing:

make delivery of the certificates accompanied by evidence of insurance, either on the face of the certificates or in a separate document.

An insured municipal bond is purchased by your client in the secondary market. After the sale, MSRB rules would require you to

investors buying long-term bonds and selling short-term bonds.

An inverted yield curve is the result of:

Asset Allocation

An investment adviser who switches among investment classes based on anticipated market changes

the fund's board of directors or outstanding shares.

An investment adviser's contract must be approved by:

variable rate demand obligations (VRDOs) or reset bonds

An investor anticipating a rise in interest rates would likely purchase:

they would require voter approval before a municipality could issue them

An investor has purchased a municipal certificate of participation (COP). COPs can be characterized by all of the following EXCEPT

buy a call.

An investor is short stock at 70. If the stock's market price is 40 and the investor anticipates the price will continue to decline, to hedge against a rise in the price the investor should:

Sales proceeds are adjusted for the stock.

An investor long 100 shares of stock writes a call against the long stock position. If the call is exercised and the investor must deliver the stock, which of the following tax consequences will occur?

a capital gain.

An investor purchases a corporate bond at par to yield 5.5% to maturity. If he sells the bond at a price equivalent to a 5% yield to maturity two years later, the investor incurs:

Which of the following situations is considered a short-swing profit under federal law? A. An investor purchases stock and within a two-week period shorts an option on the same stock (making a profit). B. An investor sells short stock and within two days covers the short position for a profit. C. An investor sells short stock and on the same day covers the short position for a profit. D. An investor purchases stock in a company with which he is also affiliated and sells the stock two months later for a profit.

An investor purchases stock in a company with which he is also affiliated and sells the stock two months later for a profit. Short-swing profit only applies to affiliates of a company who buy and sell their company stock within six months at a profit. [Module 15, Federal & State Regulations, Section 3.4]

inflation risk.

An investor purchasing long-term AAA rated bonds should be concerned most with:

mortgage bonds.

An investor seeking a high level of income combined with a moderate level of risk would purchase

a long coupon.

An official statement has a dated date of March 1, but the first interest payment is October 15. This most likely reflects:

is liable for federal taxes on its net income

An open-end investment company that does not distribute at least 90% of its net income:

the positions market attitude (bullish or bearish).

An option confirmation must include all of the following EXCEPT:

no, sometimes they are not

Are regulation T Extensions always granted

A director is selling part of the stock she owns in the company with which she is employed. Under SEC Rule 144, if your firm handles this trade for this insider, in which manner must your firm act to execute the trade?

As a broker. Any firm that sells securities for an insider that are not part of a public offering must act as a broker. Firms are prohibited from acting as an underwriter for the secondary sales of this type of security. Representatives and firms cannot solicit the sale of this stock. The 144 stock could be sold directly to a market maker, but the question asks how your firm can act in this transaction. If your firm was making a market in the stock, your firm could buy the stock from the insider, but it couldn't act on behalf of a market maker.

The Interbank Trading System is used for what?

As a market for U.S. deposits in dollars in European banks. The Interbank Trading System publishes bond quotes and can be used as a marketplace for Eurodollar bonds. The information in the trading system is used by corporations throughout the world and by municipalities in the United States.

A broker/dealer is a member of a syndicate that is underwriting a municipal bond. The investor is also a sponsor for a unit investment trust that invests in municipal bonds. Suppose the investor would like to put in an order for the unit investment trust to purchase some of the bonds. How would he enter the order?

As a member order at the takedown. This would be a designated order if the member were not the sponsor of the unit investment trust. Since the member is the sponsor, he must come last in the distribution to buyers. [Module 6, Underwriting, Section 5.11]

The rating on the issue decreases.

Consider a municipal bond issue that has been defeased. Which of the following statements is NOT true?

A young couple calls you up to open an account. In gathering their information, you find that they are not married but still want to have a joint account under JTWROS. In addition, they want to invest in small cap growth companies in foreign countries. What should you do?

Ask the couple about their situation, determine their suitability for the account and the reason for it, and then open the account. The most important aspect is to know your customer. There is no rule that requires two people to be married to open a JTWROS account. Without knowing their requirements, you would not know to refuse to open the account. The combination of the two answers is the best. [Module 11, Customer Accounts, Section 5.1]

Short call, short put

At expiration, if the market price of the underlying stock is the same as the strike price, which of the following positions would be profitable?

A broker/dealer is the lead firm in a municipal joint account for a new municipal revenue bond. In addition, the broker/dealer is in a control relationship with the municipality. At what point must the firm disclose its control relationship in writing to the clients who are purchasing the issue?

At or prior to the completion of the transaction. The firm must inform customers of the control relationship prior to entering the order. If such disclosure is not in writing, then a written disclosure must be made at or prior to the completion of the transaction

An existing customer shows an interest in opening an option account to transact in options. When must the investor receive the options disclosure document?

At the time the account is opened. The disclosure document has to be one of the first things given to the investor -- always pick the earliest time given. The OCC options disclosure can be given to the customer before the account is opened but must be delivered at least before the account is accepted or approved. The options agreement does not have to be signed for 15 days after the account is opened, so the customer must have the options disclosure document before that time. [

An investor buys 700 shares of TDY in a cash trade on Monday, August 19. When is the settlement date for payment of the stock?

August 19. Purchasing in a cash trade or on a cash delivery basis means that cash and securities are exchanged on the same day. To complete a cash trade, the money (or stock, if selling) must be in the account by 2:30 p.m. Eastern Time or a half hour after the trade if the trade is after 2:00 p.m. Eastern Time

On which of the following would an investor look at the MIG rating? A. Project notes B. IDRs C. Treasury bonds D. BANs

BANs. BANs (bond anticipation notes) have MIG ratings. MIG ratings 1, 2, 3, and 4 are for municipal notes. The other municipal notes that have MIG ratings are RANs (revenue anticipation notes) and TANs (tax anticipation notes).

Most municipal bond investors and many of the tax-free mutual funds prefer bonds with certain ratings. In general, the tax-free mutual funds prefer to invest in bank-qualified issues. What is the minimum rating for a bank-qualified municipal bond?

BBB. To be bank qualified means that trust departments of banks can invest in the bonds. Many tax-free mutual funds and closed-end investment companies only invest in BBB or higher. The only mutual funds that invest in issues other than bank-qualified plans are "high" yield tax-free bond funds -- they invest in nonrated municipal bonds

Which of the following is a means of financing foreign trade? A. Certificates of deposit B. American Depositary Receipts (ADRs) C. Banker's acceptances D. Commercial paper

Banker's acceptances. A banker's acceptance is used to finance foreign purchases of goods. Certificates of deposits are issued by banks for depositors, commercial paper is issued by corporations, and an American Depositary Receipt is used to trade foreign stock on U.S. exchanges

Below is a section of Standard & Poor's Quarterly Dividend Record relating to EGG: Declaration Payable Dividend date Ex-date Record date date $0.60 Q January 8 January 27 January 29 March 15 An owner of EGG who sells the stock on January 27 regular way would:

Be entitled to the dividend. The owner is entitled to the dividend because the transaction occurred on the ex-dividend date or later. The seller still owns the dividend on the record date because the transaction will not settle until after the record date. If the seller had sold the stock one day earlier, she would not be entitled to the dividend -- the buyer would be entitled to the dividend and would probably receive a due bill.

Newly issued Treasury notes issued to meet a specific government funding requirement. A newly issued Treasury note would have a maturity of 2 to 10 years and would not be considered a money market instrument. A Federal Farm Credit Bank note maturing in one year or less is a money market instrument, as is commercial paper issued by the finance corporation of a major automobile manufacturer with a maturity of less than one year. Municipal construction loans issued to provide short-term financing for a construction project are money market securities.

Because money market instruments are designed to meet the short-term cash needs of issuing institutions, which of the following is NOT a money market instrument?

Municipal Bond Underwriting

Best Efforts and Firm commitments refer to what

A broker/dealer is involved in many underwritings. In one of the underwritings, the firm acts as an agent. In which of the following underwriting commitments does the firm serve as an agent? A. Firm underwriting B. Standby underwriting C. Best efforts underwriting D. Self underwriting

Best efforts underwriting. In both the standby and the firm offering, the lead underwriter (and thus the syndicate) promises to buy the shares from the issuer. Thus, the firm is acting as a dealer or principal because it owns the securities, whether it sells them or not. Self underwriting is a made-up answer; if you don't know what the answer means, don't pick that answer. In best effort underwriting, whether a mini-max or an all-or-none underwriting, the underwriter is only an agent, since it will never have to own the shares

Government of the country in which the investor pays taxes.

By which government is the income from eurodollar bonds always taxed?

the lower of the yield to maturity or yield to call

Bond confirmations must disclose what?

The U.S. dollar has gone through a deflationary period and is now down 10% versus other leading currencies. What effect does this have on bond prices?

Bond prices will decline. This is mainly due to the few buyers that will want U.S. bonds -- if the dollar is falling, fewer people will want bonds. In addition, to strengthen the dollar, the Fed will start to decrease interest rates, thus causing existing bonds to increase in value.

corporate debt securities

Bond trust indentures are required for:

corporate debt securities.

Bond trust indentures are required for:

income bonds

Bondholders may not take action against the corporation if it fails to make interest payments for:

income bonds.

Bondholders may not take action against the corporation if it fails to make interest payments for:

A customer of yours is in a high tax bracket. Under certain situations, he also is subject to the Alternative Minimum Tax. He invests in municipal bonds, but does not want any bonds whose interest is subject to the AMT. Which of the following municipal investments would most likely cause the investor to file the Alternative Minimum Tax?

Bonds backed by a stadium. Pollution control bonds are the only municipal bonds absolutely not subject to AMT. Bonds for a stadium are the most likely. Watch out and make sure you see whether they are asking for the most likely or the least likely. The other two bonds could be subject to AMT or not -- if the highway will be a private highway run by a corporation or other entity and have charges (tolls), then the interest is subject to AMT. However, if it is for the public good and paid with gasoline taxes, the interest is not subject to AMT. The water bond also depends on whether a private corporation or a municipal entity is issuing the bond, with the same results -- if issued by a corporation, it is subject to the AMT, and if by the municipality with municipality control, it is not subject to the AMT.

In which of the following ways does a PAC CMO compare with a Plain Vanilla CMO? A. Both the PAC CMO tranche and the Plain Vanilla CMO tranche have low prepayment risk. B. Both the PAC CMO tranche and the Plain Vanilla CMO tranche make prepayments in a sequential order. C. The PAC CMO tranche has both low prepayment risk and low extension risk, while a Plain Vanilla CMO tranche only has low prepayment risk. D. Both a PAC CMO tranche and the Plain Vanilla CMO tranche have the same extension risk.

Both the PAC CMO tranche and the Plain Vanilla CMO tranche make prepayments in a sequential order. All CMOs make all payments sequentially, meaning the earliest ones are paid first. This is the opposite of municipal serial bonds, which are called in inverse chronological order. The Plain Vanilla CMO tranche does not have low prepayment risk -- it has high prepayment risk since there is no companion tranche to take the prepayments. The Plain Vanilla CMO has high extension risk, but the PAC CMO tranche has very low prepayment risk.

In which of the following ways does a PAC CMO compare with a plain vanilla CMO? A. Both the PAC CMO tranche and the plain vanilla CMO tranche have low prepayment risk. B. Both the PAC CMO tranche and the plain vanilla CMO tranche make prepayments in a sequential order. C. The PAC CMO tranche has both low prepayment risk and low extension risk, while a plain vanilla CMO tranche only has low prepayment risk. D. Both a PAC CMO tranche and the plain vanilla CMO tranche have the same extension risk.

Both the PAC CMO tranche and the plain vanilla CMO tranche make prepayments in a sequential order All CMOs make any and all payments sequentially, meaning the earliest ones are paid first. This is the opposite of municipal serial bonds, which are called in inverse chronological order. The plain vanilla CMO tranche does not have low prepayment risk; it has high prepayment risk, since there is no companion tranche to take the prepayments. The plain vanilla CMO has high extension risk, but the PAC CMO tranche has very low prepayment risk

to customers and the IRS annually

Broker/dealers must report cost basis and sales proceeds

Which of the following issue Treasury STRIPS? A. U.S. government B. U.S. banks C. Broker/dealers D. Investment companies

Broker/dealers. Broker/dealers buy large amounts of U.S. government bonds and/or notes. The broker/dealer strips the interest off and sells it at a discounted amount. They also take the principal that is due and sell that as a zero-coupon STRIP. The interest payments, when paid by the government, pay off the discounted interest receipts, and the principal is paid at the maturity of the bond.

A German businessman has sent equipment to the United States and will be paid in U.S. dollars in 60 days. Because the investor is seeing interest rates rise, he is concerned for his payment. He calls and asks for your advice. What would you recommend as the best course of action to protect his payment?

Buy Euro calls. This question can be confusing because it is not talking about the Euro and how it is affected by interest rates. From your studies, you know that when you see the word "protect" and the answers are looking for an options position, you would select "buy puts" to protect a bull position, and "buy calls" for a bear position. This investor is concerned with a fall in the payment (bear) value in reaction to a rise in interest rates (bear = buy calls). Now, the answer choices are narrowed down to two options — buy the interest rate calls and buy the Euro calls. Buying calls on the interest rates would take advantage of the bull position, not protection. Since it is the payment the businessman is concerned about, not the rise in interest rates, Euro calls is the better answer. If the person was more concerned with the interest rates, then he would purchase calls on the interest rates. [Module 9, Option, Sections 16.1 & 17.4]

An investor sees a favorite stock rising and falling between 85 and 102. What type of order would be entered to buy the stock when it breaks through the resistance level? A. Sell limit B. Buy limit C. Buy stop D. Buy stop limit

Buy stop. A buy stop order is entered above the resistance level. It is always entered above the market. A buy limit is entered below the market to purchase at the lowest point. Buy stop limit orders take two prices, and only one price is given here. This is a buy order, so the sell limit is not even plausible. [Module 13, NYSE, Section 3.4]

Who pays the accrued interest on a corporate bond when it is purchased in the secondary market?

Buyer The buyer always pays the seller the amount of accrued interest owed on the bond for the time the seller has held the bond since the last interest period. The corporation pays the interest when due on the interest payment dates to the holder of the bond at that time. The buyer receives the total six months interest from the corporation, but must pay the seller interest for the she has held the bonds. Don't be fooled by the question thinking it is asking who pays the interest. This is who pays the accrued interest after a trade between the buyer and the seller

Inverse Floater CMO

CMO's with High Volatility and decreasing principal payments as interest rates rise

A preferred stock that can be called by the issuing corporation, thus forcing the redemption, is called a:

Callable preferred. Callable preferred is a type of stock that can be redeemed (called) by the issuer (the company). Callable preferred gives the issuing corporation the option to call the stock back from the shareholders. If the company exercises the call, it must pay a predetermined amount that is usually described in the prospectus at the time the stock is issued. Participating preferred stock participates in dividends given to common stock, convertible preferred stock can be converted to common stock, and cumulative preferred stock can receive unpaid dividends.

Decisions by the Department of Enforcement regarding complaints:

Can be appealed to the FINRA National Adjudicatory Council (NAC) Can be appealed to the FINRA National Adjudicatory Council (NAC). Decisions by the Department of Enforcement can be appealed to the NAC. Decisions by the National Adjudicatory Council can be appealed to the SEC.

Put-Call Ratio

Can be used to gauge investor sentiment as being bearish or bullish

A customer calls his registered representative to purchase 300 shares of DOW stock, but the registered representative enters the trade for 800 shares. The trade is executed. The registered rep in reviewing the order the next day notices the mistake. After telling his supervisor, what should the registered rep do with the trade?

Cancel the trade and re-enter the order to read the correct number of shares for the customer, purchase the remainder for the firm, and let the trade stand with the contra broker/dealer. The trade has to stand with the contra broker/dealer. Since more shares were purchased than the customer ordered, the customer's order needs to be completed and the firm must purchase the extra shares. Later, the firm can sell the extra shares. However, before taking any action, the first thing the registered rep must do is to notify the supervisor.

A customer calls and has her registered rep purchase 500 shares of XYZ stock. In entering the trade, the registered rep has put the wrong account number on the order. The trade is executed. What should be done with the trade? A. Change the account number and let the trade stand B. Cancel the trade and file a DK notice C. Cancel the trade and re-enter the trade with the correct account number D. Cancel the trade with the wrong account number

Cancel the trade and re-enter the trade with the correct account number. This is called a "cancel and rebill." It is an accounting adjustment -- the firm cancels the first trade, then repurchases the security and sells it to the correct account at no additional cost. [Module 14, FINRA, Section 2.18]

earned (accrued)

Capital gains distributed by a mutual fund to shareholders are reported and taxable for the year:

taxed as ordinary income.

Cash dividends from REITs are:

An investor is analyzing the money supply. When looking at M1, what is NOT included? A. Certificates of deposit B. Demand deposits C. Checking accounts D. Cash in circulation

Certificates of deposit. M1 includes all cash in circulation, checking accounts, and demand deposits. M2 adds the timed savings deposits, and M3 adds the jumbo certificate of deposits. [Module 4, Economics & the Markets, Section 2.0]

A customer has been investing in annuities for 15 years. He calls his annuity adviser and tells her that he wants to annuitize and start his retirement. What will the insurance company managing the annuity do with the separate account? A. Liquidate the account B. Change the account and annuitize the amount of annuity units times their net asset value C. Keep the account intact and start a systematic withdrawal of the accumulation units D. Make periodic withdrawals from the account as determined by the amount requested by the annuitant

Change the account and annuitize the amount of annuity units times their net asset value. The separate account is actually kept intact, but the accumulation units are changed to annuity units, based on the person's age, sex, and type of payout via an actuarial formula. Then the withdrawals are based on the amount of annuity units needed multiplied by their net asset value as determined each month. [Module 8, Retirement Plans, Section 4.2]

A customer has been investing in annuities for 15 years. He calls his annuity adviser and tells her that he wants to annuitize and start his retirement. What will the insurance company managing the annuity do with the separate account? A. Liquidate the account B. Change the account and annuitize the amount of annuity units times their net asset value C. Keep the account intact and start a systematic withdrawal of the accumulation units D. Make periodic withdrawals from the account as determined by the amount requested by the annuitant

Change the account and annuitize the amount of annuity units times their net asset value. The separate account is actually kept intact, but the accumulation units are changed to annuity units, based on the person's age, sex, and type of payout via an actuarial formula. Then the withdrawals are based on the amount of annuity units needed multiplied by their net asset value as determined each month. [Module 8, Retirement Plans, Section 4.2]

Which of the following has the biggest impact on the tranches of a CMO? A. Credit risk of the underlying securities B. Changes in interest rates C. The tax implications of holding real estate D. The sales of new homes

Changes in interest rates Changes in interest rates. CMOs are a debt security. All debt securities are affected by interest rates. You were trying to outthink this one if you picked anything else.

value of underlying securities held in the separate account.

Changes in payments on a variable annuity correspond most closely to fluctuations in the

What constitutes a violation of the FINRA Conduct Rules?

Charges to forward proxies. Charging the customer to forward proxies is a violation of the FINRA Conduct Rules. The issuer or the company that issues proxies pays this charge. A broker/dealer can charge transfer fees and can charge for holding stock, but the charges must be fair and reasonable. Firms can also charge for lending money to a customer, which is the case for a customer who uses margin to purchase securities

a concession.

Nonmembers of a syndicate who are assisting in its sale of bonds buy the bonds at a discount called:

A shares

Class of Shares that are front end load and can be reduced or eliminated by break points

C shares

Class of shares that are 12-1b fees charged quarterly with small back-end load in the first year

B shares

Class of shares that are back end load that declines over time combined with 12-b fees

D shares

Class of shares that is level load plus redemption fees

The prime rate is the rate of interest that:

Commercial banks base their interest charges for loans to businesses. The prime rate is the base rate that banks charge their business customers. Banks base their interest rate charges on the current interest rates, the credit standing or rating of the business, and the amount borrowed.

The use of federal funds would be most used by: A. Commercial banks B. Federal Reserve Bank C. Savings and loan banks D. Investment bankers

Commercial banks. "Federal funds" is money that is being kept at the Federal Reserve Bank. Most commercial banks, and all those belonging to the Federal Reserve System, have accounts at the Federal Reserve Bank. Each commercial bank must keep in a special reserve account a designated percentage of the money they are holding for their customers (generally 20%). This is also the account from which the commercial banks borrow and lend with each other to meet their reserve requirements. Often, banks have more than they need and other times have less than they need, so they borrow from each other. In addition, the banks may borrow from the Fed to meet the requirement which they do through their federal funds. Finally, all trading of Treasury securities settle the money owed through their federal funds.

Which of the following is exempt from registration with the SEC prior to being issued? A. Commercial paper B. Corporate bonds C. An ADR D. A variable annuity

Commercial paper. Commercial paper is issued for less than 270 days, and thus is exempt from registration with the SEC. Yes, ADRs have to be registered through the bank that brings them into the U.S. The separate account of the variable annuity is registered with the SEC as either a unit investment trust or as an open-end investment company. Corporate bonds are always registered

Which of the following is exempt from registration with the SEC prior to issuance? A. Commercial paper B. Corporate bonds C. ADRs D. A separate account for a variable annuity

Commercial paper. Commercial paper is issued for less than 270 days, and thus is exempt from registration with the SEC. Yes, ADRs must be registered through the bank that brings them into the U.S. The separate account of the variable annuity is registered with the SEC as either a unit investment trust or as an open-end investment company. Corporate bonds are always registered.

Which of the following causes the alpha of a stock to change? A. A higher short-interest ratio in the stock in the marketplace B. Company earnings have increased more than the anticipated earnings for the latest period reported C. Changes in world politics and the effect of those actions on the economy D. The declaration of a dividend by the company

Company earnings have increased more than the anticipated earnings for the latest period reported. Alpha is a volatility measurement that is based on aspects brought on by the company. Where beta is the measure of a stock against the market in general, alpha is the volatility of a stock due to some aspect of the company itself, such as change in management, greater or lesser earnings, and other factors brought on by the company. These cause the volatility in the stock greater or less than was expected. Declaring dividends has no effect, while changes in world politics is a factor in beta, not alpha. The short-interest ratio is just anticipation by market participants as to the direction of the stock, not an alpha or beta factor.

are highly compensated, have fewer years until retirement

Compared to defined contribution plans, defined benefit plans give the highest return to employees who

not be tied to specific investment banking transactions, can be tied to the firm's investment banking revenue

Compensation paid to research analysts may:

FINRA broker/dealers are permitted to: A. Tell investors to buy mutual funds shortly before a dividend or capital gain distribution is paid B. Arrange for a customer to obtain credit to buy open-end investment company (mutual fund) shares C. Provide a discount to nonmember broker/dealers when selling them investment company shares D. Continue to compensate retired registered representatives for contractual plan sales, if a contract is signed with the registered representative who has since retired

Continue to compensate retired registered representatives for contractual plan sales if a contract is signed with the registered representative who has since retired. Broker/dealers can compensate retired registered representatives only if a contract is signed between the broker/dealer and the registered representative prior to the registered representative's termination from the company. The practice of telling investors to buy mutual funds shortly before a dividend or capital gain payment is known as "selling dividends," and is a rule violation. Arranging for a customer to obtain credit to buy open-end investment company shares is not permitted since new issues cannot be purchased on margin. Open-end mutual fund companies only issue new shares. Providing a discount to nonmember broker/dealers when selling investment company shares is not permitted because discounts cannot be given to nonmembers.

A preferred stock that can be changed into shares of common stock of the same company any time the owner of the preferred stock desires is called:

Convertible preferred stock. Convertible preferred stock allows the owner to convert at their will.

guaranteed as to payment of principal and interest by another corporation.

Corporate bonds that are guaranteed are

Which of the following is not a leading indicator as defined by the Conference Board? A. Corporate profits B. CCI C. S&P Index D. New housing starts

Corporate profits. Corporate profits are a lagging indicator, not a leading indicator. All of the others are leading indicators, even machine tool orders. Machine tool orders are one of the oldest of the leading indicators, but remain a true indicator. [Module 4, Economics & the Markets, Sections 5.1 & 5.3]

any written or electronic communication that is distributed or made available to 25 or fewer retail investors within any 30 calendar-day period

Correspondence, one of the three categories of communication with the public, is defined as

a stable market

Covered call writing normally occurs in what kind of market

sells a put on a stock he has sold short.

Covered put writing is a strategy where an investor:

Income bonds.

If a mutual fund's objective is income, it would NOT hold which of the following securities in its portfolio?

A corporation has developed a special type of widget. To be able to make these widgets in large quantities, the corporation is issuing a bond to build a widget factory, even though the U.S. government has not approved the widget patent. What type of bond is this?

Debenture bond. The proceeds from the bond issue will be used to build a factory. Since it is not for the purchase of equipment (rolling stock), it can't be an equipment trust certificate. There are no bonds referred to as "defensive" or "special situation" bonds. Debenture, collateral, equipment trust certificate, subordinated debenture, and adjustment bonds are types of bonds, not defensive or special situation bonds

Treasury notes.

Debt instruments put up for auction by the U.S. treasury that offer intermediate maturities best describes:

commercial paper or a promissory note

Debt normally issued by big corporations with reliable credit ratings who seek to finance short-term needs

An investor would like to buy a municipal revenue bond. As her registered representative, you show her a particular bond that was issued by New York City. Which of the following is the most important factor to consider for your investor's purchase of the tax-free bonds? A. Good title to the property B. Debt service coverage C. Legislative changes D. Amount of outstanding debt issued by the city

Debt service coverage. You may be tempted to choose legislative changes because changes in the tax laws may affect the tax-free status of the bonds. However, this is a revenue bond, and debt service coverage and competitive projects are factors in revenue bonds. General obligation bonds tax the general population to recoup the amount borrowed and to pay for the interest paid to investors. Essentially, this question asks whether you understand the difference between a revenue bond and a GO bond. [Module 5, Municipal Securities, Sections 9.1 - 9.5]

total of interest and principal payable by the issuer plus any amount required to be deposited into a sinking fund

Debt service is best described as the:

A person employed by a corporation has a retirement plan that will pay her the equivalent of eight years salary at retirement. What type of retirement plan is this?

Defined benefit plan. Since the plan specifies the amount the employee will receive at retirement, it has to be a defined benefit plan. If this stated how much the employee could contribute, it would be any one of the other three, depending on if there were matching funds with the 401(k) plan; sharing in the profits with the profit-sharing plan; or just receiving contributions from the employer with the defined contribution plan.

An announcement in the financial section of the newspaper states that the "money supply" (M1) has dropped for the week. This means that:

Demand deposits (checking accounts) and currency in circulation in the banking system have declined. Demand deposits (checking accounts) and currency in circulation have declined. M1 is composed of demand deposits, currency in circulation, and all interest-bearing checking accounts. Savings are in M2. M1 is the most liquid of the components in the money supply. . [Module 4, Economics & the Markets, Section 2.0]

the discount rate is decreased by FRB.

Disintermediation is a movement of funds which could result from all of the following EXCEPT:

money is tight.

Disintermediation is most likely to occur when:

All of the following are on the balance sheet but not on the income statement, except: A. Net worth B. Current assets C. Inventory D. Dividends for the year

Dividends for the year The dividend is not on the balance sheet; rather it is found on the income statement right under earnings. Any dividends declared but not yet paid for the quarter are found in current liabilities, but they are only present dividends, not dividends for the year

American depositary receipts.

Dividends may be paid to holders of:

real estate taxes.

Each of the following are generally used to service state GO bond issues EXCEPT:

No.

Does a bond in the primary market have capital gain at maturity?

a funding technique that will cause the average cost per share to be less than the average price per share.

Dollar cost average

You have had a client for three years. He is interested in purchasing 1,000 shares of an IPO that is about to be brought to market. Your firm is one of the syndicate members of the offering. The lead underwriter is looking for indications of interest. What is the first thing you should do as the rep for the client?

Don't do anything because of your relationship with the client. This answer comes under the "know you customer" rule. You should already know where the person works and if they are a restricted person or not. You cannot confirm the transaction since this is prior to the release of the issue. Note: The lead underwriter is looking for indications of interest which only occurs during registration. Once released, orders are taken (not indications of interest), which also eliminates the answer "enter an order."

A customer has purchased an oil and gas developmental DPP. Which of the following statements best describes this program?

Drilling and building on proven land. This is the definition of a development program. Wildcatting or exploratory is drilling on unproven land, and the two dealing with and selling the oil or building and selling the oil incorporate an income program.

Which of the following is not subject to the depletion allowances in a direct participation program? A. Oil B. Timber C. Minerals D. Drilling equipment

Drilling equipment. Drilling equipment is not a wasting asset. It is a piece of equipment that is depreciated. [Module 16, DPP, Section 10.1]

be treated as a nonmember by other members.

During a period of suspension, a suspended member must:

cancel an order placed before the halt.

During a trading halt, an investor can:

the disclosure that it was prepared by the underwriters.

Each of the following would be disclosed to potential municipal bond buyers in the official statement of a new municipal bond issue EXCEPT:

joint accounts/tenants in common

Each party specifies a percentage of interest in the account. The interest of a deceased tenant passes to the estate of the decedent.

customer account processing.

Employees of a FINRA member firm must be fingerprinted if involved in any of the following EXCEPT:

reviewed by a principal of a finra member firm

Every transactions made by a registered representative for a customer's account must be:

You have a customer who holds a cash account and a margin account. He tells you that he has read that EGG is about to take off due to contracts received. He also thinks that another company, PPL, is heading down. He tells you to purchase 2,000 shares of EGG that is trading at 35 and sell short 1,000 shares of PPL that is trading at 61. What type of account should the customer use for the transactions? A. Purchase the EGG and short the PPL in the cash account, since the amounts will offset each other B. Execute both transactions in the margin account and meet the margin requirement on each transaction C. Short the PPL stock in the margin account and use the proceeds from the sale to purchase the EGG in the cash account D. Purchase the EGG in the margin account and use either the cash or margin account to short the PPL stock

Execute both transactions in the margin account and meet the margin requirement on each transaction . The long transaction can be done in either account, but that is not offered as a choice. However, the short account must be executed in the margin account, since the stock is being borrowed. The investor cannot use the proceeds from the short sale to purchase other stock. That money will be used to buy the stock when the customer wants to cover the short position. [Module 11, Customer Accounts, Sections 1.0, 2.1, & 2.2]

A customer of yours had previously purchased $50,000 of a 15-year municipal bond that has a put feature. Now, 10 years later, interest rates start rising and do so for six months. Which of the following actions would you suggest to your customer to take advantage of her situation?

Exercise the put option and purchase new, higher-interest bonds. Since interest rates have risen, the value of the bonds has declined; therefore, selling the bonds is not a great idea. Gifting the bonds would be good, but the customer may have a tax consequence. Holding the bonds does not allow the customer to take advantage of the higher-interest rate bonds that she can get by exercising the put on her current bonds and purchasing more bonds.

increasing consumer demand for goods and services, increasing industrial production, and rising stock markets and property values.

Expansions in the business cycle are characterized by:

number of registered reps employed, number of branch offices registered with finra

FINRA fees are based on what

the member firm must forfeit the concession earned.

FINRA rules require that, if a customer redeems mutual fund shares within 7 business days of purchase:

Trades in Nasdaq-listed securities and exchange listed securities when they occur OFF the trading floor

FINRA's Trade Reporting Facility (TRF) electronically facilitates the reporting of trade data such as price and volume for

backing away.

Failure to honor a firm quote is called:

Which of the following is not backed by the credit of the U.S. government? A. Treasury bills B. Public Housing Authority (PHA) bonds C. Government National Mortgage Association (GNMA) bonds D. Federal National Mortgage Association (FNMA) bonds

Federal National Mortgage Association (FNMA) bonds. The U.S. government does not back FNMA. Remember, "The government doesn't cover your fanny" because these securities are issued by an agency that is not backed by the U.S.

A registered representative should request an extension of time for an investor to pay for stock that he has purchased by the:

Fifth business day. The investor is supposed to pay by the third business day, but is really given until the fifth business day. If "the check is in the mail," an extension of time will be given, but not granted, until that fifth business day.

A customer calls and says he wants to buy 8,000 shares of EFG at 42.25 or better. He wants you to enter the order and have the whole order executed. He also tells you if you can't get all 8,000 shares purchased immediately, you are to cancel the order. What type of order is this?

Fill or kill order. Since all of the order must be executed immediately or it will be cancelled, it is a "fill-or-kill" order. Fill or kill is similar to an immediate or cancel order, but the latter does not have to be executed in its entirety. All or none orders do not have to be executed immediately; they can be put through in an hour, a day, a week, or a longer period of time. Market orders do not include prices

16 months.

Financial information CANNOT be used in a mutual fund prospectus if the information is older than:

An individual or an affiliated group of people can have the maximum exercise limits on outstanding options for how long?

Five consecutive business days. The question is asking for the minimum amount of time that an investor or an affiliated group can exercise the maximum amount of options. An investor or affiliated group can exercise 25,000 options in a five-day period

What is the maximum contribution that can be made by one parent in a year to an account in a 529 plan?

Five times the amount of the gift-tax allowance. The maximum under 2013 rules is $70,000, or $14,000 × 5, though there cannot be any more gifts for five years. The exam uses the phrase "five times the gift-free amount." This way, when the dollar amount increases in any given year, they do not have to change the exam answers

within 30 calendar days after the issuer delivers the securities to the syndicate.

For a new issue municipal syndicate account, settlement of the account must occur:

For which of the following reasons would a broker/dealer issue a DK notice to another broker/dealer? A. For payment of accrued interest B. For payment of a stock dividend that was not transferred in time for settlement C. For a trade in which the prices did not match D. For a trade of stock in which no ex-dividend date was given

For a trade in which the prices did not match. The DK ("don't know") notice is between broker/dealers confirming the information of the trade. When something does not match -- price, amount, securities, or even the contra B/D -- a DK notice is sent. It has nothing to do with payment of dividends (a due bill) or ex-dividend dates, nor is it about accrued interest.

25 round lots.

For reporting purposes, an order to sell 25 shares of an OTC equity security priced at $230 per share is:

payable date.

For tax purposes, cash dividends are taxable to stockholders as of the:

Share identification

For the purpose of reporting sales to the IRS, which method available to investors by the IRS offers the most flexibility in anticipation of the investor's year-end tax needs?

firm commitment.

For the underwriting of a municipal bond issue, competitive bids are submitted by underwriters as a:

From where may a stock not be tendered? A. From a custodial account B. From a long account C. From an UGMA account D. From a short account

From a short account. Since an investor who is short does not own the stock, he may not tender, or give up, what he does not own. To tender stock, the investor must own or be in a position to deliver the shares. [Module 15, Federal & State Regulations, Section 3.6]

Holders of secured debt, general creditors, holders of subordinated debentures, preferred stockholders

From first to last, in what order would claimants receive payment in the event of bankruptcy?

Which of the following is found on a balance sheet and not on an income statement? A. Retained earnings B. Funded debt C. Earnings for the year D. Interest cost

Funded debt. Funded debt is the long-term debt of a company -- the bonds. Retained earnings are found on the income statement and balance sheet, while the interest cost is only on the income statement, since it is paid before taxes. Earnings are also on the income statement, since dividends are declared from them

Function Allocation DPP

General partner takes all Tangible Drilling Costs and the limited partner takes intangible drilling costs

A registered representative (RR) has created an advertisement that he wants to send to tentative prospects regarding a particular investment strategy. What does he need to do?

Get approval from his branch manager. An RR must always receive the branch manager's approval for any advertisement that is sent. If the advertisement involves mutual funds, the firm is required to review and approve the ad and send a copy to FINRA for its review within 10 days of first use.

The purpose of cumulative voting is to:

Give small stockholders the chance to gain representation on the board of directors. These small stockholders may not have fair representation if they are not able to accumulate their votes in an attempt to elect a favored candidate to the board of directors. In this way, small shareholders may be able to sway management or the board of directors to accept some of their ideas about running the corporation. The object of cumulative voting is for small shareholders to gain seats, not to gain complete control, of a corporation's board of directors

After nine consecutive months of steady decline, the economy is said to be in a recession. Which of the following is measured to determine if the economy is in a recession? A. S&P 500 B. Gross Domestic Product C. Consumer Price Index D. Stock market

Gross Domestic Product. When the Gross Domestic Product (GDP) declines, the economy declines. If the GDP declines for two quarters (six months), it is considered a recession. The stock market and the S&P 500 (which is part of the stock market) may also be declining at the same time as the economy, but it is the GDP that defines a recession or depression (six consecutive quarters of decline). The Consumer Price Index is one component used to measure inflation.

One of your customers calls and tells you he wants to buy 300 shares of DOW stock on margin. He has a restricted CASH account. Which of the following is true of this customer's situation? A. He cannot execute the transaction. B. He can execute the transaction if he deposits 100% of the margin prior to the transaction. C. He must have listed securities in a different account to cover the purchase. D. He must have unlisted securities in a different account to cover the purchase.

He can execute the transaction if he deposits 100% of the margin prior to the transaction. This is a tricky question, yet it is pretty straightforward. This customer has a restricted CASH account. What does "restricted" mean? He failed to pay on time for some previous stock he purchased, or brought the stock in way too late. As a result, his account is "restricted," or what used to be called "frozen" -- he can only buy or sell as long as the cash or securities are in the account prior to executing the trade. Yes, his "cash" account is restricted, but so is his margin account. How do we know he has a margin account? Because he is going to buy the stock on margin, and you can't buy on margin in a cash account. Listed or unlisted stock in the account has no effect -- he has to have 100% of the required funds in the margin account prior to the trade being entered. This question attempts to confuse you with the cash account. [Module 10, Margin, Section 4.2]

Your customer has been investing for a number of years, but does not have an employer-sponsored retirement plan. He has decided to make a contribution to a traditional IRA. You review your customer's information — he is employed, married, and 47 years old with a 39-year-old wife who contributes to her employer's pension plan. Together, they make $140,000. Which of the following is true?

He can make a full tax-deductible contribution to his regular IRA, but his wife cannot. This couple's income is over the maximum allowed for a person (the wife) who is covered by a plan at work, and under the maximum for a person (the husband) who is not covered by a plan at work but whose spouse is. Therefore, his contribution is tax-deductible, while her contribution is not.

A married couple has a combined income of $120,000. They have contributed to traditional IRAs for the past nine years. The husband's employer starts a pension plan. What effect will his pension have on their future contributions?

He can make non-tax-deductible contributions to his IRA account, and she can continue to make tax-deductible contributions to her IRA account. They are making under the income limit of $178,000, but above the $95,000 ceiling for having a retirement account. Since he is part of a retirement plan, he is allowed to continue to contribute but the contributions must be non-tax-deductible. She can still make tax-deductible contributions because she does not have a retirement plan. [Module 8, Retirement Plans, Section 11.0]

An investor has been accepted into a real estate limited partnership. The general partner sends the new limited partner a copy of the partnership agreement, which the new limited partner signs and returns. What is the new limited partner signing when he signs the partnership agreement?

He is signing a power of attorney allowing the general partners to act in the limited partner's behalf. This is a power of attorney outlining the rights and duties of the limited partner and the general partner, and giving the general partner certain rights.

Which of the following is a suitable investment for a couple with moderate income and less than 15 years to retirement? A. Growth stocks B. 10-year government bonds C. Zero-coupon municipal bonds D. High-yield corporate bonds with less than 20 years to maturity

High-yield corporate bonds with less than 20 years to maturity. Since the couple only has moderate income, they cannot afford the risk of losing significant value in growth stocks. Government bonds may be safe, but their yield will not be significant. Municipal bonds are best for individuals who desire tax-free income, but a zero-coupon bond only pays interest upon maturity.

Which of the following would you consider a suitable investment for a couple with moderate income with less than 15 years to retirement? A. Growth stocks B. 10-year government bonds C. Zero-coupon municipal bonds D. High-yield corporate bonds with less than 20 years to maturity

High-yield corporate bonds with less than 20 years to maturity. Since there is only moderate income, the chance to recover an investment in stocks if the market is negative is not very appropriate for these people. The government bonds are a good safe investment, but corporate bonds will give a greater yield. The zero-coupon bond will take too long to mature and since it is in a municipal bond, it is only good for someone who needs tax-free income. [Module 18, Scenario-Based & Financial Statement Questions, Sections 3.0 - 3.3]

You have a 52-year-old conservative investor in a moderately high tax bracket. All of the following would be suitable investments for this person, except: A. U.S. government bonds B. Municipal bonds C. High-yield corporate bonds D. Money market investments

High-yield corporate bonds. The person is in a relatively high tax bracket, so the high yield of corporate bonds is not needed as much as preservation of capital. All of the others are safer; perhaps not as much income will be generated, but the principal will be safer, which is the main concern of a person over 50 years old. The individual person is limited to money market instruments in the form of T-bills and possibly negotiable CDs, but not commercial paper, BAs, or repurchase agreements. The other money market instrument would be money market mutual funds, but these are not for investment. The high-yield corporate bonds are a bit risky for the conservative investor. [Module 18, Scenario-Based & Financial Statement Questions, Sections 3.0 - 3.3]

An individual entering a noncompetitive bid for U.S. Treasury bills will receive which of the following yields for his purchase?

Highest yield. The yield is determined after the competitive bidding is over. The competitive bidding by larger purchasers of Treasuries determines the yields of the T-bill for the noncompetitive purchasers. The noncompetitive bidders always receive the highest yield as bid by the competitive bidders. Remember, if the question asks for the price, choose the lowest price because the lowest price will receive the highest yield.

stay the same.

Holders of long straddles would like the underlying stock to do all of the following EXCEPT:

from grants

How are GANS funded

on an annualized discount yield basis

How are T-bills quoted?

the discount is accredited and they must be taxed annually

How are zero coupon bonds taxed

Annual dividend/EPS

How do you find dividend payout ratio

30 days

How many days must you wait to by shares of a newly offered stock

3 dealers or more

How many dealers must be contacted to determine a prevailing market

2 or more

How many quarters of ecomonic decline is a recession

at least 6

How many quarters of economic decline is is a depression

Upon request to the broker/dealer; no restrictions exist.

How may a municipal firm retail customer gain access to an MSRB rule book?

0.8

How much interest expense may be deducted on a bond that is bank qualified

3 times profit or 5 million dollars, whichever is greater

How much is the penalty for insider trading

59 1/2

How old must you be to withdraw from a variable annuity with no tax penalty?

An investor is using Rule 144A to sell a large amount of her shares of a NYSE-listed corporation. Which of the following people might use Rule 144A for this sale? I. A corporate officer selling 100,000 shares of the company's stock II. A holder of 100,000 shares III. A 10% stockholder of the company selling a large quantity of the company stock IV. A control person selling 300 of his company shares

I. A corporate officer selling 100,000 shares of the company's stock III. A 10% stockholder of the company selling a large quantity of the company stock This question is really asking if you know who must use Rule 144 in the sale of their shares, and thus, who would consider using Rule 144A to avoid the registration. Of the choices given, the only two people who fit these criteria are the corporate officer selling 100,000 shares and the control person who owns 10% of the company's stock. Both are required to file Form144 under Rule 144; to avoid having to do that they would sell under Rule 144A. However, if the sale is less than 5,000 shares and $50,000, filing Form 144 is not a requirement; therefore, a control person selling 300 shares is not required to file Form 144. Since the question does not say if the holder of 100,000 shares is an officer or a control person, we assume they are not; therefore, the sale of shares to a QIB would not be under Rule 144A

A self-employed defined contribution plan is: I. A retirement plan for self-employed individuals II. A tax-deferred investment plan III. A tax-free trust IV. A retirement plan for employees of nonprofit organizations

I. A retirement plan for self-employed individuals II. A tax-deferred investment plan A self-employed defined contribution plan is a type of retirement plan for self-employed individuals and their employees. A self-employed defined contribution plan is also tax-deferred, not tax-free. Yes, the self-employed individuals earn the money this year "free" of taxes, but somewhere down the line they will have to pay the taxes on that money. This usually happens when they withdraw the money during retirement. 403(b) plans are for employees of nonprofit organizations, such as teachers and nurses

A self-employed defined contribution plan is: I. A retirement plan for self-employed individuals II. A tax-deferred investment plan III. A tax-free trust IV. A retirement plan for employees of nonprofit organizations

I. A retirement plan for self-employed individuals II. A tax-deferred investment plan A self-employed defined contribution plan is a type of retirement plan for self-employed individuals and their employees. A self-employed defined contribution plan is also tax-deferred, not tax-free. Yes, the self-employed individuals earn the money this year "free" of taxes, but somewhere down the line they will have to pay the taxes on that money. This usually happens when they withdraw the money during retirement. 403(b) plans are for employees of nonprofit organizations, such as teachers and nurses.

Which of the following transactions by a corporate insider would be exempt from filing under Rule 144? I. A sale of less than $50,000 II. A sale of less than 10,000 shares III. A sale of less than $100,000 IV. A sale of less than 5,000 shares

I. A sale of less than $50,000 IV. A sale of less than 5,000 shares A sale of less than $50,000 and less than 5,000 shares are exempt from filing under Rule 144. Under Rule 144, insiders selling less than 5,000 shares and a total dollar value of less than $50,000 are not required to file Form 144. If insiders sell more than 5,000 shares and/or $50,000, they must file Form 144 or sell through Rule 144A to a QUI (qualified institutional buyer). [Module 15, Federal & State Regulations, Section 2.5]

The Securities Investor Protection Act of 1970, which established the Securities Investor Protection Corporation (SIPC), covers: I. A single account II. A commodities account III. A joint account IV. A custodian account

I. A single account III. A joint account IV. A custodian account The Securities Investor Protection Act, which established the Securities Investor Protection Corporation (SIPC), covers a single account, a joint account, and a custodian account, among others. However, SIPC does not cover commodity accounts. The term "separate account" refers to each account that has a different owner or group of owners. If there is more than one account under the same name with a broker/dealer, all the accounts are grouped together. However, if the same investor has an account in his name, an account in both his and his spouse's names, and his name as custodian for a minor's account, each account is treated separately

An analysis of a general obligation bond includes the: I. Population growth II. Tax collection record III. Ad valorem taxes IV. Tax base

I. Population growth II. Tax collection record III. Ad valorem taxes IV. Tax base . All the items listed are included in the analysis of a general obligation bond. Know these. [Module 5, Municipal Securities, Sections 2.2 & 9.1]

Which two of the following can make a fully deductible IRA contribution? I. A single person who earns $50,000 and is part of her employer's pension plan II. A married person who is self-employed earning $85,000 and has a self-employed defined contribution plan III. A married person who earns $120,000 and is part of his company's pension plan IV. A single person who has $75,000 in income from dividends and interest on her investments

I. A single person who earns $50,000 and is part of her employer's pension plan II. A married person who is self-employed earning $85,000 and has a self-employed defined contribution plan A fully deductible contribution can be made when a person has a pension plan and makes under the lower threshold, which is $59,000 for a single person and $95,000 for a married person. If the person has a retirement plan with his employer and makes more than the higher threshold, a contribution can be made but it is not tax-deductible. In addition, the income must be earned income, not dividends and interest from investments.

The types of annuities that pay the most per month (with all else being equal) are: I. Annuities on men II. Annuities on women III. Annuities that offer 10-year certains IV. Annuities that offer no options

I. Annuities on men IV. Annuities that offer no options

A municipal bond could be purchased at a premium or at a discount. In which two ways would the premium or discount be treated when determining a gain and a loss? I. As straight-line accretion for bonds purchased at a discount II. As a loss that cannot be deducted when purchase at a premium III. Added as interest basis for bonds purchased at a discount IV. Subtracted as interest basis for bonds purchased at a premium

I. As straight-line accretion for bonds purchased at a discount II. As a loss that cannot be deducted when purchase at a premium Straight-line is nothing more than taking the amount of the discount and dividing that by the number of years to maturity. Think of the words "straight-line" -- it means the "same amount each year." If the bond is a 10-year bond that is purchased for $800, the discount of $200 is straight-lined (added to the cost) over the 10 years. In essence, this bond will increase in value $20 each year until maturity. If it is sold prior to maturity, the new cost basis will be the straight-line amount the bond has increased in value. Muni bonds purchased at a premium do not recoup the loss like corporate bonds. There is no such thing as added or subtracted interest basis. [Module 17, Taxation, Sections 5.1 & 5.2]

Your customer has just purchased $25m of EGG zero-coupon bonds that are quoted at 78.70. The bond will mature in 2027. Which two of the following statements are true regarding this bond? I. At maturity the investor will have no capital gain. II. At maturity the investor will have a capital gain of $213 per bond. III. The investor will not have to claim any income until maturity and will be taxed on it at that time. IV. Each year an accreted amount of the bond will be claimed as income and taxed each year.

I. At maturity the investor will have no capital gain. IV. Each year an accreted amount of the bond will be claimed as income and taxed each year. At maturity the investor will have no capital gain because each year an accreted amount of the bond will be claimed as income and taxed each year. This is a zero-coupon bond. Unless the bond is purchased in a tax-deferred account, an IRA, or Keogh, the accretion each year must be claimed. Therefore, there is no capital gain at maturity -- only one-year's interest accretion will be claimed. The person is taxed each year on this income as ordinary income, not capital gain, and it MUST be claimed. The bondholder may not elect to wait until maturity.

Which of the following are major dealers of repurchase agreements? I. Broker/dealers II. Banks III. Corporations IV. The Fed

I. Broker/dealers II. Banks III. Corporations IV. The Fed Yes, all of these entities deal in repos. As a matter of fact, even the Fed deals in repurchases mainly because it has the most money to play with. The broker/dealers put buyers and sellers together, while banks and corporations are the ones in most need. [Module 2, Corporate Debt Securities, Section 14.5]

Which of the following statements is true regarding the issuance of collateralized mortgage obligations (CMOs)? I. CMOs have investment-grade credit ratings. II. CMOs are issued by the government. III. CMOs are backed by agency pass-through securities held in trust. IV. CMOs give the holder a limited form of call protection that is not present in regular pass-through securities

I. CMOs have investment-grade credit ratings. III. CMOs are backed by agency pass-through securities held in trust. IV. CMOs give the holder a limited form of call protection that is not present in regular pass-through securities . Because CMOs are backed by pass-through securities issued by GNMA and FNMA, they have investment-grade ratings. They are not issued by the government, but by broker/dealers. Because CMOs split the whole mortgage security into tranches, the holder of a CMO only has to be concerned with early calls if the maturity is getting close. In regular mortgage securities, it is the luck of the draw as to which ones are called

Which two of the following items in an oil and gas program can be passed-through for taxes and affect ordinary income? I. Capital gains II. Depreciation III. Intangible drilling expenses IV. Capital losses

I. Capital gains IV. Capital losses All of these can be passed through to a limited partner, but only capital gains and capital losses affect ordinary income. Capital gains and capital losses only occur upon the sale of the partnership interest. Depreciation and intangible drilling expenses cannot be taken against ordinary income; however, they can be used to offset income from the program or another program. [Module 16, DPP, Section 10.1]

Which of the following statements regarding collateralized mortgage obligations are true? I. Collateralized mortgage obligations have high credit ratings. II. Collateralized mortgage obligations are backed by the federal government. III. Collateralized mortgage obligations have set maturity dates. IV. Collateralized mortgage obligations have set coupons.

I. Collateralized mortgage obligations have high credit ratings. IV. Collateralized mortgage obligations have set coupons. CMOs have high credit ratings in general, because most (not all) are issued and guaranteed by GNMA, FNMA, and Freddie Mac. CMOs involve minimal risk because they are also backed with real estate. Most CMOs are rated AA or AAA for risk. The coupons are set, because the mortgages that underlie CMOs pay back the interest rate, and the amortized principal amounts that are paid off as tranches mature. When mortgages are refinanced, the repayments or prepayments increase, and the companion tranches will be paid, which protects the main tranches. However, the interest will not be touched. The dates for the tranches are not maturity dates for the CMO; only the final date of the loan is the maturity date. That is why these are called tranches instead of maturity dates. The main tranches act like maturity dates, but tranches could be paid off at an earlier or later date than expected.

A new customer to the firm is assigned to you. In gathering the necessary information from the customer and determining his suitability for investments, he informs you that he wants a numbered cash account so it is not obvious to whom the account belongs. He explains that he has nosey in-laws who are always looking over his shoulder. To open the account, the customer has to do which two of the following? I. Come into the office to attest as to who he is II. Sign a discretionary authorization form III. Sign a margin agreement IV. Fill out a new account form

I. Come into the office to attest as to who he is IV. Fill out a new account form The customer must fill out a new account form and come into the office to attest as to who he is. Numbered accounts must be monitored similarly to regular accounts and suitability information must also be collected on the customer. There is no discretion being given, and since this is a cash account, no margin agreement is needed.

Which of the following are affected when a company that has declared a dividend pays a dividend? I. Current assets II. Current liabilities III. Net worth IV. Working capital

I. Current assets II. Current liabilities When the dividend is paid, the funds come out of current assets and the current liability is reduced, so just these two are affected. Net worth and working capital are affected when the dividend is declared, but not when it is paid.

An investor calls you to ask about hedge funds. She is interested in learning about the main investments transacted in hedge funds. Which three of the following are hedge fund investments? I. Derivatives & options II. Collateral mortgage obligations III. ETFs IV. Investment companies

I. Derivatives & options II. Collateral mortgage obligations III. ETFs Derivatives and options are the investments that are used the most for hedge funds. Derivatives (puts and calls) are used for protection and to take advantage of the market. Hedge funds also use both inverse and leveraged ETFs to hedge stock positions in the U.S. However, hedge funds do not invest in investment companies since the investors would have to pay the management fee twice.

A trust indenture points out which two of the following? I. Duties of the trustee II. Rights of the bondholder III. Obligations of the trust department IV. Regulations of FINRA

I. Duties of the trustee II. Rights of the bondholder Duties of the trustee and rights of the bondholder. The trust indenture is a contract between the issuer and the purchaser of the bond with the trustee being the enforcer for the investor. The trustee may be a bank or another entity, so the trust department of a bank may not always be the trustee. The indenture is an agreement on how the bond will be issued and the rights of the bondholders.

Representatives fills out a new account card, partnership agreement, principal gives written acceptance of account, first trade is executed

If a registered representative is opening a brokerage account for a partnership, in which order, from first to last, would the following events ordinarily take place?

Which of the following are true regarding a specialist on an exchange? The specialist: I. Executes limit and stop orders II. Is responsible for keeping a fair and orderly market III. Places limit and stop orders in a book to be executed when the price of the stock meets the order price IV. Makes a market in stocks listed on an exchange

I. Executes limit and stop orders II. Is responsible for keeping a fair and orderly market III. Places limit and stop orders in a book to be executed when the price of the stock meets the order price Specialists execute limit and stop orders and are responsible for maintaining a fair and orderly market for the stock to which they are assigned. Specialists do not make markets in listed stocks. Making a market occurs in OTC trading. Specialists buy and sell if need be, but the rest of the floor crowd makes the market. Specialists try to execute orders out of their book as the prices of their stock are reached.

Which two of the following statements are reasons that an investor goes into a direct participation program? I. Flow through of income II. Flow through of property III. Flow through of capital gains IV. Flow through of losses against ordinary income

I. Flow through of income III. Flow through of capital gains Flow through of income and flow through of capital gains. All investors want the income. In addition, if the DPP is making money, the investor will realize a flow through of capital gains as the DPP is wound up (closed down). Flow through of losses against ordinary income no longer exists. Since 1987, investors cannot take losses (these are not capital losses; rather they are depreciation, depletion, and expenses) against ordinary income. They can only be taken against income and capital gains on other DPPs. [Module 16, DPP, Section 9.1]

Which of the following actions can limited partners perform without losing their status as a limited partner and acquiring status as a general partner? I. Going to the site of the general partner and inspecting the books and records II. Voting to dissolve the limited partnership III. Voting to remove a general partner IV. Voting on the sale of some or all of the property held by the partnership

I. Going to the site of the general partner and inspecting the books and records IV. Voting on the sale of some or all of the property held by the partnership Limited partners can inspect the books and records and vote on the sale of property but cannot vote on dissolving the partnership or removing a general partner. Dissolving the partnership or removing a general partner places limited partners in the role of managing the partnership, which affects their limited liability status and make them a general partner.

The Quotron screen on a registered rep's desk is displaying Level 1 of Nasdaq Workstation II. Which two of the following will appear on the RR's screen? I. Highest bid II. Lowest bid III. Highest ask IV. Lowest ask

I. Highest bid IV. Lowest ask The highest bid and the lowest ask. These are the inside price. Remember that market makers sell at the ask price -- therefore, customers buy at the ask. Also, market makers buy at the bid, so customers sell at the bid. Since market makers buy low and sell high and customers do just the opposite, the best price to customers is the lowest ask (the price at which the customer buys) and the highest bid (the price at which the customers sells).

Which two of the following statements are true regarding the comparison of plain vanilla CMO tranches and planned amortization class (PAC) CMO tranches? I. Holders of plain vanilla CMO tranches have a higher risk of prepayment. II. Holders of PAC CMO tranches have a higher risk of prepayment. III. Holders of plain vanilla CMO tranches have a lower risk of prepayment. IV. Holders of PAC CMO tranches have a lower risk of prepayment. V. Holders of both plain vanilla CMO tranches and PAC CMO tranches have the same risk due to people refinancing and selling their property.

I. Holders of plain vanilla CMO tranches have a higher risk of prepayment. IV. Holders of PAC CMO tranches have a lower risk of prepayment. Since plain vanilla CMOs do not have companion securities, any prepayment goes directly to the next maturity year coming up. PAC and TAC CMOs have companion securities that take the prepayment risk, and PAC CMOs have second companion securities that take the overflow and the underpayment risks as well. [Module 3, Government Securities, Sections 9.1 & 9.2]

A company needs more cash to expand. The board of directors votes to issue more shares of common stock. How will the issuance of five million shares of stock affect the balance sheet? I. Increase cash II. Increase equity III. Increase total liabilities IV. Decrease working capital

I. Increase cash II. Increase equity The issuance of stock is an equity position; therefore, the liabilities will not be affected. The company will receive cash from the sale of the stock, and the stockholders are considered equity owners of the company. Since only the asset side of the balance sheet is affected, cash and working capital will increase, not decrease. Therefore, cash, total equity, and working capital increase, and the liabilities remain the same. [Module 18, Scenario-Based & Financial Statement Questions, Sections 1.0 & 1.1 ]

Which two of the following statements concerning convertible bonds are true? I. Interest rates are usually lower than on nonconvertible bonds. II. Convertible bondholders are owners of the corporation. III. If the underlying common stock should decline to the point where there is no advantage to convert the bonds into common stock, the bonds will sell at a price based on their inherent value without regard to the conversion. IV. Interest rates are usually higher than on nonconvertible bonds.

I. Interest rates are usually lower than on nonconvertible bonds. III. If the underlying common stock should decline to the point where there is no advantage to convert the bonds into common stock, the bonds will sell at a price based on their inherent value without regard to the conversion. Interest rates are usually lower than nonconvertible bonds and if the stock price declines, the value of the bond interest will be higher, discouraging investors to convert to stock. Convertible bondholders remain bondholders until they convert; therefore, they are creditors to, not owners of, the corporation.

When a registered representative opens a new account for an investor, what information is needed? I. Investment objectives II. Financial status III. Investment experience IV. Financial needs

I. Investment objectives II. Financial status III. Investment experience IV. Financial needs All the items listed are needed when a new account is opened. This helps determine the suitability of the customer for particular investments. Registered representatives should always have all this information to make sound, financial recommendations to clients

Which of the following are true of an overnight "repo"? I. It has no liquidity risk. II. It has no price risk. III. It has no interest-rate risk. IV. It is competitive with the federal funds rate.

I. It has no liquidity risk. II. It has no price risk. III. It has no interest-rate risk. IV. It is competitive with the federal funds rate. All of these are true of overnight repos. Since they are only overnight, there is no risk. They have relatively low interest, but the money never changes hands. It is more of a bookkeeping item to help banks meet their reserve requirement.

A client has heard from friends that the use of leveraged ETFs in conjunction with the rest of his portfolio can be an excellent enhancement. You as his registered rep have the obligation to explain leveraged ETFs to him. You tell your client which of the following? I. Leveraged ETFs use two times the amount of invested capital for the investments through borrowing. II. Leveraged ETFs use a large amount of cash in short-term securities and have a small portfolio of derivatives. III. Leveraged ETFs with short shares of the index funds protect in a down market. IV. Expense ratios are generally lower in leveraged ETFs.

I. Leveraged ETFs use two times the amount of invested capital for the investments through borrowing. II. Leveraged ETFs use a large amount of cash in short-term securities and have a small portfolio of derivatives. EFTs borrow two and up to three times the invested capital, which they use to buy short-term securities and then write derivatives to generate more capital. The drawback is that there are higher (not lower) expense ratios due to management fees that are higher than regular ETFs, interest cost on the borrowing, and all the transaction costs. Leveraged funds do not short shares of the index or any of the investments -- those are inverse ETFs, which are used for portfolio hedges. [Module 7, Investment Companies, Section 2.4]

Securities that trade in which of the following can be traded through an ECN? I. Listed stock II. Nasdaq stock III. Pink Sheet stock IV. OTCBB stock

I. Listed stock II. Nasdaq stock Only Nasdaq stocks and stocks trading on exchanges can be traded on an ECN. This is a means for individual investors to show their price for a stock. Investors showing the stock must be either a broker or have an account with a B/D that can enter the order. Pink Sheet and OTCBB traded stocks cannot be shown as individual trades on an ECN

If the United States devalues the dollar in comparison to other currencies, it would have the effect of: I. Making U.S. exports more competitive in foreign markets II. Making U.S. exports less competitive in foreign markets III. Making foreign exports less competitive in U.S. markets IV. Making foreign exports more competitive in U.S. markets

I. Making U.S. exports more competitive in foreign markets III. Making foreign exports less competitive in U.S. markets Devaluing the U.S. dollar would make U.S. exports more competitive in foreign markets. This is because the U.S. manufacturer can now charge less since they get more foreign currency for the dollar. At the same time, foreign imports become less competitive in U.S. markets. This is because it takes more U.S. dollars to purchase the foreign goods since the foreign manufacturer needs to get a certain price. Foreign goods would cost more U.S. dollars, and U.S. goods would cost fewer foreign dollars. [Module 4, Economics & the Markets, Section 3.1]

Blue-sky laws (state security laws) may vary, but all have provisions for: I. Misrepresentation or misleading statements II. Sale of securities by persons not registered in the state III. Sale of securities of non-U.S. (foreign) issuers

I. Misrepresentation or misleading statements II. Sale of securities by persons not registered in the state III. Sale of securities of non-U.S. (foreign) issuers Misrepresentation or misleading statements, the sale of securities by persons not registered in the state, and the sales of securities of non-U.S. (foreign) issuers are all covered under some aspect of each state's blue-sky laws.

You have a customer with a margin account who now wants to start trading options. He opens an options account. Which two of the following are required to open the options account? I. Obtain the signed options agreement II. Obtain the customer's signature on the new account form within 15 days III. Obtain a general securities principal's signature approving the account IV. Determine the type of options trading the customer wishes to do

I. Obtain the signed options agreement IV. Determine the type of options trading the customer wishes to do The customer must sign the options agreement, agreeing to all of the requirements that must be met, and return it in 15 days. In addition, the type of option trading must be determined at the time the account is opened. If he agrees with all of the information on the options account form, the customer does not need to sign the new options account form or return it. Also, the customer's signature is not necessary since the as already been obtained on the options agreement. An ROP, which could also be the BOM, must approve the account. A general securities principal can approve all accounts except options and municipal new accounts. A very important aspect on the new account form is determining which types of trades the customer can do. This is especially important if the customer wants to write uncovered options but has neither the suitability nor the required minimum income and deposit.

An investor has done a lot of trading in currency options. Which of the following concerns would the investor have regarding the investments? I. Pegging II. Revaluation III. Devaluation IV. There is no relationship of the dollar with the other currency

I. Pegging II. Revaluation III. Devaluation IV. There is no relationship of the dollar with the other currency These are the four main concerns of a person who invests in currency options. If a country pegs their currency to the American dollar, it is unlikely that the currency will move enough to make a profit for the investor. Devaluation and revaluation are great concerns for writers of options, since that is how the buyers make their money. When there is no relationship between the foreign currency and the American dollar, the greatest risk and reward can be achieved. [Module 9, Options, Sections 17.1 - 17.4]

A municipality has issued a special tax bond. This tax could be backed by which two of the following? I. Sales taxes II. Estate taxes III. Property taxes IV. Income taxes

I. Sales taxes III. Property taxes Sales and property taxes. Income taxes are applicable for all income earners, not just a special few. A special tax bond is used when taxes are increased just for a project. Special taxes could include sales tax (to repair a fallen bridge, as in San Francisco) or a property tax (such as when a new housing development is built and taxes are used for the development's infrastructure). Property taxes can be ad valorem taxes or a specific tax as a property tax on specific properties. A special tax bond will have a specific amount and an ad valorem tax is a property tax that can increase in amount over the years as the value of the property increases. Income taxes are not increased to pay for projects. Estate taxes go into the general fund of the state and the U.S. government. [

For a long period of time, an investor has been consistently investing in Treasury bills. The investor will be subject to: I. Stable principal II. Stable interest III. Fluctuating principal IV. Fluctuating interest

I. Stable principal IV. Fluctuating interest Investing in T-bills long term gives the investor stable principal and fluctuating interest. This is true because T-bills are very safe and only appreciate, so they have a stable principal. T-bills do fluctuate in their discounted price, so the investor will receive fluctuating interest rates.

In general, which of the following investments are used by hedge fund managers? I. Stock II. Corporate bonds III. Options IV. ETFs

I. Stock II. Corporate bonds III. Options IV. ETFs Hedge fund managers use leverage with stocks (margin) and also invest in bonds of distressed companies. They use options such as puts and calls and other interest rate derivatives for protection and to take advantage of the market. They also use currencies to hedge stock in foreign markets as well as ETFs to hedge against stock positions in the U.S. markets. [Module 7, Investment Companies, Section 2.6]

A registered rep opens a new account for a customer. Which of the following are required? I. Tax identification II. Names and ages of beneficiaries III. Passport number IV. Financial status

I. Tax identification IV. Financial status The customer's tax identification and financial status are among the details required to open a new account. Names and ages of beneficiaries and passport numbers are not required. However, the customer's Social Security number is required. The actual date of birth is not required, only the age. The customer's employer is also required, mainly to make sure he or she is not employed by another FINRA member firm. [

You have a client who has $20 million in securities with your firm. He wants to do some short-term investing and asks you for advice. You suggest Jumbo CDs. He asks you about them, and you tell him which two of the following? I. The CDs are negotiable and can be sold if the money is needed immediately. II. The CDs are not negotiable and must be held until maturity. III. The CDs are part of M2. IV. The CDs are part of M3.

I. The CDs are negotiable and can be sold if the money is needed immediately. IV. The CDs are part of M3. . The Jumbo CDs are part of M3. They make up the difference between M2 and M3. They are also negotiable, which means they can be sold in the secondary market. The difference between CDs and Jumbo CDs is that people purchase CDs as time deposits from banks in small amounts of $1,000 up to $100,000, while Jumbo CDs start at $100,000, can be bought and sold in the secondary market, and are excellent for large investors wanting to invest short-term.

Which of the following Bond Buyer Indices are A-rated or better? I. The general obligation 11-bond index II. The general obligation 20-bond index III. The general obligation 25-bond index IV. The general obligation 40-bond index

I. The general obligation 11-bond index II. The general obligation 20-bond index The 11-bond index is AA-rated and the 20-bond index is A+ rated. The 40-bond index is BBB-rated. The 25-bond index is a revenue index and there are no ratings on revenue bonds. [Module 6, Underwriting, Section 5.3]

the National Adjudicatory Council

If a registered representative is suspended by FINRA's Department of Enforcement, his first appeal would be to:

A registered representative who violates securities rules and regulations can be punished by: I. The SEC II. The NYSE III. FINRA IV. The state in which the violation occurred

I. The SEC III. FINRA IV. The state in which the violation occurred The SEC, FINRA, and/or the state in which the violation occurred can sanction a registered representative who violates securities rules and regulations. The registered representative can be disciplined by FINRA for any self-regulatory rules. In addition, the SEC and the state where the violation occurred can discipline a person as well as have legal charges brought against them. The punishment can result in the representative being fined, disbarred, censured, and -- in extreme cases -- sent to jail. The NYSE does not impose any disciplinary procedures on anyone except those specifically working on the floor of the exchange. Any reps or other people violating NYSE rules are handled by FINRA.

California has a state income tax. Which two statements are true for a buyer of a corporate bond? I. The bond is subject to state income tax. II. The bond is exempt from state income tax. III. The bond is subject to federal income tax. IV. The bond is exempt from federal income tax.

I. The bond is subject to state income tax. III. The bond is subject to federal income tax. Since the person is buying corporate bonds, they will pay both state and federal taxes on the interest earned on the bonds. However, municipal bond interest is exempt from federal taxes, and in most cases, from state taxes in the state they are issued. Government bond interest is exempt from state taxes only.

Which two of the following statements are true regarding collateralized mortgage obligations? I. The credit rating of CMOs is at least investment grade. II. Agency pass-through securities held in a trust are the backing for CMOs. III. Government agencies issue CMOs. IV. The government guarantees CMOs.

I. The credit rating of CMOs is at least investment grade. II. Agency pass-through securities held in a trust are the backing for CMOs. The government does not issue the CMOs -- broker/dealers do that -- nor does the government guarantee CMOs. The pass-throughs backing the CMOs are GNMA and FNMA, not the government. Limited and full-call protections are available on CMOs, especially with the PACs and the TACs. [Module 3, Government Securities, Sections 8.1, 8.2 & 9.1]

A new customer, looking to open an account, is assigned to you. She wants to set up a cash account that other family members cannot find. You tell her about a numbered account. Which two of the following are true of a numbered account? I. The customer's name must be on file and she must show a photo ID. II. She must sign a letter of authorization for trading in the account. III. She must sign the customer agreement. IV. The general securities principal must approve the account.

I. The customer's name must be on file and she must show a photo ID. IV. The general securities principal must approve the account. The customer's name must be on file and she must bring a photo ID into the office. The BOM, or a general securities principal, must approve the account. No customer agreement is required -- that is for a margin account. Since no one will have trading authority, a letter of authorization is not necessary. [Module 11, Customer Accounts, Section 5.1 & Tutorial Slide 5]

You have a customer who has a margin account. He calls and tells you he wants to purchase 3,000 shares of NANOSYS, a marginable Pink Sheet stock. You enter the order and the trade occurs at $17.50 per share. Which two of the following must be stated on the trade confirmation sent to the customer? I. The date and time of the transaction and the settlement date II. If the firm acted in a principal capacity, only III. Whether the securities are in registered or book-entry form IV. The type of transaction and the price at which the trade occurred

I. The date and time of the transaction and the settlement date IV. The type of transaction and the price at which the trade occurred The date and time of the transaction and the settlement date as well as the type of transaction and the price at which the trade occurred. These are important to the client and tell him when the trade occurred and the price. The confirm must state if the trade was done as either a broker or as a dealer, not just if the firm acted in a dealer (principal) capacity. All stock are in registered form, although many of the investors have the B/D firm hold the securities. [Module 11, Customer Accounts Tutorial Slide 11]

Which of the following statements regarding the euro currency are true? I. The euro is a currency of the consolidated countries of Europe. II. The euro can be converted into the currency a country used before it switched to the euro. III. The euro can change in value in relation to the U.S. dollar. IV. The euro is being used by all countries in Western Europe as their monetary means of exchange.

I. The euro is a currency of the consolidated countries of Europe. III. The euro can change in value in relation to the U.S. dollar. The euro is a currency of the consolidated countries of Europe, but not all the countries of Western Europe. For example, England is not part of the consolidated countries, and is still using the British pound. The euro can and does change in value to the U.S. dollar, almost on a daily basis. The euro cannot be converted into the currency a country used before switching to the euro.

If a FINRA member sends a DK form, which of the following statements are true? I. The form is sent to the contra broker/dealer. II. The form is sent to the customer. III. The form is sent to FINRA and/or the NYSE. IV. The form is sent for unmatched orders.

I. The form is sent to the contra broker/dealer. IV. The form is sent for unmatched orders. The form is sent to the contra broker/dealer for unmatched orders. A trade took place, but its confirmation was sent to the wrong broker/dealer. DK stands for "don't know." Therefore, the form is sent back to the broker/dealer who mistakenly sent a confirmation for a trade to the wrong broker/dealer. [Module 14, FINRA, Section 2.17]

a violation of FINRA rules.

If a registered representative marked a solicited order ticket as unsolicited, this act is:

A bond analyst uses which of the following to determine the strength of a general obligation municipal bond? I. The population growth of the area II. The per capita income of the citizens III. The industrial development of the area IV. The past performance of the payment of interest V. The income and revenues of the municipal projects in the area

I. The population growth of the area II. The per capita income of the citizens III. The industrial development of the area IV. The past performance of the payment of interest A municipal bond analyst looks at the community, the population growth, the per capita debt, and the payment history of the municipality when analyzing a general obligation bond. The analyst also reviews industrial growth in the area because of the taxes that will be generated. General obligation bonds are only paid by taxes imposed on the community and the businesses in a municipal area. If this had been a revenue bond, the analyst would only look at the facilities that would generate the income. The income and revenues of other municipal projects are not as important for a bond analyst to review for a general obligation bond; however, this information may be important in evaluating a revenue bond.

The trading desk of a B/D has received an order to purchase stock for a client. The order is entered and the trade is executed. The next day, the firm receives a DK notice. Which two of the following would cause a DK notice to be sent? I. The security could not be located. II. The contra broker/dealer does not know of the registered rep in the firm. III. The broker/dealer could not find the contra broker/dealer. IV. The number on the account is wrong.

I. The security could not be located. III. The broker/dealer could not find the contra broker/dealer. The security could not be located and/or the broker/dealer could not find the contra broker/dealer. A DK ("Don't Know") notice is between firms, thus any answer choices with "the rep" or "the account" are incorrect -- they would cause problems with confirmations within the firm, not the confirmation between the transacting firms. A DK notice is sent if one of the firms is not the contra broker/dealer, the securities are not available, the number of shares is incorrect, the price is incorrect, or any other problem with the trade between the firms. Errors on confirmations within the firm are sent to the registered rep for verification. [Module 11, Customer Accounts, Section 8.0; Module 14, FINRA, Section 2.17]

A new customer to the firm is opening an account in which he expects to buy and sell securities on margin. In opening the margin account, which two of the following are required on the new account form? I. The signature of a principal of the firm II. The customer's date of birth III. The customer's signature IV. The customer's Social Security number

I. The signature of a principal of the firm IV. The customer's Social Security number A principal of the firm must always approve and sign the new account form. In addition, the customer's Social Security number is required. The date of birth is not required. You must only determine that the customer is old enough, or their approximate age, to be able to enter into contracts. The customer's signature is not required for a margin account or any other account. The customer's Social Security number and employer information are required as well.

When interest rates are increasing, what happens to interest-rate options? I. The value of calls goes up. II. The value of calls goes down. III. The value of puts goes down. IV. The value of puts goes up.

I. The value of calls goes up. III. The value of puts goes down. Calls go the way of the market (in this case interest rates), and puts go the opposite direction

Which two of the following must be shown on the confirmation ticket of an 18-year municipal bond that is selling at a premium and is callable in 10 years? I. The yield to call on the bond II. Whether the bond is in registered or book-entry form III. The call date of the bond IV. The name of the trustee bank

I. The yield to call on the bond III. The call date of the bond Callable bonds require the date that the bonds can be called (the call date), and the lower yield between the yield to call and yield to maturity. Since this bond is trading at a premium, the yield to call generally will be lower, and thus must be shown. If the yield to call is given, the yield to maturity is not needed, although it will often be shown on the confirmation as well. The yield to call and call date must always be shown on premium bonds. Whether the bond is in registered form or book-entry does not need to be shown. What must be shown is if the bond is in book-entry form only -- if in registered form, nothing has to be stated. The trustee bank will be in the OS, but not on any confirmations.

It is halted.

If trading is halted in a listed stock, what happens to the trading in the stock's listed options?

the brother whose Social Security number is on the account

If two brothers open a joint account tenants in common, at year's end, the member firm carrying the account will send Form 1099 to:

A municipal bond registered rep may recommend investors to enter into municipal bond swaps for which two of the following reasons? I. To incur a capital loss for the investor for tax purposes II. To increase commissions III. To incur a capital gain for tax purposes IV. To improve the quality of their bonds

I. To incur a capital loss for the investor for tax purposes IV. To improve the quality of their bonds To incur a capital loss for tax purposes and to improve the quality of the bonds being held by the investor. A bond swap is when one or two bonds are sold and similar bonds are purchased in their place. A registered rep can recommend a bond swap to incur a capital loss, but recommending a bond swap to incur a capital gain is almost unethical. It is unethical to enter into a bond swap for the sole purpose of generating commissions from the investor. [Module 5, Municipal Securities, Section 10.1]

The issuance of preferred stock by a company would have an immediate effect on which of the following balance sheet items? I. Total assets II. Current liabilities III. Working capital IV. Shareholder's equity

I. Total assets III. Working capital Preferred stock is equity and the issuance of preferred affects shareholder's equity as well. You cannot pick three choices, so pick the only answer that has two of the three -- in this case, total assets and working capital. Equity could also be an answer, but it was not a choice. [Module 18, Scenario-Based & Financial Statement Questions, Sections 2.3, 2.4, 3.0, & 3.1]

Which of the following securities can be purchased on margin in a long account? I. Treasury bonds II. Extended Index LEAP options III. New issues IV. OTC Pink Sheet stock

I. Treasury bonds II. Extended Index LEAP options Extended Index LEAPS with more than a nine-month duration and Treasury bonds can be purchased on margin. However, LEAPS with less than nine months to expiration must be paid in full. New issues may never be purchased on margin. While OTC marginable stock is marginable, OTC Pink Sheet stock is not. OTC stock does not trade on Nasdaq, but is included in the Federal Reserve Board's list of stocks that are allowed to be traded on margin. [Module 9, Options, Section 10.0; Module10, Margin, Sections 1.0 & 8.0]

Which of the following can enter and execute trades through an Electronic Communication Network? I. Two individuals II. A market maker and an institution III. Two market traders IV. Two institutions doing business

I. Two individuals II. A market maker and an institution III. Two market traders IV. Two institutions doing business Yes, two institutions doing business can enter and execute trades through ECN. They were the first to trade in this manner. Today, market makers, specialists, individuals, and the traders on the trading desks of broker/dealers all use ECN platforms to execute trades

Which two of the following securities are exempt from state and local taxes? I. U.S. government bond II. Federal Home Loan Bank bonds III. GNMA bonds IV. State of New York bonds

I. U.S. government bond II. Federal Home Loan Bank bonds . U.S government bonds have always been exempt from state and local taxes. In 2009 Congress also made Federal Home Loan Bank bonds and Federal Farm Credit System bonds exempt from state taxes. State of New York bonds are not exempt from state and local taxes except for residents of that state. The states cannot tax issues from the federal government, its territories, or its agencies. However, states can tax other state securities. Government National Mortgage Association (GNMA) securities are guaranteed by the government but are subject to both state and federal taxes.

Which two of the following have a fixed rate of interest when issued? I. U.S. government bonds II. U.S. government notes III. U.S. government Treasury bills IV. U.S. government STRIPS

I. U.S. government bonds II. U.S. government notes T-bills are sold at a discount and have no interest. The other two bonds are notes and do have a stated interest rate. A STRIPS is a zero-coupon bond and is sold at a discount. The amount of the discount is the total interest to be received, and it is amortized over the life of the bond.

Securities can be sold under Rule 144 if: I. Unregistered securities are owned for at least six months prior to the sale. II. They are sold in a principal (dealer) transaction only. III. Buy orders are solicited from potential customers in anticipation of the sale provided it is done 60 days before the sale is made. IV. Current public information regarding the issuing company is made available.

I. Unregistered securities are owned for at least six months prior to the sale. IV. Current public information regarding the issuing company is made available. Unregistered securities are owned for at least six months prior to the sale and current public information regarding the issuing company is made available. Rule 144 securities cannot be sold in a dealer transaction because that would mean the dealer is buying the securities for its account. Rule 144 securities must be sold in an agency transaction -- to the public. Buy orders cannot be solicited prior to the sale.

Amber Corporation has issued 40 million shares of common stock and 2 million shares of preferred stock. Which two of the following are rights of the common stockholders? I. Vote for the board of directors II. Vote on the election of officers III. Vote on the decision to give cash dividends IV. Vote on the takeover of another company

I. Vote for the board of directors IV. Vote on the takeover of another company Common stockholders have the right to vote for the members of the board of directors and on any major issues, such as mergers, some major takeovers, and changing or continuing the corporate strategy. The board is responsible for the election of officers and the declaration of dividends.

Which two of the following are defined as money market instruments? I. Treasury notes II. Commercial paper III. Banker's acceptances IV. American depositary receipts

II. Commercial paper III. Banker's acceptances Money market instruments have maturities of less than one year. Of the possibilities, only banker's acceptances and commercial paper are defined as money market instruments. American depositary receipts facilitate trading securities, not short-term money. T-notes mature in one to 10 years.

income from convertible bonds.

Income from all of the following is partially exempt to a corporate investor EXCEPT:

A client of yours has a passive investment that pertains to oil and gas wells. Which of the following is true of this investment? I. Your client has limited liability in his investment. II. You client can help in the management of the property. III. Your client may have to make additional contributions. IV. Your client cannot vote on the sale of the property when the decision is to be made.

I. Your client has limited liability in his investment. III. Your client may have to make additional contributions. This is a passive investment, so it must be a limited partnership. In limited partnerships, investors do have limited liability -- the amount contributed. One of the points buried deep in the partnership agreement is the fact that GPs can call upon LPs to make additional contributions to the plan. This is one of the pitfalls of LPs. Unless the partnership agreement has taken the voting privileges away, limited partners can vote on the sale or refinancing of the property. Limited partners cannot manage the property or they lose their limited liability status. [Module 16, DPP, Sections 1.3 & 10.2]

A municipality is issuing a bond for a corporation. What kind of bond is this? A. General obligation bond B. Debenture bond C. IDR bond D. RAN

IDR bond. An IDR is a bond that is issued by a municipality and then sold to a corporation for $1. Its real name is Industrial Development Revenue bond, and it is used by corporations to better the community. If the community will not benefit, then the IDR does not get the tax benefit, and the municipality will not issue the bond. The general obligation bond is issued by a municipality for its own use and paid for by property taxes. A RAN is not a bond, but a short-term note, and is used while awaiting revenues. A debenture bond is issued by the corporation itself. [Module 5, Municipal Securities, Section 2.3]

You have a customer who has a long margin account with an $80,000 debit balance. Due to the decline in the stock price he is holding, your customer receives a maintenance margin call. Which two of the following will meet the margin call? I. More shares of the underlying stock II. A corporate bond convertible into marginable stock equal to twice the call III. Cash equal to the call IV. Cash equal to 50% of the call

II. A corporate bond convertible into marginable stock equal to twice the call III. Cash equal to the call Since you can only pick two answers, and the stock of the convertible bond is equal to twice the amount owed, the convertible bonds would be allowed. Since the deposit of stock is not stated as equal to twice the call, this answer is not fully correct. Only if the stock were worth twice the amount of the call would this be a correct answer. Cash equal to the call amount is always acceptable. Cash equal to 50% of the call is not allowed, since it is not sufficient. [Module 10, Margin, Section 4.5]

Which two of the following are subject to the FINRA 5% Policy on charges for commissions and markups? I. A mutual fund purchase from a broker/dealer II. A listed stock purchased on an exchange III. A Nasdaq stock transaction in the fourth market IV. A principal transaction of a Pink Sheet stock

II. A listed stock purchased on an exchange IV. A principal transaction of a Pink Sheet stock The FINRA 5% Policy only applies to secondary transactions, not to new issues or any transaction that has a prospectus, with the exception of ETFs and ETNs. ETFs and ETNs trade on exchanges and thus are subject to the 5% Policy even with new issues of those securities. The only two choices that involve secondary trading are the principal transaction of a Pink Sheet stock and listed stock. Mutual funds are sold in the primary market from the issuer to the investor. The fourth market involves institutions and ECNs, which are individuals trading among themselves, and typically do not have markups, markdowns, or commissions.

Which of the following married individuals with an adjusted gross income of $120,000 can open a full tax-deductible IRA? I. A person with a self-employed defined contribution plan II. A self-employed artist with no self-employed defined contribution plan III. A corporate employee with a corporate pension plan IV. A corporate employee with no pension plan

II. A self-employed artist with no self-employed defined contribution plan IV. A corporate employee with no pension plan A self-employed artist with no Keogh and a corporate employee with no pension plan. Married individuals with any amount of income may open a fully tax-deductible IRA if they have no pension plan with an employer or qualified retirement plan of their own, such as a Keogh. In addition, an individual with income below the lower threshold can have another retirement plan and still open a fully tax-deductible IRA. If the joint adjusted income is over the lower threshold, and there is a retirement plan, a tax-deductible contribution can be made (the amount is reduced for every $1,000 the income goes above the lower threshold). If the adjusted gross income (the test only says "gross income") is above the higher threshold, then no tax-deductible contribution can be made. Be careful when a married person has a spouse who is part of a retirement plan (this spouse is not.)

Which of the following married individuals with an adjusted gross income of $120,000 can open a full tax-deductible IRA? I. A person with a self-employed defined contribution plan II. A self-employed artist with no self-employed defined contribution plan III. A corporate employee with a corporate pension plan IV. A corporate employee with no pension plan

II. A self-employed artist with no self-employed defined contribution plan IV. A corporate employee with no pension plan A self-employed artist with no Keogh and a corporate employee with no pension plan. Married individuals with any amount of income may open a fully tax-deductible IRA if they have no pension plan with an employer or qualified retirement plan of their own, such as a Keogh. In addition, an individual with income below the lower threshold can have another retirement plan and still open a fully tax-deductible IRA. If the joint adjusted income is over the lower threshold, and there is a retirement plan, a tax-deductible contribution can be made (the amount is reduced for every $1,000 the income goes above the lower threshold). If the adjusted gross income (the test only says "gross income") is above the higher threshold, then no tax-deductible contribution can be made. Be careful when a married person has a spouse who is part of a retirement plan (this spouse is not.)

A registration statement has been filed with the SEC. A registered representative can: I. Send a research report to his clients II. Accept indications of interest from his clients III. Send a preliminary prospectus (red herring) to his clients IV. Accept orders and payments from his clients

II. Accept indications of interest from his clients III. Send a preliminary prospectus (red herring) to his clients A registered representative can only accept indications of interest and send red herrings when the registration is being reviewed, which is also called the cooling off period. Once the registration is effective, the issue can be sold, orders can be accepted, and research reports can be sent. Remember, the indication of interest is the customer saying he wants the issue, but it is not a binding order for either the customer or the broker/dealer. An order must be filled, and it is usually followed by payment. No payments can be accepted, even with an indication of interest, because then it technically becomes an order.

A customer would like to invest $100,000. He has heard of securities called ETFs and ETNs, and would like information about them. You explain that they are different products but have some similarities. Which two of the following statements are true of both ETFs and ETNs? I. Both ETFs and ETNs are for sophisticated investors only and require a minimum net worth. II. Both ETFs and ETNs trade on the floor of an exchange. III. Both ETFs and ETNs generate income in the form of dividends or interest. IV. Both ETFs and ETNs track indices.

II. Both ETFs and ETNs trade on the floor of an exchange. IV. Both ETFs and ETNs track indices. Both ETFs and ETNs trade on the floor of an exchange, and both track indices. In addition to tracking indices, an ETF may also track a managed pool of securities. Neither ETFs nor ETNs are for sophisticated investors only, nor does either require a minimum net worth. While ETFs may generate income in the form of dividends or interest, ETNs do not, and only offer the possibility of gains over the holding period

An investor is purchasing municipal debt securities when there is an inverted yield curve on new securities. How can the investor increase her income in her bond portfolio? I. Buy long-term bonds II. Buy short-term bonds III. Buy bonds selling at a discount IV. Buy poor quality bonds

II. Buy short-term bonds IV. Buy poor quality bonds Poor quality bonds will always get you higher yields than higher quality bonds, and thus more income. Discounted bonds are at a discount because their interest is so low relative to bonds that are currently being issued — not a good way to increase one's income. When there is an inverted yield curve, the short maturities have the higher yields.

You are told by the son of one your customers that his father died two days ago. Which of the following should you do after finding out that your client is deceased? I. Notify the customer's wife II. Close all open orders III. Find the name of the beneficiary of the account IV. Find the name of the appointed trustee for the customer

II. Close all open orders IV. Find the name of the appointed trustee for the customer The first thing you do after learning a client has died is to close all open orders. The wife should already know of his demise, so scratch that. However, it is important to find out who the trustee is for the deceased customer's account. The trustee will take care of the account and notify all interested parties. [Module 11, Customer Accounts, Section 5.5]

The Federal Reserve Board controls the money supply by setting and adjusting the different lending rates. Which of the following rates does the Fed directly influence? I. Prime rate II. Discount rate III. Federal funds rate IV. Margin requirements

II. Discount rate III. Federal funds rate IV. Margin requirements The prime rate is not determined directly by the Fed -- it is set by the banks that are lending. The Fed determines both the discount rate and the federal funds rate. Granted, the discount rate influences the prime rate, but the prime rate is the rate for those who borrow money from a bank. The Fed determines Reg T, which influences the rate at which investors borrow money for margin. However, adjusting margin requirements is the least used method for influencing the money supply.

A municipal securities broker's broker does which two of the following trading activities? I. Executes trades for retail customers II. Executes trades for broker/dealers III. Executes trades for institutions IV. Executes trades for the firm's own account to fill an order

II. Executes trades for broker/dealers III. Executes trades for institutions A broker's broker executes trades for other brokers and institutions, but not for its own account. A broker's broker acts as a broker and never buys for the firm's own account in a principal capacity. A broker's broker does not execute trades for retail customers

A person who sells variable annuities must be registered with: I. The SEC II. FINRA III. The NYSE IV. The state insurance commission

II. FINRA IV. The state insurance commission A person who sells variable annuities must be registered with FINRA and the state insurance commission. A person who sells variable annuities is a person who sells securities, and sellers must be registered with FINRA as well as with the state department of insurance. Variable annuities are not sold on any exchange, so the sellers do not need to be registered with the NYSE. Representatives and broker/dealers are not registered with the SEC, only the issuers. If this question had asked with whom the variable annuity itself had to be registered, then the SEC would have been included, not FINRA

A customer of yours is paying his yearly taxes. He has an income of over $300,000, which includes income from municipal bonds. He has capital gains and capital losses from stock that he owns. He may be subject to the alternative minimum tax. Which of the following could be subject to the alternative minimum tax? I. Interest on general obligation bonds II. Interest on an IDR bond III. Interest on revenue bonds IV. Capital losses on his stock

II. Interest on an IDR bond III. Interest on revenue bonds The AMT is a tax that is an "add-on tax" for those in a high tax bracket who are not paying taxes on certain items. These items could be subject to AMT if their income tax due is so low in comparison to their total income that the person is not paying their fair share. The low tax that is owed is due to not needing to pay taxes on these tax preference items. Interest on municipal bonds that are issued for private use is subject to the AMT. The municipal bonds subject to AMT are many of the revenue bonds that are for private use projects as well as IDR bonds that are issued for corporations (for private use, unless the private use is for pollution control). Therefore, IDR bonds could be subject to AMT, as could revenue bonds. However, capital losses are always a deduction -- only $3,000 can be deducted against the income, but any amount can be deducted against capital gains. Only revenue bonds are subject to AMT, not general obligation bonds.

Which of the following is an advantage for a customer to enter an order to sell stock through an ECN? I. ECN offers more liquidity. II. It is possible to enter the order during market hours or after market hours. III. The individual entering the order can remain anonymous. IV. Orders are matched with corresponding orders as soon as they entered.

II. It is possible to enter the order during market hours or after market hours. III. The individual entering the order can remain anonymous. IV. Orders are matched with corresponding orders as soon as they entered. Orders that are entered on an ECN are not as transparent -- not as many people see the orders -- and thus, they are not as liquid. Traders who might execute the trade will not see the order if they do not go to that particular ECN. There are many ECNs, and checking all of them would be a hardship for traders who are trying to keep up with orders. Also, often the orders are very large and individual investors do not purchase that many. All the other choices are advantages for a customer to sell stock through an ECN

The order book official on the CBOE is allowed to do which two of the following? I. Keep a fair and orderly market II. Keep a book of the orders for customers III. Act as a dealer IV. Execute orders after they have been accepted

II. Keep a book of the orders for customers IV. Execute orders after they have been accepted OBOs, or order book officials, are just like specialists on an exchange, except OBOs cannot execute orders for their own accounts. In essence, they cannot be a dealer. Since they cannot buy and sell for their own accounts, they cannot keep a fair and orderly market like the specialist can. [Module 12, OTC, Section 2.0]

Which two of the following are considered advantages of trading Nasdaq stock on an ECN? I. Liquidity and transparency II. Larger orders can be shown III. Market makers showing prices on the ECN can enter better bid and ask prices IV. Orders are only entered during market hours

II. Larger orders can be shown III. Market makers showing prices on the ECN can enter better bid and ask prices There are no restrictions on the size of the orders shown, and large orders can be entered. However, the orders can be entered during market hours or after market hours. The two biggest drawbacks are transparency, and thus, liquidity

A spread order.

If a customer wishes to buy 1 XYZ option and sell another XYZ option, but he is not willing to spend more than $300, which of the following orders should be entered?

Only the principal

If a customer wishes to open a cash account, who must sign the new account form?

within 5 business days.

If a customer wishes to purchase a nonexempt security in a cash account, Regulation T requires a broker/dealer to receive payment in full:

In which two of the following does Regulation T not apply to the settlement of trades? I. ADRs II. Municipal bonds III. Government bonds IV. Listed options

II. Municipal bonds III. Government bonds Regulation T does not cover government bonds and municipal bonds in any manner, although most broker/dealers apply the same principle of selling out if a customer does not pay with two business days after regular way settlement. The purchase of municipal and U.S. government bonds are not subject to the extra two-day extension. Reg T is involved in the purchase of listed options and ADRs, which are stock. [Module 10, Margin, Section1.0]

A corporate bond is selling at a discount. Rank the following from lowest to highest for this bond: I. Basis II. Nominal yield III. Current yield

II. Nominal yield III. Current yield I. Basis Bonds selling for a discount always have the nominal yield as the lowest. There is no way to determine the lower of basis (yield to maturity) or current yield unless the length of the bond is known. For this reason, there will only be one answer starting with nominal, so always start with the nominal, whether determining from highest to lowest for a premium bond, or from lowest to highest for a discount bond.

When bonds are selling at a premium, rank the following from highest to lowest: I. Basis II. Nominal yield III. Current yield

II. Nominal yield III. Current yield I. Basis Nominal yield, current yield, basis (yield to call basis) is the order from highest to lowest when bonds are selling at a premium. Just remember, when listing from highest to lowest on a premium bond, or when listing from lowest to highest on a discount bond, always start out with the nominal yield and the other yields follow inverse alphabetically: N, C, B for Nominal, Current, Basis. [Module 2, Corporate Debt Securities, Section 7.5]

What two positions constitute an opening sale? I. A long call II. Short against the box III. Short a put IV. Selling a long position

II. Short against the box III. Short a put . An opening order to sell, or better known as an opening sale, is always a short sale. Always pick the answer that contains the term "short" because short is an opening sale

A variable annuity separate account is: I. The guaranteed minimum cash value of the annuity II. Similar to a mutual fund III. A type of insurance policy IV. Covered under the Investment Company Act of 1940

II. Similar to a mutual fund IV. Covered under the Investment Company Act of 1940 Variable annuity separate accounts are similar to mutual funds in that they invest in many securities that appreciate; therefore, they are covered under the Investment Company Act of 1940. A variable annuity's separate account does not represent a guaranteed minimum cash value of the annuity. A variable annuity is not an insurance policy. There are no guarantees, so it cannot be a type of insurance.

With which of the following must a variable annuity contract be registered? I. FINRA II. State insurance department III. NYSE IV. SEC

II. State insurance department IV. SEC A variable annuity contract must be registered only with the SEC and the state insurance department. Those involved in selling the variable annuity contract -- registered representatives and broker/dealers -- must be registered with FINRA. Since insurance companies issue annuities, they must register with the department of insurance in each state where they do business. A variable annuity contract does not have to be registered with the NYSE.

A new customer has called your firm and you have responded. The customer wants to open a cash account to trade options. He wants to execute covered calls as well as purchase puts and calls. Which two of the following must sign the new account form? I. The client II. The BOM if registered as an ROP III. You as the registered rep IV. Any ROP of the firm

II. The BOM if registered as an ROP IV. Any ROP of the firm Either the BOM if registered as an ROP or any ROP of the office. The client does not have to sign the new option account form, nor does the RR. Only the BOM/ROP of the firm gives the approval and signs the new options account form. [Module 9, Options, Section 13.0; Module 11, Customer Accounts, Section 5.0 & Tutorial Slide 14]

A new customer has called your firm and you have responded. The customer wants to open a cash account to trade options. He wants to execute covered calls as well as purchase puts and calls. Which two of the following must sign the new account form? I. The client II. The BOM if registered as an ROP III. You as the registered rep IV. Any ROP of the firm

II. The BOM if registered as an ROP IV. Any ROP of the firm Either the BOM if registered as an ROP or any ROP of the office. The client does not have to sign the new option account form, nor does the RR. Only the BOM/ROP of the firm gives the approval and signs the new options account form. [Module 9, Options, Section 13.0; Module 11, Customer Accounts, Section 5.0 & Tutorial Slide 14]

73

If a customer writes 1 Jul 80 put at 7 and the put is exercised when the market price is at 70, for tax purposes, what is the effective cost basis of the stock put to the seller?

market value of the securities on the date of gift.

If a father makes a gift of securities to his 10-year-old daughter, gift taxes would be based on the:

Building permits.

Increases in which of the following indicators are regarded as predictors of the level of business activity?

One of your customers calls and tells you to purchase $50M 6% Atlanta, GA 2024 general obligation bonds. The bonds are callable in 2016 and are selling at a 3.25 yield. You execute the trade for the client. Which two of the following must be shown on the confirmation of the trade? I. The bonds are priced at a premium and are quoted at a 3.25 yield to maturity. II. The bonds are callable in 2016. III. The bonds are in registered form. IV. The total price including the accrued interest

II. The bonds are callable in 2016. IV. The total price including the accrued interest If the bonds are callable, this must be shown. They are callable in 2016; therefore, this must be stated on the confirmation. Only bonds in book entry form must be shown; bonds in registered form do not need to be on the confirmation. The yield that the bonds are selling at is the yield to call, not the yield to maturity, and the lower of the yield to maturity and the yield to call must be shown. Therefore, the fact that the bonds are selling at a 3.25 yield means they are priced at a high premium (they are over 10 years to maturity), so the yield shown must be a yield to call. The lower of the yield to call or yield to maturity must be quoted when showing the bond's pricing, and that lower yield must also be on the confirmation. The total price, including the cost of the bonds and the accrued interest, must be shown. [Module 11, Customer Accounts; Tutorial Slide 15]

One of your customers calls and tells you to purchase $50M 6% Atlanta, GA 2024 general obligation bonds. The bonds are callable in 2016 and are selling at a 3.25 yield. You execute the trade for the client. Which two of the following must be shown on the confirmation of the trade? I. The bonds are priced at a premium and are quoted at a 3.25 yield to maturity. II. The bonds are callable in 2016. III. The bonds are in registered form. IV. The total price including the accrued interest

II. The bonds are callable in 2016. IV. The total price including the accrued interest If the bonds are callable, this must be shown. They are callable in 2016; therefore, this must be stated on the confirmation. Only bonds in book entry form must be shown; bonds in registered form do not need to be on the confirmation. The yield that the bonds are selling at is the yield to call, not the yield to maturity, and the lower of the yield to maturity and the yield to call must be shown. Therefore, the fact that the bonds are selling at a 3.25 yield means they are priced at a high premium (they are over 10 years to maturity), so the yield shown must be a yield to call. The lower of the yield to call or yield to maturity must be quoted when showing the bond's pricing, and that lower yield must also be on the confirmation. The total price, including the cost of the bonds and the accrued interest, must be shown. [Module 11, Customer Accounts; Tutorial Slide 15]

An investor has $72,000 to invest in mutual funds. He decides to invest equal amounts in three mutual funds. The broker does not tell the customer that the three funds offer reduced sales charges on investments in excess of $25,000. Based upon the above information, which of the following is true? I. The investor has achieved diversification by buying three mutual funds and will probably make a profit. II. The broker has violated the "breakpoint sale" provision of FINRA as to a quantity discount on a large purchase. III. The broker has not violated FINRA Rules of Fair Practice because he followed the customer's instructions and purchased the three different mutual funds.

II. The broker has violated the "breakpoint sale" provision of FINRA as to a quantity discount on a large purchase. By not telling the customer of the reduced sales charges, the broker has violated the breakpoint sale provision of FINRA. If he had told the customer, or the customer had signed a letter of intent and then proceeded to buy the three mutual funds, no violation would have occurred. The broker didn't tell of the breakpoint, so he violated the rules, even though he did follow the customer's instructions. We cannot tell whether the customer has achieved diversification or not, and there is no sure profit.

One of your customers has just purchased a $25M 7% New York City 2025 general obligation book-entry bond, callable in 2020, at a premium of 110. Which of the following must be stated on the confirmation to the customer? I. The higher of the yield to maturity or yield to call II. The fact that the bond is in book-entry form III. The fact that the bond is callable IV. The name of the contra broker/dealer in the trade

II. The fact that the bond is in book-entry form III. The fact that the bond is callable The fact that the bond is in book-entry form and is callable. The confirmation must show the lower of the yield to maturity or yield to call, not the higher of the two. This confirm must have the yield to call since the bond is at a premium. As stated in the question, the bonds are in book-entry form; therefore, it must be stated on the confirm. The bond is callable, which must also be stated on the confirmation. The name of the contra B/D is never shown. [Module 11, Customer Accounts, Section 5.1 & Tutorial Slide 18]

At the end of 2011, an investor has 2,000 shares of UAL stock that he sells for a $10,000 loss. He also has received a K-1 report from his oil-drilling program that he has a net of $5,000 in losses after his depletion allowance. Which two of the following statements are true if the investor has an earned income of $120,000 from his job? I. The investor can deduct $3,000 of the passive losses from his earned income. II. The investor cannot deduct any of the $5,000 passive losses from his earned income. III. The investor can deduct $3,000 of the loss on his UAL stock from his earned income. IV. He can deduct all $10,000 of his loss on the UAL stock from his earned income.

II. The investor cannot deduct any of the $5,000 passive losses from his earned income. III. The investor can deduct $3,000 of the loss on his UAL stock from his earned income. The investor can only deduct $3,000 of the loss on his UAL stock from his earned income. The passive losses are not allowed against ordinary income, and the investor is only allowed $3,000 of the $5,000 capital loss. [Module 17, Taxation, Sections 2.2 & 2.3]

Which of the following should be taken into consideration when evaluating a customer's investment objectives? I. Whether or not the investor owns a home II. The investor's life and disability insurance III. The investor's tax bracket IV. The approximate age of the investor

II. The investor's life and disability insurance III. The investor's tax bracket IV. The approximate age of the investor Home ownership is not a factor since the net worth figure must exclude the value of the primary residence. The date of birth is not needed, but the approximate age is — this helps to know what advice to give and the customer's best interest. Life and disability insurance shows that the investor has complete coverage. The tax bracket is an important factor, particularly if the customer is in a high tax bracket.

Interest on a private purpose municipal bond. If more than 10% of a bond's proceeds go to private entities, the interest on the bond is a tax preference item for alternative minimum tax purposes.

If a high-income customer is subject to AMT, which of the following preference items must be added to adjusted gross income to calculate his tax liability?

An investor in municipal bonds is most concerned with which two of the following risks prior to purchasing a municipal bond? I. The investor's purchasing power risk II. The issuer's credit risk III. The bond's call date IV. The bond's legislative risk

II. The issuer's credit risk III. The bond's call date All of these risks should be considered before an investor purchases any municipal debt securities. However, the most important risks are credit risk and the call date of the bond. Credit risk is the ability of the issuer to pay interest and principal for the bond in a timely manner. The call date is always a concern for all bondholders, especially municipal bond buyers. They don't want their bonds to be called. Yes, if the bond appreciation fails to keep up with inflation, the investor will have less money in real dollars -- this is called purchasing power risk. However, this is only a small concern. Legislative risk is important for municipals, because the bond's tax-exempt status depends on state and federal laws. Again, this could be a concern, but generally is never considered.

Which of the following are considered market indicators for the issuance of new municipal revenue bonds? I. Treasury bond index II. The placement ratio III. 30-day visible supply IV. Bond Buyer 20 Bond Index

II. The placement ratio III. 30-day visible supply Of the choices given, only the placement ratio and the 30-day visible supply are considered market indicators for new municipal revenue bonds. The revenue index is also an indicator for muni revenue bonds, but it was not one of the choices. The Bond Buyer 20 Bond Index is for general obligation bonds, not revenue bonds. The yield for the 20 Bond Index is way too low for revenue bonds because the bonds in this index are rated bonds. Revenue bonds are not rated. The Treasury bond index is a basis for all interest rates, but it is not really a factor in determining the price of the municipal revenue bonds

If there is an appreciation in the exchange rate of the dollar, which of the following statements are true? I. U.S. exports will be more competitive in foreign markets. II. U.S. exports will be less competitive in foreign markets. III. Foreign imports will be more competitive in the United States. IV. Foreign imports will be less competitive in the United States.

II. U.S. exports will be less competitive in foreign markets. III. Foreign imports will be more competitive in the United States. When the dollar goes up, U.S. goods are less competitive and foreign goods are more competitive. It does not really matter which market, just remember, when the dollar is up, U.S. goods are less competitive and foreign goods are more competitive and vice versa.

An investor has purchased 1,000 ADRs on Nissan Motors. Each ADR has three shares. Nissan sends out a dividend of $.10 per share of stock. Which of the following will reduce the amount realized by the investor? I. A Japanese tax on the dividend II. U.S. federal taxes III. A state income tax IV. An excise tax

II. U.S. federal taxes III. A state income tax The federal tax is always taken on a foreign security. In addition, state income tax (if applicable) affects the amount received. The Japanese tax becomes a tax credit to the investor, and therefore, has no effect on the amount realized by the investor. Currently, the excise tax does not include dividends and interest on securities. [Module 17, Taxation, Section 6.10]

Which two of the following statements are true regarding collateralized mortgages and prepayments? I. When interest rates fall, prepayments fall. II. When interest rates fall, prepayments rise. III. When interest rates rise, prepayments rise. IV. When interest rates rise, prepayments fall.

II. When interest rates fall, prepayments rise. IV. When interest rates rise, prepayments fall. As interest rates fall, people will be able to refinance their mortgages, so prepayments to CMOs will increase. The opposite is true when interest rates increase -- very few people refinance and there are few, if any, prepayments. The only prepayments are those cases in which people sell their homes.

The Trust Indenture Act of 1939 covers: I. A purchase of $7 million worth of Treasury bonds II. A private placement of $8 million of corporate notes III. A $20 million issuance of corporate bonds sold interstate IV. A sale by a brokerage firm in five states of $25 million worth of corporate debentures

III. A $20 million issuance of corporate bonds sold interstate IV. A sale by a brokerage firm in five states of $25 million worth of corporate debentures Only corporate bonds are issued under a trust indenture; therefore, they are subject to the requirements of the Trust Indenture Act of 1939. Remember, corporate debentures are bonds. Treasury bonds are not covered under the Trust Indenture Act of 1939 because they are exempt. Private placements (Regulation D offerings) are considered exempt securities under the Securities Act of 1933.

He pays no estate tax.

If a married couple establishes a JTWROS account with a balance of $1 million and the wife dies, what is the husband's estate tax liability

25.

If a member firm is underwriting an initial public offering of common stock for ABCD Corp., a new issue that qualifies for Nasdaq listing, a prospectus must be provided to all purchasers for how many days following the effective date?

the marketability of the issue will decrease

If a municipal bond rated BBB is pre-refunded, all of the following statements are true EXCEPT

position trading.

If a municipal firm purchases a block of municipal bonds in anticipation of a price increase, the firm is engaged in:

Which of the following transactions are subject to the reporting rules of Rule 144? I. An employee purchases shares of his company's common stock. II. An employee sells shares of his company's common stock. III. An officer of a company sells shares of her company's common stock. IV. An officer of a company purchases restricted shares of her company's common stock from another employee.

III. An officer of a company sells shares of her company's common stock. Rule 144 is only concerned with the sale of restricted stock. Remember, restricted stock is stock that has not been registered and has been sold in a private placement; it is also stock that is being held by an insider. Choice IV also addresses restricted stock, but it is OK for an employee to sell restricted stock to an officer of the firm as long as it is done as a private placement. Regular employees are not subject to Rule 144 unless they have a large amount of shares (usually causing them to be considered an insider), or if they are in possession of material, nonpublic information about the company. Otherwise, an employee selling shares is allowed with no restrictions under Rule 144

Which two of the following statements about investment companies are true? I. All dividend distributions are exempt from taxes under current federal income tax laws. II. An investment company qualifies as a "regulated" investment company if it distributes 90% of its capital gains to its shareholders. III. Dividend and capital gains distribution can never be added together when computing the yield of a mutual fund. IV. A "regulated" investment company will be taxed at corporate income tax rates on all income and dividend distributions that are not distributed to its shareholders.

III. Dividend and capital gains distribution can never be added together when computing the yield of a mutual fund. IV. A "regulated" investment company will be taxed at corporate income tax rates on all income and dividend distributions that are not distributed to its shareholders. Regulated investment companies have special tax treatment -- they pay no tax on dividends and interest passed on to shareholders. They pay tax only on money they keep. Also, dividends and capital gains distribution can never be added together to compute yield. No dividends are exempt from federal taxes, and it is not true that regulated investment companies must pass 90% of their capital gains to the shareholders, but only income from dividends and interest.

A client comes to you wanting to make a $50,000 purchase. He has heard of a new type of securities called ETNs, and asks you about them. Which two of the following do you tell him are characteristics of ETNs? I. ETNs are registered under the 1940 Investment Company Act. II. Interest will be paid on a yearly basis. III. ETNs may trade in the marketplace at a price that is lower than the purchase price if the credit ratings of the issuer have been downgraded. IV. An ETN does not provide principal protection.

III. ETNs may trade in the marketplace at a price that is lower than the purchase price if the credit ratings of the issuer have been downgraded. IV. An ETN does not provide principal protection. ETNs trade in the market based on the ratings of the issuing bank or B/D. If the ratings go down, so do the value of ETNs since they are guaranteed by the bank or B/D. The only protection that ETNs give is the promise of the bank or B/D. ETNs are registered under the Securities Act of 1933, not the Investment Company Act of 1940. They are not an investment company, but are securities issued by the bank or B/D. Even though they are notes -- exchange-traded notes -- they do not pay interest, just the principal value, which the investor hopes is at a higher price than when purchased. The appreciation is the gain. [Module 7, Investment Companies, Section 2.5]

A corporate bond is selling at a premium. Rank the following from highest to lowest for this bond: I. Current yield II. Basis III. Nominal yield

III. Nominal yield I. Current yield II. Basis Bonds selling for a premium will always have the nominal yield the highest. There is no way to determine the higher of basis (yield to maturity) or current yield unless the length of the bond is known. For this reason, there will only be one answer starting with nominal, so always start with the nominal, whether determining from highest to lowest for a premium bond, or from lowest to highest for a discount bond

After opening a margin account with your firm, a customer tells you to purchase 500 shares of UAL at the market on the NYSE. You enter the order, it is executed, and the stock is purchased at $46.50 per share. Your firm then sends a confirmation to the customer. Which two of the following are required to be on the confirmation? I. That the transaction was closing purchase II. The date and time of the transaction III. That the firm acted as a principal in the transaction and the amount of commission IV. The symbol of the stock, the amount of shares transacted, and the price per share

III. That the firm acted as a principal in the transaction and the amount of commission IV. The symbol of the stock, the amount of shares transacted, and the price per share The date and time of the transaction and the stock symbol, amount of shares transacted, and the price per share. These are required to be on the confirmation. Since the stock was not purchased out of the inventory of the firm, but on an exchange, it has to be a broker transaction (not a principal transaction). A quick refresher: Dealer transactions have a markup, not a commission, and it does not have to be shown on the confirmation. This was an opening purchase, not a closing purchase

Which of the following statements are true regarding the issuance of an ADR? I. The receipts are issued by any bank in the U.S. II. The receipts are issued as one receipt per share of stock. III. The holders of the receipts are entitled to all declared dividends. IV. The holders of the receipts can sell their receipts anytime they wish.

III. The holders of the receipts are entitled to all declared dividends. IV. The holders of the receipts can sell their receipts anytime they wish. ADRs are receipts for shares of stock in a foreign company. The amount of shares is determined by the value of the stock and the monetary exchange rate in the country where it is issued. They are not issued by any bank in the U.S., but by a foreign bank with a branch in the U.S. or by a U.S. bank with a branch in a foreign country. The holders of the ADRs receive dividends, if declared, and the dividends are converted into American dollars. The receipts represent shares of stock, and like a stockholder, the holder of the receipts can sell at anytime. [Module 1, Equities, Sections 11.0 - 11.2]

A broker/dealer is the lead firm in a municipal joint account for a new municipal revenue bond. In addition, the broker/dealer is in a control relationship with the municipality. In what manner must the firm disclose its control relationship to the clients who are purchasing the issue? I. The relationship must be orally disclosed prior to the sale of the securities. II. The relationship must be orally disclosed prior to or at the completion of the transaction. III. The relationship must be disclosed in writing prior to the sale of the securities. IV. The relationship must be disclosed in writing prior to or at the completion of the transaction.

III. The relationship must be disclosed in writing prior to the sale of the securities. IV. The relationship must be disclosed in writing prior to or at the completion of the transaction. The disclosure must be in writing — it cannot be oral. The disclosure is supposed to take place prior to entering into the contract (when showing the bonds); if it is not, then it must be sent in written form at or before the completion of the transaction (the settlement date). In no way does an oral disclosure satisfy the requirement

A customer opens an options account with your firm. Under FINRA rules, which two of the following are required on the new account form? I. The signature of the registered rep II. The signature of the customer III. The signature of the BOM or other registered options principal IV. The date the options disclosure document was sent

III. The signature of the BOM or other registered options principal IV. The date the options disclosure document was sent The signature of the BOM or other registered options principal and the date that the options disclosure document was sent. Neither the customer nor the registered rep has to sign the new account form for an options account. The name of the registered rep managing the account must be given, but not the person's signature. Because options involve more risk than general securities, the customer must sign the options agreement (not an answer choice) and verify that the information on the new account form is accurate. The customer has 15 days to sign and return the options agreement

adjustment bonds.

If ABC Corporation reports a loss for the year, it is obligated to pay interest on all of the following EXCEPT:

municipal bond fund.

If ABC Fund pays regular dividends, offers a high degree of safety of principal, and appeals especially to investors seeking tax advantages, ABC is a(n):

writer short term gains buyers long term gains

If LEAPS options positions are maintained for more than 12 months

Its paid-in surplus will increase.

If XYZ Corporation sells an additional 1 million common stock with a par value of $1 for $10 per share, which of the following is TRUE?

out-of-the-money.

If XYZ closed at 41.10 and the XYZ Feb 50 calls closed at .35, the calls are:

Best offering price quoted in the interdealer market.

If a Nasdaq market maker is selling stock to a customer from inventory and the firm has held the shares to be sold for several months, what price should the dealer use as a basis for a markup?

not approve the order. Because this is a debit spread, the maximum gain occurs if both sides are exercised. If this occurs, the investor earns $5 (buy stock at 55 when the short put is exercised and sell stock at 60 by exercising the long put). Because the net premium paid for the spread is $5, there can never be any gain. This spread is not economical.

If a ROP is asked to approve a discretionary order to buy 1 XYZ Oct 60 put and sell 1 XYZ Oct 55 put for a net debit of $5, he should:

yield based on price

If a bond is sold to a customer at par, under MSRB rules, all of the following must be disclosed to the customer on his confirmation EXCEPT

I.MSRB rules prohibit the broker/dealer from acting as an underwriter for the issuer unless they meet the criteria of specific allowable exceptions., IV.There are some underwriting functions that the broker/dealer in their advisory capacity may be allowed to participate in such as assisting with the preparation of the official statement.

If a broker/dealer is acting as a financial advisor to a municipality, which of the following statements is TRUE?

net worth. Net worth is not affected by the issuance of long-term debt because it does not represent ownership. Assets will be affected (increased) by the issuance of long-term bonds. Liabilities will be affected (increased) by the amount of the issuance. Working capital will also increase.

If a company issues $10 million in par value convertible debentures, all of the following balance sheet items will be affected EXCEPT:

noncallable bonds.

If a customer believes that interest rates have peaked and wants to buy long-term, fixed-income securities providing semiannual interest payments, you would recommend:

cost of stock purchased less premium.

If a customer buys 100 shares of stock and writes one out-of-the-money call against his long position, the breakeven point is the:

straddle.

If a customer buys 5 ABC Sep 50 calls at 5 and 5 ABC Sep 50 puts at 3, this position is called a:

write a straddle

If a customer does not anticipate that a stock's price will change and he wants to take an option position, he would most likely:

its designated examining authority

If a customer does not pay for securities purchased within 2 business days of regular way settlement date, the broker/dealer may request a time extension from

its designated examining authority.

If a customer does not pay for securities purchased within 2 business days of regular way settlement date, the broker/dealer may request a time extension from:

Investment-grade corporate bond.

If a customer is in a low federal income tax bracket and his main investment objective is current income, which of the following securities should the agent recommend?

Ratio Write

If a customer is long 300 shares of ABC, and writes 5 calls against the position, this is an example of a:

issue exercise instructions.

If a customer is long ABC Sep 30 calls and the stock becomes subject to a trading halt on the floor of the NYSE, the customer is permitted to:

$10,000 no later than Monday, May 22.

If a customer purchases $10,000 worth of stock in a cash account on Monday, May 15, under Regulation T how much must the customer deposit, and when must the deposit be made?

accreted value.

If a customer sells a zero-coupon bond before maturity, gain or loss will be the difference between sales proceeds and:

the opening of accounts for third parties is prohibited

If a customer wants to open an account in the name of her adult son and wants the account to be approved for uncovered option writing, her request should be refused because:

The member firm opening the account must send duplicate confirmations to the employing member firm if the employing member firm has requested them to do so and The employing broker/dealer must receive prior written notice in order for the account to be opened but need not grant prior approval.

If a registered representative of a FINRA member firm wants to open an account with another member firm, which of the following statements are TRUE?

The underlying XYZ security

If a writer of an XYZ equity call option is assigned, which of the following should be delivered to the OCC?

6 months

If an individual fails a FINRA qualification exam three consecutive times, a fourth attempt may NOT be made for:

A unit investment trust

If an investment company invests in a fixed portfolio of municipal or corporate bonds it is classified as:

Long-term capital loss

If an investor buys a LEAPS contract on issuance and allows it to expire unexercised, what is the investor's tax consequence at expiration?

Purchasing power risk.

If an investor has a fixed-annuity contract with an insurance company, which of the following risks is assumed by the investor

capital gains rates on capital gains distributions and ordinary income rates on dividends.

If an investor has received dividends and capital gains distributions on mutual fund shares she has held for 4 months, the investor will pay:

write an uncovered straddle

If an investor interested primarily in speculation does not expect the price of DWQ stock to change, he or she will:

General obligation bond.

If an investor is in the highest federal income tax bracket and is subject to the alternative minimum tax, which of the following securities should an agent recommend?

He must accept the exercise notice.

If an investor maintaining a short equity option is assigned an exercise notice, which of the following statements is TRUE?

investors who purchased bills at this auction paid more for them than purchasers last week.

If an investor watches the latest T-bill auction fall to 4.71% from 4.82%, the best interpretation is that:

the difficulty in finding another investment with a like yield.

If interest rates are dropping, an investor with a maturing bond will be most concerned with:

remain unchanged.

If interest rates increase, the interest payable on outstanding corporate bonds will:

A writer of an uncovered put has been exercised. The company of the underlying stock declares a dividend. When would the writer of the put be entitled to the dividend?

If notice is sent to the OCC before the ex-dividend date. To get the dividend, you must buy before the ex-dividend date. Being exercised before the ex-dividend date counts as buying before the ex-dividend date. [Module 9, Options, Section 3.2; Module 14, FINRA, Section 5.1]

A good-till-cancelled order is handled on the book of the specialist in which of the following ways? A. The order expires automatically after six months. B. If properly renewed, the order will not lose its place on the book. C. The order never has to be reviewed. D. The order will have to be re-entered by the firm after one year.

If properly renewed, the order will not lose its place on the book. This is true of any order that has been entered. The GTC order expires at the next confirmation date on the last business day of April or October. Therefore, it is possible that it could automatically expire in less than six months. The order must be reviewed by a principal of the firm at a minimum of every six months. Because a GTC order will automatically expire at the next confirmation date, Choice (D) is a false statement

decrease the money supply.

If the FOMC sells government securities in the open market, it will:

Federal funds rate.

If the Fed begins selling securities in the open market to tighten credit, what is the first interest rate to feel this change in the Fed policy?

purchase securities in open market operations. To counter a slowing economy, the Fed would probably try to inject liquidity into the economy by purchasing U.S. government securities from primary dealers in a repurchase arrangement. The other choices would tend to tighten liquidity.

If the U.S. economy shows signs of slowing, the Federal Reserve might:

fed fund rate will go down, Bond prices will likely rise

If the fed is making purchases in the open market what is likely to happen?

He is not entitled to the dividend.

If the holder of a call tenders an exercise notice after the ex-dividend date for a cash dividend, which of the following statements is TRUE?

The specialist on the NYSE has received an order via SuperDot. All of the following are true, except: A. If it is a buy order, the specialist will execute it at the lowest ask; if it is a sell order, the specialist will execute it at the highest bid. B. If there is excessive trading, the floor governor will determine the price at which the order will be executed. C. If the order has been placed with the crowd, the highest bid and the lowest ask are not used. D. The order must be given loud and clear to the floor.

If there is excessive trading, the floor governor will determine the price at which the order will be executed. This is not true — the floor governor has no say or control over the pricing of securities executed with the specialist or anyone else. This order entered via SuperDot goes directly to the specialist. The specialist can either execute the order out of the book at the lowest ask (if a buy order) or the highest bid (if a sell order), and then announce the trade to the crowd; or, the specialist can give the order to the crowd to execute. If the order is placed with the crowd, the bid and ask are not used; rather, the trading is between the bid and the ask. Of course, the order must be loud and clear. [Module 13, NYSE, Sections 1.3 & 1.5]

Two survivors continue as co-tenants with the decedent's estate.

If three individuals have a tenants in common account with your firm, and one individual dies, which of the following statements is TRUE?

dividend distributions will be reinvested at net asset value and capital gains distributions will be reinvested at net asset value

If you invest in a front-end load mutual fund and choose automatic reinvestment, you should expect that: I. dividend distributions will be reinvested at net asset value. II. dividend distributions will be reinvested at the public offering price. III. capital gains distributions will be reinvested at net asset value. IV. capital gains distributions will be reinvested at the public offering price.

a properly-executed assignment to the brokerage firm on the reverse side of the certificate.

If your client has a certificate registered in his own name, to be a good delivery, the certificate must be accompanied by:

An investor trades in options with his registered rep. Lately, he has been branching out and is now going to trade in foreign currency options. He asks his rep how the options will settle when he is trading them. You tell him: A. In Euro currency only B. In the foreign currency of the option only C. In American dollars only D. In both American dollars and the foreign currency

In American dollars only. As long as the options are trading on the U.S. option exchanges, the trading of options (buying and selling in both opening and closing transactions) is always in American dollars. If the option trades were on the European Exchange, then the options would settle in Euro currency only. However, if the question had stated that the foreign currency options were exercised, then the exercise would settle in both American dollars and the foreign currency. Please read the questions carefully. [Module 9, Options, Section 17.2]

The excess securities above 140% of the accounts debit balance.

In a customer's margin account a broker/dealer must segregate:

Issuer.

In a new municipal offering, who is responsible for hiring bond counsel?

sells securities to the customer with an agreement to buy them back

In a repurchase agreement between a broker/dealer and a large institutional customer, the broker/dealer:

twice the margin call.

In a restricted margin account, if a customer fails to pay for a new purchase, the broker/dealer must sell out stock with a value of:

FIFO.

In a rising price environment, which of the following inventory valuation methods will result in the highest reported earnings?

a minimum of 10 years of variable payments, followed by additional variable payments for life.

In a variable life annuity with 10-year period certain, a contract holder receives

Short straddle.

In a volatile market, which of the following option strategies carries the most risk?

syndicate members on a pro rata basis.

In an undivided syndicate, liability for unsold securities rests with the:

property values. AN INCREASE IN PROPERTY VALUE IS GOOD

In analyzing a municipal government obligation bond, an increase in all of the following would be a negative indication EXCEPT:

long puts

In determining a violation of position limits, short calls are aggregated with:

decrease.

In its attempt to increase the money supply, the Federal Open Market Committee (FOMC) buys T-bills. This action should cause the yield on T-bills to:

True interest cost.

In its notice of sale in the "Bond Buyer", an issuer states that it will take into consideration the timing of interest payments when evaluating bids. The issuer will be using which of the following methods in its bid selection?

The call loan rate

In margin accounts, the interest rate charged to customers by a broker/dealer is most closely related to the

Likely bid.

In municipal bond language, what is a workable indication?

With the purchase confirmation

In the case of an unsolicited order, a prospectus must be delivered to the purchaser of a unit investment trust:

with the purchase confirmation. A purchaser of newly issued securities must receive a prospectus no later than by receipt of the purchase confirmation. However, any solicitation must be preceded or accompanied by a prospectus.

In the case of an unsolicited order, a prospectus must be delivered to the purchaser of a unit investment trust:

to the client either before or during the sales solicitation.

In the sale of open-end investment company shares, the prospectus must be delivered:

A customer of yours has been purchasing a variable annuity through Executive Life Insurance Company for the past 10 years. Where does the insurance company keep the annuity units for the variable annuity? A. In a custodial account at a bank B. In the separate account at the insurance company C. In the general account of the insurance company D. In a trust account at a trust company

In the separate account at the insurance company. All units of a variable annuity, both accumulation and annuity units, are kept in the separate account. They are the units that make up the separate account. These units represent the money invested in the separate account for each investor, and the value of each unit is determined by taking the total value of the separate account and dividing this value by the number of accumulation and annuity units outstanding. [Module 8, Retirement Plans, Section 2.2]

A municipal bond is trading at a premium. When determining the dollar price that a customer will pay for the bond, the MSRB rules require which of the following to be considered? A. Partial calls B. Sinking fund calls C. In-whole calls D. Catastrophe calls

In-whole calls. If a bond is selling at a premium, the quote from the broker to a client must take into consideration the fact that the bond could all be called. Therefore, the price of all premium bonds must take into account the possibility of in-whole calls, and if the bond is callable at par, the broker must quote the yield to call.

The accumulation units in a variable annuity are best described as:

Increasing units with fluctuating values. These are the units an annuity holder is purchasing. When accumulation units are purchased, the number of units purchased depends upon the price of the accumulation unit. Then, as the account accrues dividends, interest, and capital gains, more units are purchased. However, it is the value of the accumulation units that shows the change in the annuity itself. Of course, each additional payment made by the investor will continue to purchase additional accumulation units, but that is only because more money has been put into the separate account. Therefore, during the accumulation period, the number of accumulation units will always be increasing in the account. Annuity units, on the other hand, are always a fixed number of units received, but have fluctuating values as well.

Which of the following protects against systematic risk? A. Index options B. Interest-rate options C. Stock options D. Currency options

Index options. Index options and systematic risk go together. To protect yourself from systematic risk, use index options. [Module 9, Options, Section 15.1]

What is the mathematical way to measure the performance of a group of securities?

Index. An index is composed of securities, usually stocks, with a mathematical formula basis using a certain number of shares per stock x the stock price x a multiplier. Delta, beta, and R-squared are all measures of volatility.

Which of the following is a coincidental indicator? A. CPI B. Industrial production index C. Corporate profits D. CCI

Industrial production index. The industrial production index is the only coincidental index you need to know. Corporate profit: is a lagging indicator The consumer confidence index: is a leading indicator The CPI is not an indicator at all, although it is reported monthly.

Which of the following is an example of a coincidental indicator of the Department of Commerce? A. Housing starts B. Capital goods orders C. S&P 500 D. Industrial production index

Industrial production index. This is the only coincidental indicator used on the test. There are four of these indicators, but this is the only one for the Series 7.

A husband and wife have a joint account. The husband wants to sell 200 shares of a stock. He wants the check made payable to himself immediately so he can wire the cash to his wife. As his registered representative, you should: A. Advise the cashier to make the check payable to him B. Inform the customer that the check is to be made payable to both parties C. Do not execute the trade unless you have a confirmation from the wife D. Do the trade as directed and send the check to the wife

Inform the customer that the check is to be made payable to both parties. For all joint accounts, checks must be made out to both parties named on the joint account. This is true in the banking industry as well. [Module 8, Customer Accounts, Section 4.2]

bearish.

Institutional managers are moving to increase their cash position. This action would be viewed as:

Casing

Intangible drilling costs would include all of the following EXCEPT:

Which of the following is true regarding the movement of interest rates during a period of deflation? A. Interest rates have risen. B. Interest rates have fallen. C. Interest rates will stay the same. D. There is no correlation between interest rates and inflation.

Interest rates have risen. The high interest rates are causing deflation. Interest rates are high, so people do not borrow. To counter deflation the Fed will start putting money into the market, and thus interest rates will fall. However, this question is dealing with deflation, not countering deflation. This is really asking, what causes deflation? [Module 4, Economics & the Markets, Section 2.0]

The economy has been doing very well for the last two-years. A customer of yours has been investing in various stocks and some bonds. Which of the following situations would be an inflationary indication for the market? A. Interest rates start decreasing. B. Interest rates start increasing. C. Interest rates are remaining the same. D. Market prices of stock start decreasing.

Interest rates start decreasing. When interest rates are low or start falling, the availability of money and credit increases. This has an inflationary effect on the economy. The Fed prefers to have some inflation around 1%-3%. This question is asking for the first hint of a possible rapid inflationary condition. Don't confuse this with a question that states that inflation or deflation is already present and asks how it can be countered.

The U.S. dollar has gone through a deflationary period, and is now down 10% versus other leading currencies. What effect does this have on interest rates?

Interest rates will decline. High interest rates cause deflation, so to counter this movement, the Fed will decrease interest rates to stimulate the economy, and other interest rates will fall in order to compete

Trading currencies through the Interbank System would be most similar to trading in which of the following options? A. Foreign currency options B. Index options C. Listed options D. Interest-rate options

Interest-rate options. The Interbank System is used for determining interest rates, dealing in Eurodollars, and hedging interest rates. The system is dealing with the movement of interest rates and the lending/borrowing of money, much like the way interest-rate options are traded. The Interbank System is where dealers can contact each other to borrow Eurodollars and determine the interest rates.

When a broker/dealer unnecessarily comes between a customer and an efficient trade, it is called:

Interpositioning. When a B/D comes between the best market and the customer, the customer could end up paying more for the trade in commissions. This action is called interpositioning, and if the customer is charged more for the trade, the transaction is a violation of FINRA rules. Disintermediation is taking money out of banks and depositing it into other investments such as money markets and money market mutual funds. Intertrading is just trading between firms, and market making is the original trade. [Module 14, FINRA, Section 2.2]

Wall Street Company is a mutual fund with $25 million in assets. Its main investment strategy is in utility stock as a specialized investment company. Under which of the following SEC acts is the company registered? A. Securities Act of 1933 B. Securities Exchange Act of 1934 C. Maloney Act of 1938 D. Investment Company Act of 1940

Investment Company Act of 1940. Mutual fund shares are covered under the Investment Company Act of 1940, and even though they are a new issue, they are not covered under the Securities Act of 1933 as new issues. The Maloney Act of 1938 governs all broker/dealer firms since they must all be members of FINRA. The broker/dealer firms also have to meet the requirements of the Securities Exchange Act of 1934.

dollar-cost averaging

Invests the same dollar amount each period over a length of time. Lowers average cost per share over a length of time

Sell limit order

Is executed when the stock price is rising and it is at or above the limit price

PDQ, Inc., a company that specializes in deliveries, has stock trading on Nasdaq. The company has hired an underwriter to bring to the market more stock as a "when issued" stock. Which of the following best describes "when issued" stock? A. It is stock that the company has issued but might be taken off the market. B. It is stock that has been approved by the SEC, has been sold by an underwriter, but has not been approved by the present stockholders. C. It is stock that has been registered with the SEC, has been sold by an underwriter, but has not yet been delivered to purchasers. D. It is stock that has been sold by an underwriter prior to the registration being completed with the SEC and the states in which it is sold.

It is stock that has been registered with the SEC, has been sold by an underwriter, but has not yet been delivered to purchasers. The issuer has contracted with an underwriter, and sales have been made to purchasers who will receive the shares when the security is fully issued. These securities can be traded in the secondary market, but certificates have not been issued. The stock is traded in book-entry form only. [Module 6, Underwriting, Section 1.2]

A customer of yours buys common stock and then sells the stock at a loss. Within 30 days, she purchases a substantially similar security. What is true of the investor's loss on the stock for tax purposes? A. It is deducted against other capital gains. B. It is allowed if the position is held for more than one year. C. It is allowed if the position is held for less than one year. D. It is used in determining the cost basis in the newly purchased security.

It is used in determining the cost basis in the newly purchased security. This is a different way of stating that the loss cannot be deducted. Since the customer held the stock for less than 30 days and then bought a substantially similar security, the sale of the original stock is considered a wash sale. Convertible preferred stock, convertible bonds, or a call would all be considered substantially similar securities. The convertible preferred stock and convertible bond give the investor the opportunity to convert into the common stock of the company, and both will move with the value of the underlying stock. Buying a call affords the customer the opportunity to buy the stock again. [Module 17, Taxation, Section 6.4]

A customer of yours has entered a GTC buy limit order on a stock. The order is sent to the floor of the exchange, and it is put into the specialist's book. Which of the following is true of the order? A. It needs to be executed within a week or the order is cancelled. B. It needs to be renewed on a monthly basis or it is removed. C. A buy limit order cannot be entered in the specialist's book. D. It needs to be renewed at six months or it is removed.

It needs to be renewed at six months or it is removed. Firms must regularly review their outstanding orders, and every six months, the orders must be reconfirmed or cancelled, according to NYSE rules. If an order is submitted as a good-till-cancelled order, the broker/dealer is obligated to review the order and make sure that it is executed if the price of the security approaches the GTC limit price.

Below is a section of Standard & Poor's Quarterly Dividend Record relating to EGG: Declaration Payable Dividend date Ex-date Record date date $0.60 Q January 8 January 27 January 29 March 15 The dividend is paid to stockholders who have their name shown on the books of the transfer agent as of:

January 29. The record date is the date on which the transfer agent notes the shareholders who are entitled to receive the dividend, which is then sent on the date it is to be paid. Do not confuse this question for when a person has to be on the company records when the dividend is paid.

A variable life annuity can be described as a:

Lifetime payment of fluctuating payouts. A variable annuity is just what the name implies -- a changing (variable) amount of payments (annuity). In this case, it is cash flow of fluctuating payouts paid for a lifetime. Generally, annuities are payments for life, but there are occasional ones that have limited time payments

A customer exercises 120 contracts of DOW Chemical July 30 calls on Monday, July 10. When does he have to pay for the stock from the contracts?

July 13. When exercising call and put options, the payment is the same for any purchase of stock -- three business days. Since the trade is on Monday July 10, the three business days are Tuesday, Wednesday, and Thursday. Therefore, payment will be due on Thursday, July 13

A customer buys 100 shares of Intel Corp. stock on margin on Tuesday, July 1. The customer is required to pay for the purchase on:

July 7. Don't be misled by the description of the account. Customers who have margin accounts must still pay for trades, but they can use the margin balance to pay for the trade by the settlement date. July 1 plus three business days is Friday, July 4, which is a holiday. The next business day is Monday, July 7.

A customer buys 1,000 INTL Aug 70 call options on Thursday, June 18. When does he have to pay for the calls?

June 19. Regular way delivery for trading call and put options is one business day. Since the trade is on Thursday, one business day is Friday. This is when the payment for the contracts is required to be paid

Many economic theories give people direction in determining how the economy will move. Which of these theories best describes an increase in taxation and an increase in government spending? A. Supply-side economics B. Keynesian theory of economics C. Macroeconomics and the economy D. Monetarist theory on economics

Keynesian theory of economics. The Keynesian theory emphasizes the change in government taxation and spending. Many people believe that this theory means increased taxes and increased government spending, but in actuality, Keynes felt that a change in the direction of the government would stimulate an economy. The supply-side looks toward lower taxes and less government spending, while the monetarist theory affects the economy by changing the money supply. Macroeconomics is just the study of the economics of a country and the changes therein.

The economic theory that is in favor of the government using manipulation to influence the business cycle through government spending and taxation is called:

Keynesian. This is the definition of Keynesian theory. Supply-side theory advocates tax cuts (increasing the supply of money to people). Monetarist theory changes the money supply

OTC, non-Nasdaq

Last-sale information is always available for all of the following securities EXCEPT

An investor would like to buy a general obligation bond. As her registered representative, you show her a particular bond that was issued by New York City. Which of the following is the most important factor to consider for your investor's purchase of the tax-free bonds? A. Good title to the property B. Debt service coverage C. Legislative changes D. Competitive projects

Legislative changes. This is tricky. Legislative changes may affect the tax-free status of the bonds. You may want to choose debt service coverage or competitive projects, but because this question asks about a general obligation bond, neither of these factors comes into play. If the answer choices had included per capita debt, it would be a valid choice. Debt service coverage and competitive projects are factors in revenue bonds. General obligation bonds tax the general population to recoup the amount borrowed and to pay for the interest paid to investors. Good title may be applicable for revenue bonds. Are there other factors that an investor must be concerned about? Absolutely. Other factors include per capita debt, the credit rating of the municipality, and the size and wealth of the population. Essentially, this question asks whether you understand the difference between a revenue bond and a GO bond.

How many shares can be traded on the Small Order Execution System (SOES)?

Less than 1,000 shares. The SOES cannot trade more than 1,000 shares. That is the limit. [Module 12, OTC, Section 9.3]

bid and ask quotes for each market maker.

Level I Nasdaq service provides subscribers with all of the following information EXCEPT:

Which annuity pays the highest monthly payments? A. Life annuity B. Joint and last survivor C. Period certain D. Joint and two-thirds survivor

Life annuity. A life annuity always provides the highest monthly payments because when the annuitant dies, the remaining money reverts to the insurance company. For this reason, a life annuity payout option becomes a bet between the insurance company and the annuity holder. The insurance company is betting the annuitant dies before the mortality table age. The annuitant is betting she lives past that time. The second highest monthly payments can be expected from the period-certain payout option, because the payments are paid to the annuitant for the life of the annuitant regardless of how long the person lives. If the annuitant dies prior to the end of the specified time period, then payments will continue to the beneficiaries until the end of that time period. The joint and last survivor and joint and two-thirds survivor payout options generate the smallest payments. This is because the life expectancy of two individuals is involved, increasing the insurance company's chances of losing the bet

Which of the following will have the highest payout on a monthly basis for a person starting the annuity period at age 63? A. Life annuity B. Joint and last survivor annuity C. Period-certain life annuity D. Unit refund annuity

Life annuity. This will always have the greatest payout because if the person lives for one payment and then dies, the insurance company keeps all of the money left in the account. It is a straight bet, with the insurance company betting the person dies and the annuitant betting on living at least until the predicted age according to the mortality schedules used at the start of the annuity period. [Module 8, Retirement Plans, Section 4.2]

11:59 pm ET on the third Friday of the expiration month

Listed options expire at

ratio write.

Long 200 XYZ shares Short 5 XYZ Jul 50 calls at 5

Auction Rate Securities (ARSs).

Long term securities issued by municipalities that use a Dutch auction method to reset short term interest rates known as "clearing rates" are

A client buys an extended period S&P 240 LEAP call option. At expiration, the call expires unexercised. What does the client have?

Long-term capital loss. This is a LEAP and the question stated an "extended period." This must mean more than one year. Therefore, the person "owned" the LEAP for over a year, making it a long-term capital loss.

An investor has purchased 50 OEX LEAPS with a 22-month expiration date. The options expire unexercised. Which of the following statements is true of the tax treatment of the premium? A. Ordinary loss at expiration B. Short-term loss at expiration C. Long-term loss at expiration D. Short-term loss at the time of sale

Long-term loss at expiration. These are LEAPS, which have 22 months until they expire; therefore, when they do expire unexercised after 22 months, the loss has to be long-term. It is more than one year. [Module 17, Taxation, Section 6.1]

Refunding would probably occur when interest rates are:

Lower than when the issue was originally sold. Refunding usually occurs when rates drop, such as in 2010. Interest rates dropped lower than interest rates had been in the previous 75 years, so many corporations refinanced their outstanding debt with lower interest-rate bonds.

whether the trade was made as an agency transaction, whether the sale was made from the dealers inventory

MSRB rules state that a customer confirmation must indicate which of the following?

Which of the following companies would be most affected by changes in the business cycle?

Machine tool companies. The business cycle is a term used to describe the cyclic patterns of business expansion and contraction in the general economy. Cyclic businesses are affected by this pattern of expansion and contraction. Machine tool orders are usually one of the aspects of the economy that is affected first during a contraction. As people stop buying, inventories will build up and retailers will stop ordering. This causes manufacturers to stop producing and, thus, their tools do not need to be replaced as often. When tool orders stop coming in, the economy is heading into more difficult times.

The Federal Intermediate Credit Bank (FICB): A. Makes agricultural loans to farmers B. Makes loans to finance residential mortgages C. Makes business loans to veterans D. Makes loans to railroads

Makes agricultural loans to farmers. The Federal Intermediate Bank makes loans to farmers for agricultural purposes. The FICB, the Federal Land Bank, and the Bank for Cooperatives make almost all agricultural loans in the United States

the DEA. (designated examining authority)

Margin requirements on exempt securities (U.S. government securities and municipal securities) are set by:

An investor is considering investing in hedge funds. Which of the following risks do hedge funds minimize? A. Credit risk B. Market risk C. Interest-rate risk D. Liquidity risk

Market risk. There is no liquidity in hedge funds. You can only invest in and liquidate at specified times, so liquidity is practically non-existent. Credit risk is very high, since hedge funds invest in derivatives, currencies, and just about every other type of investment. The main reason they do so is to minimize market risk; however, there is a high credit risk. There is also a reasonable amount of interest-rate risk because hedge funds use leverage. By using leverage, they are borrowing, and if they do not profit by as much as the cost of the borrowing, they increase interest-rate risk. The whole reason for leveraging the investments in risky securities such as commodities and derivatives is to minimize market risk. [Module 7, Investment Companies, Section 2.6]

Tactical asset allocation.

Market timing is normally associated with which of the following portfolio management styles?

positions in margin accounts based upon current market prices

Marking to the market is used to adjust the:

positions in margin accounts based upon current market prices.

Marking to the market is used to adjust the:

When shareholders of common stock are issued cumulative voting rights, which of the following statements is true? A. If the shareholders do not vote, their vote carries over to the next election. B. Minority shareholders have an opportunity to elect a director. C. All shareholders must vote for more than one director. D. Common shareholders only have one vote for each director being elected.

Minority shareholders have an opportunity to elect a director. In cumulative voting, minority shareholders are able to pool all of their votes to vote for one or more directors. Each shareholder has one vote for each share they own multiplied by the number of directors being elected. Each shareholder can cast all votes to one director or split them amongst several directors.

A customer buys a U.S. government bond in a cash transaction at 10:30 a.m. Eastern Time on Monday April 2. She must pay for the bond by:

Monday, April 2 at 2:30 p.m. Eastern Time. The investor must pay for the trade on the trade date because this is a cash transaction. Any trade up to 2 p.m. Eastern Time must be paid by 2:30 p.m. that same day. If the trade occurs after 2 p.m. Eastern Time, the investor has a half hour to wire or deliver the money. If it were regular way trade, the money would have to be in by 2:30 p.m. Eastern Time the following day. [

Honeywell's board of directors announces a dividend to stockholders on record as of Thursday, July 5, payable on Tuesday, July 30. The ex-dividend date would be:

Monday, July 2. Make sure you do a calendar to do this problem. Start with Thursday July 5, and go back the full week. Don't forget that July 4th is a holiday, so put an H over it. The ex-dividend date is two business days before the record date. Two business days before the record date is July 2, which is actually three days due to the holiday.

A corporation has declared a dividend payable to stockholders of record on Wednesday, July 23. Which of the following statements is correct? A. Monday, July 21 is the last day an investor could buy the stock and receive the dividend. B. Tuesday, July 22 is the ex-dividend date. C. The phrase "dividend on" or "with the dividend attached" would apply from Monday, July 21 to Friday, July 25. D. Monday, July 21 is the first day that an investor could buy the stock and not receive the dividend.

Monday, July 21 is the first day that an investor could buy the stock and not receive the dividend. First, make a calendar for the days before July 21 and for the days after July 21 to find the record date. Monday, July 21 represents the ex-dividend date or two business days prior to the record date. If an investor purchases the stock regular way on Friday, July 18, she would own the stock on July 23 and thus receive the dividend with the stock purchase. If the investor purchases the securities on Monday, regular way, the investor would own the stock on Thursday, July 24, which is later than the record date. Be sure not to confuse "selling ex-dividend" with the "ex-dividend date." Tuesday, July 22 is not the ex-dividend date, but the stock does trade ex-dividend on that day. The "ex-dividend date" (or ex-date) is the first day the stock trades ex-dividend. The two days after the ex-date are just ordinary days that the stock is trading ex-dividend.

A retired investor needs the dividends from his stock investment for income. He owns stock that has been decreasing in value and is going to sell it. If the record date for the next $.25 dividend is Wednesday, June 19, when is the first day the investor can sell the stock in a regular way transaction and be able to keep the dividend?

Monday, June 17. In order to receive the dividend, the investor must have his name on the roster of owners of the stock on the record date. If he sells before June 17, his name will not be listed as an owner. Count two days back from the record date to determine the ex-dividend date, which is the first date an investor can sell the stock and still receive the dividend. No due bill is required on that date.

banker's acceptances

Money market instruments guaranteed by a bank that are used to provide capital for exporters to foreign countries are called:

Which of the following is not part of the capitalization of a company? A. Long-term bonds B. Money market securities C. Short-term registered notes D. Zero-coupon corporate bonds

Money market securities. Capitalization of a company is composed of all securities issued for more than one year. Registered notes are always going to be more than one year because they are "registered," meaning registered with the SEC. Remember that if they are less than 270 days, registration with the SEC is not needed. Therefore, both the long-term and zero-coupon bonds are long-term and part of the capitalization. The notes are more than a year, so they are also part of the capitalization. The only choice that is not long-term is the money market securities. If stock had been a choice, it would also be part of the capitalization of the company. [Module 2, Corporate Debt Securities, Section 14.1]

Which method of payment is most often used when interest is paid on CMOs?

Monthly. Today, most CMOs pay interest monthly to match the payments for the loans by the homeowners. For many years, CMOs backed by FNMA and Freddie Mac paid quarterly, but now these two are paying monthly. GNMA-backed CMOs always have paid monthly. Now, many private issuers as well as GNMA, FNMA, Freddie Mac, and others make payments monthly, while some issuers, such as broker/dealers, still pay quarterly. Very few pay interest semiannually.

A school bond is issued that has a covenant stating that "if the revenues are insufficient to pay maintenance costs and debt service the city has the authority, but not the obligation, to make annual apportionment of funds necessary to meet the debt service." This bond is a:

Moral obligation bond. These are backed by the issuer, but if the issuer cannot pay, the city, county, state, or other entity could come in and pay. They are "morally" obligated to do so, but if they don't have sufficient income, they do not need to make the payment either

Which of the following pays the interest payments and principal due on an industrial development revenue bond? A. Taxes from the municipality B. Fines and other income to the municipality C. Mortgage and lease payments D. A sinking fund

Mortgage and lease payments. Municipalities usually do not own real estate, so this is probably the best answer of those given. A sinking fund is used to make payments for a term bond, or to keep excess revenues for calling in bonds once the call date has been reached. Fines and other income go into the general fund, not to pay for a corporate bond. Taxes are for GO bonds. The only remaining answer choice is mortgage and lease revenues to pay the interest payments and principal due on the IDR, which is owned by either the municipality or a corporation. [Module 5, Municipal Securities, Section 2.3]

The bond with the 9-year call.

Mr. Donald, an analyst, is comparing two discounted 8% AA bonds. Both have 20 years to maturity. One of the bonds is callable in 4 years and the other is callable in 9 years. If interest rates fall, which will have the greatest increase in price?

revenue bonds.

Municipal bonds that are backed by the income from specific projects are known as:

Which of the following is not considered an appropriate investment for a pension plan portfolio? A. Treasury bills B. Municipal bonds C. Treasury bonds D. Corporate bonds

Municipal bonds. Municipal bonds are not considered an appropriate investment for a pension plan. Because tax-exempt interest does not benefit a plan that is already tax-deferred, municipal bonds are typically not suitable investments for a pension plan. Additionally, since funds in a pension plan grow tax-deferred, the interest generated by a municipal bond is generally lower than that of a comparable corporate bond or from Treasury bills or Treasury bonds. Treasury securities and corporate bonds are generally considered appropriate investments for a pension plan portfolio.

James A. Middlehoffer is part of his company's pension plan. He knows that the pension plan is investing in all of the following, except: A. Common stock of growth companies B. Government bonds C. High-yield corporate bonds D. Municipal bonds

Municipal bonds. Pension plans do not invest in municipal bonds. The income is tax-deferred until the money is withdrawn. Then the money is fully taxed. There is no sense getting tax-free money that will eventually be taxed. In addition, the income from municipal bonds is much lower than the income from high-yield corporate bonds.

a deferred sales load

Mutual fund Class B shares assess

unrealized capital gains.

Mutual fund shareholders are NOT taxed on:

A portion of variable annuity payments can be invested in separate accounts, which are similar to:

Mutual funds. A portion of variable annuity payments is invested in one or more separate accounts, which are similar to mutual funds. The insurance company is required to register the separate account as an investment company under the Investment Company Act of 1940. Any change in value of each accumulation unit within a separate account is based on the dividends and interest received from the investments and the net capital gains or losses on the securities in the account. Dividends, interest, and capital gains from the underlying investments are used to purchase additional accumulation units within the separate account

Within ten seconds of a trade

NASDAQ market makers are required to report trades

within 10 seconds

NASDAQ market makers are required to report trades

Suppose a municipal bond rating service wishes to evaluate the creditworthiness of a city of Santa Rosa general obligation bond. Which of the following would the rating service use in its evaluation? A. Competitive projects B. Net debt per capita C. Taxpayer's attitudes toward debt D. Operating revenues

Net debt per capita. Evaluating debt per capita (per person) is the method that is used to analyze a general obligation bond. This is because those bonds are only paid from real estate taxes; therefore, you must look to see how taxed each person is to determine if that person can pay. Taxpayers' attitudes toward debt are always negative, but that has no bearing on their ability to pay. Competitive projects and operating revenues are for revenue bonds. [Module 5, Municipal Securities, Section 9.1]

Which of the following is used to find net direct debt per capita? A. Net municipal debt over city population B. Net municipal debt over county population C. Net municipal debt over state population D. Net municipal debt plus overlapping debt over city population

Net municipal debt over city population is "net DIRECT debt." However, if the question had asked for the "net debt" (not direct), then the answer would have included the overlapping debt. Watch out for this "little" trick.

Book entry, registered

New issues of municipal securities are available in which of the following forms?

A California resident buys 600 shares of Sierra Springs Water Co., a California company, as a new offering under Rule 147. How long does she have to hold the stock before she can sell it to a resident of Nevada?

Nine months. Rule 147 offerings must be held by residents of the state in which they were issued for nine months before they can be sold to a nonresident. This investor could have sold the offering to another resident within the state of California at anytime, but can only sell to a nonresident nine months after the issue is brought to market.

Who pays the accrued interest on a defaulted municipal bond when it is purchased in the secondary market? A. The buyer B. The seller C. The broker/dealer firm where the trade occurred D. No one

No one. Since the bond is in default, there is no accrued interest to pay out. [Module 5, Municipal Securities, Section 8.5]

A client tells his rep to enter a trade to purchase 300 shares of EGG at the market. The rep enters the order and then realizes he has entered the wrong account number on the trade ticket. What should the registered rep do?

Notify the principal, cancel the order, and rebill under the correct account number. This is a simple correction, so no new transactions are required. The principal must always be notified of the problem and then must correct it. Although some firms do this automatically, reps should make the change if they see it. For the test, firms do not automatically make these corrections — someone has to correct it or the wrong account will be billed. [Module 14, FINRA, Section 2.20]

A customer calls you in the morning and tells you to enter an order to purchase 500 shares of ABC stock at the market. The order is executed as requested. Later in the day, she calls you and tells you she did not really want that stock. What should you do? A. Cancel the order B. Tell the customer she has to take the shares and she must pay for them in three business days C. Enter an order to sell the shares D. Notify your supervisor

Notify your supervisor. This is not a decision that you can make. You cannot sell the shares if the customer is denying that she wants to purchase them. The order cannot be cancelled. This will be up to the sales manager to work out. The account is frozen for 90 days if she does not pay for them. [Module 14, FINRA, Section 2.18]

Standard and Poor's dividend record is showing GTE as having the following dividend: GTE Declare date Ex-date Record date Payable date $.25 October 1 October 14 October 16 October 31 To receive the $0.25 dividend from GTE, you must be the owner of the stock on which of the following dates?

October 16. To be the owner and receive the dividend, you must own the stock on the record date — in this case, October 16. October 1 (the declaration date) is the date the issuer's board of directors declares that the dividend will be paid. October 14 (the ex-date) is the first trade date a buyer of the stock will no longer be entitled to the dividend because the trade will settle after the record date; it is also the date that the stock's market price is dropped by the amount of the dividend. October 31 (the payable date) is the date the dividend will be paid to those who were shareholders of record on the record date.

Which of the following direct participation programs are not an equipment leasing program? A. Railroad cars B. Computers C. Oil and gas pipes and casings D. Airplanes

Oil and gas pipes and casings. These are items used by an oil and gas drilling and income program. Yes, they are equipment, but they are small equipment. All of the other items -- railroad cars, computers (jumbo computers for corporations), and airplanes -- are big pieces of equipment. That is what an equipment program include

Rebecca is the president of a small, growing company. The company has grown to the point where she would like to add a retirement plan to employee benefits. She is trying to decide whether to establish a defined benefit or defined contribution plan. As her registered representative, you tell Rebecca that a defined benefit pension plan would be most beneficial to which of the following?

Older employees. A defined benefit pension plan is most beneficial to older employees because the limit on the amount that can be contributed per employee is higher than for defined contribution plans and older employees have fewer years to retirement. A defined contribution plan tends to favor younger employees since they have a greater number of years to contribute to the plan prior to retirement.

falls below the exercise price minus the premium paid.

On exercise of the option, the holder of a put will realize a profit if the price of the underlying stock:

must be canceled

On the basis of a major decline occurring within a few minutes of the close, trading is halted on all markets for the remainder of the trading day. Under the market wide circuit breaker (MWCB) rules, market-on-close (MOC) orders pending at the time trading is halted

priority, precedence, then parity.

On the trading floor, the highest bid and offer receive first consideration. When several bids at the same price occur, the trade will be awarded based on:

In a spin-off where one company, the parent company, has divested itself of some equity and some debt to form a separate company

One company is guaranteeing the debt service of another company (guaranteed bond). In which of the following scenarios is this most likely to occur?

bullish market.

One of your customers notices that the short interest on the NYSE is high. When she asks you for an interpretation, you should tell her that this signals a:

Annually

Open-end investment companies may not distribute long-term capital gains to their shareholders more frequently than:

A customer is interested in purchasing a collateralized mortgage obligation. His main concern is safety, yet he wants some yield and only wants to go eight years. Which of the following is the safest tranche? A. Companion tranche B. PAC tranche C. TAC tranche D. Extension tranche

PAC tranche. The PAC main tranche is the safest, since it has prepayment and extension risk protection. TACs do not have protection from extension risk. The companion tranche could be an extension risk companion tranche or a prepayment companion risk tranche, but in either case, they are far more risky than the main tranches of either the PAC or TAC. [Module 3, Government Securities, Section 9.0]

A 66-year-old customer of yours will be retiring in a month. She has asked you to contact the insurance company that holds the variable annuity she has contributed to over the last 15 years. She wants you to begin the process for her to take the money in a retirement payout, and spread it over the next 15 years, at least. She asks you how the money that she receives is going to be taxed. You will tell her which of the following statements? A. None of the withdrawals will be taxed since she is over 59 1/2 and she has put money in for more than the five-year requirement. B. All of the appreciation will be taken as the first payments and will be fully taxed at her tax bracket. C. Part each payment will be the return of her contribution and will be tax-free, and part each payment will be the appreciation and will be fully taxed at her tax bracket. D. Part each payment will be the return of her contribution and will be tax-free, and part each payment will be the appreciation and will be taxed as a capital gain.

Part of each payment will be the return of her contribution and will be tax-free, and part of each payment will be the appreciation and will be fully taxed at her tax bracket. There is no penalty since she is over 59 1/2, and she only pays taxes on the appreciation. Since the taxes on the annuity's appreciation are deferred, the appreciation is NOT taxed as a capital gain but as ordinary income. However, since the variable annuity is being taken as a retirement plan, she will receive her contributions and the appreciation in proportion to the amount of each in the account at the time the money is taken.

Treasury bonds and notes are quoted in terms of:

Percentage of par in 1/32 of a point. Treasury bonds and notes are quoted in terms of a percentage of par. Municipal securities are quoted in yield to maturity and corporate bonds in dollars and cents as a percentage of par. Corporate bonds also use a percentage of par, but in dollars and cents. Corporate stock is now quoted in dollars and cents, and when on the ticker tape, just in cents with no dollars.

OCC.

Performance of the terms of a standardized listed option contract are guaranteed by the:

Which of the following methods is used to calculate the municipal placement ratio found in the Bond Buyer? A. Placements for the week divided by the 30-day visible supply B. Placements for the week divided by the new issues for the week C. Total issues traded for the week divided by the new issues for the week D. New issues divided by the 30-day visible supply

Placements for the week divided by the new issues for the week. This is important to underwriters and is the only answer choice that makes sense.

Which of the following collateral mortgage obligations has the greatest prepayment risk? A. PAC CMO tranches B. TAC CMO tranches C. Plain vanilla CMO tranches D. PAC extension-risk companion securities

Plain vanilla CMO tranches have the greatest prepayment risk. This is a difficult question, so you must read it carefully. PAC and TAC CMO tranches have the least risk because the reference here is to prepayment risk. Unless "prepayment companion securities" is stated, the main tranche must be assumed. Reference may be made to TAC and PAC tranches, which are the main tranches. If the companion securities are referred to, either the word "companion" or the phrases "prepayment risk" or "extension risk" will be used. The PAC CMO tranche is the main tranche, not either of the companion securities. Since the plain vanilla has no companion securities, that must be the answer because there is no prepayment companion security to take the extra principal payments received. Therefore, the plain vanilla is the correct answer. [Module 3, Government Securities, Sections 9.1 & 9.2]

Which of the following collateral mortgage obligations has the greatest prepayment risk? A. PAC CMOs B. TAC CMOs C. Plain vanilla CMOs D. PAC CMO companion tranches

Plain vanilla CMOs. This is a difficult question, so you must read it carefully. PAC and TAC CMO tranches have the least risk because the reference here is to the main tranche. Unless "companion" tranche is stated, the main tranche must be assumed. Reference may be made to TAC and PAC tranches, which are usually the main tranches. If the companion tranches are referred to, either the word "companion" or the phrases "prepayment risk" or "extension risk" will be used. The PAC CMO companion tranche could be a prepayment companion or an extension companion tranche, so it is partly correct. Since the plain vanilla has no companion tranches, that must be the answer because there is no prepayment companion tranche to take the extra principal payments received. Therefore, the plain vanilla is the most correct answer. [Module 3, Government Securities, Sections 9.1 - 9.2]

An investor has received a dividend from his shares of Wacha Macallit REIT at the end of the quarter. How is the dividend taxed to the investor?

Portfolio income. The sucker answer is ordinary income and the other one is passive income. No, it is neither of these. This is income off an investment, which eliminates ordinary income. Passive income has to be directly from the property, as in limited partnerships. Capital gain is upon the sale of all or part of the property. If portfolio income were not an answer, then you would have to pick ordinary incom

In what order are the muni bonds allocated in a new muni bond offering? A. Pre-sale, member orders at the takedown, designated orders, net sales to the group B. Net sales to the group, pre-sale, designated orders, member orders at the takedown C. Pre-sale, net sales to the group, member orders at the takedown, designated orders D. Pre-sale, net sales to the group, designated orders, member orders at the takedown

Pre-sale, net sales to the group, designated orders, member orders at the takedown. The pre-sale is for institutional investors who buy before the interest and the price are determined. Once the issue is being sold, the sales where the largest amount of the syndicate benefits come first, then those that are designated. Finally, after the selling period is about over, the issue is doled out and the member orders are taken and filled.

What is PSA? A. Predictable prepayment speeds B. Pre-settlement advice C. Schedule of payments D. Public Securities Administration

Predictable prepayment speeds. The true meaning of PSA is Prepayment Speed Assumptions. The old association, called the Public Securities Association, now called the Bond Market Association, set up the schedule of predicted payments for mortgages. It became known as the PSA, and is today the benchmark of the predictability of how soon mortgages will be paid off. Then, depending on where interest rates are and what they are expected to do, the tranches for a new issue of a CMO (PAC, TAC, or plain vanilla) will have their tranches ready. The issuing broker/dealer will make its own amounts for the tranches, but it will be based on the PSA. [Module 3, Government Securities, Section 8.1]

A broker/dealer is usually a Nasdaq market maker in many stocks. The firm is presently acting as an investment banker for the issuer of one of the stocks that it makes a market in, and is selling some of the new issue to an investment company. In which of the following OTC markets will this trade take place? A. Primary market B. Secondary market C. Third market D. Fourth market

Primary market. This question is messing with your head. It mentions the market maker, which normally would trade in the secondary market. The question also is trying to fool you with the investment company to make you choose the fourth market. However, the fourth market is only for trades between investment companies or other institutions. The fact that the firm is now acting as an investment banker means the primary market. [Module 12, OTC, Sections 3.3 - 3.6]

One of your customers has a cash account. The market has been slipping and he wants to hedge against possible loss of market value. You tell him about how options can protect his holdings. He wants to take advantage of your suggestions, so you open an account for him and get the account approved. The option disclosure document must be sent to the customer by which of the following time frames?

Prior to the opening of the account. Always pick the earliest time of those given. In this case, the earliest is prior to the time the account is opened, the correct answer under all circumstances. All the other answers given are actually too late. [Module 9, Options, Section 13.0]

GMNA

Private lending institutions approved by GNMA originate eligible loans and sell the mortgage-backed securities to investors

ERISA covers all of the investing policies of which of the following? A. Private sector pension plans B. Individuals C. Banks and insurance companies D. Government and public pension plans

Private sector pension plans. This was the reason for the passage of ERISA — to protect company pension plans from fraudulent managers. Employees of all corporations who have retirement plans are protected by ERISA, whereas employees of the federal government and municipalities are not covered by ERISA. [Module 8, Retirement Plans, Section 9.0]

NYSE issues and NASDAQ securities

SEC regulation SHO mandates a locate requirement for short sales that is applicable to

What does Fannie Mae do?

Provides a secondary market for FHA- and VA-insured mortgages. FNMA (Fannie Mae) provides a secondary market for FHA- and VA-issued mortgages as well as conventional mortgages. The difference between FNMA and GNMA is that FNMA is actually a corporation and issues conventional mortgages, while GNMA is a government agency that only issues FHA and VA loans

A municipality has previously issued a $50 million serial bond issue. Interest rates drop by 3%, and the municipality decides to pre-refund part of the outstanding issue. They intend to use the proceeds from the new issue on the first call date, three years from now. What is the safer placement of the proceeds from the new issue from an investor's viewpoint? A. Deposit it into a bank account B. Purchase other high-grade municipal bonds C. Purchase U.S. Treasury notes D. Purchase high-grade corporate bonds

Purchase U.S. Treasury notes. The only securities that can be purchased are U.S. Treasury securities. T-bills are too short term, and bonds are usually too long term. Bank accounts are at risk for over $100,000, so not a good choice. Purchasing other muni bonds or corporate bonds is not safe enough.

Inflation Risk

Purchasing long-term AAA bonds should be concerned with what as the most risk

What is the greatest risk of a long-term bond?

Purchasing power risk. Purchasing power risk is the greatest type of risk because the money in long-term bonds is usually tied up for 10 to 30 years. For example, consider what $50,000 was worth 20 years ago versus what it is worth now

at any time within the first 30 days of the receipt

Pursuant to Federal Reserve Board Regulation T, cash dividends received in a customer's margin account can be withdrawn when?

FALL

Reinvestment risk is the chance that, after purchasing a bond, interest rates:

A municipality has been experiencing a cash flow problem. What would it use to take care of the money shortage for a short period of time? A. CLN B. BAN C. RAN D. Commercial paper

RAN. A RAN is a revenue anticipation note, which is issued to give the municipality cash until revenues start to be received on a project. If the municipality were waiting for taxes, then the answer would have been a TAN. The BAN is a bond anticipation note, issued when a bond has been approved and the underwriting is taking place. Corporations issue commercial paper for money for a short period of time. A CLN is a construction loan note, otherwise known as a PN, project note.

Which of the following has the greatest risk to an investor? A. General obligation bond B. Mortgage bond C. Industrial development revenue bond D. Railroad equipment trust certificate

Railroad equipment trust certificate. Railroad equipment trust certificates are backed by railroad cars, which could be sitting idle and losing money instead of generating money to make interest payments. [Module 2, Corporate Debt Securities, Section 5.1; Module 5, Municipal Securities, Section 2.2]

A customer purchases 1,000 shares of EGG stock for $48 per share in her margin account. The total cost of the transaction is $48,000 and the customer deposits the Reg T requirement. The B/D firm pledges part of the securities of the customer as collateral for a loan to make the full payment. What is this called?

Re-hypothecation. The customer pledges (hypothecates) the stock to the B/D in exchange for the loan to pay for the stock, and the firm then re-pledges (re-hypothecates) 140% of the loan in stock for the loan from the bank. Mark-to-the-market is checking the account for the value at the end of the day compared to the day before -- B/Ds do this daily on all customer margin accounts for SMA and margin calls. Intermediation is people putting money in the banks. [Module 14, FINRA, Section 2.11]

Only when it is used, it is not affected by market value decreasing

SMA in a long account

Microsoft, Inc. calls 1 million shares of convertible preferred stock for redemption. The corporation announces that the convertible preferred will be redeemed at $20 per share plus an accumulated dividend of $0.12. Each share of convertible preferred can be converted into 1/2 share of common. The convertible preferred stock is selling at $19. The common stock has 2 million shares outstanding with earnings of $2.50 per share, and is selling in the market at 35.75. Which of the following alternatives would be the best for a convertible preferred stockholder to pursue? A. Redeem the shares B. Convert the shares C. Sell the shares D. All alternatives would be equally attractive

Redeem the shares. By redeeming the shares, the investor will receive $20 per share. If the investor converts the shares, the investor will receive 1/2 share of common stock per share of convertible preferred stock. At a stock price of 35.75 per share, the value received per share of convertible preferred would be 17.875 per share ($35.75 divided by 2). By selling the shares, the investor will receive $19 per share. Therefore, the investor will receive the highest value by redeeming the shares at $20 per share. The $0.12 per share dividend will not add significantly to the price and there is no guarantee that the earnings will be paid to any investors.

combining separate purchases made by a client and his business partner in their respective IRA accounts. Two unrelated adults may not combine transactions to receive a breakpoint

Reduced sales charges are allowed under all of the following circumstances EXCEPT:

both written and electronic forms of communication are subject to spot checks, upon written request from finra, the member must submit that material requested within the time frame specified by FINRA

Regarding FINRA spot-checks of a member firm's communications with the public, which of the following statements is CORRECT?

A zero coupon bonds duration is equal to its maturity

Regarding a bonds duration, which measures the time it takes for a bond to pay for itself which of the following statements is TRUE?

a salesperson who works for the issuing firm's underwriter.

Regarding the sale of a new issue, a customer becomes a restricted person if he is:

next business day.

Regular way settlement for U.S. government bonds is:

US government bonds

Regulation T does not apply to what

within 5 business days.

Regulation T requires payment from a customer in a margin account:

50% of the short market value, 2,000 minimum

Regulation T requires what

Funding.

Regulations regarding how contributions are made to tax-qualified plans relate to which of the following ERISA requirements?

Transfer agent

Responsible for issuing new shares , canceling shares, and distributing capital gains

As an investor in a general obligation bond, you are least concerned with: A. Risk of changes in the market B. Risk of changes in the issuer's credit rating C. Risk of the issuer's ability to pay D. Risk that the flow of funds is paid from net or gross revenue

Risk that the flow of funds is paid from net or gross revenue. A GO, or general obligation municipal bond, is paid by property taxes, or in the case of states, income and sales taxes. This money is set aside automatically, so the flow of funds is not a factor for a GO bond. The flow of funds is only found in a municipal revenue bond. This tells the investor in a revenue bond what order the debt service and maintenance are in, along with other mandatory payments. The risk of the issuer's ability to pay is really the credit (rating) risk, so they are the same answer stated differently. All investors must consider the market price of bonds in case they ever have to liquidate due to unforeseen circumstances.

The SEC defines restricted stock owned by an insider as securities received from an issuing company in a transaction not involving a public offering and, therefore, not registered with the SEC. The rule that allows the sale of such unregistered stock under certain conditions is SEC:

Rule 144. Rule 144, also known as "the insider's rule," governs how unregistered stock and stock held by an officer, director, or principal of a company can be sold. Rule 144 specifies the amount of unregistered or restricted stock that can be sold and the time period in which it must be sold.

9

Rule 147 prohibits a nonmember of a state from selling a stock for how many months

regressive

Sales tax is what kind of tax:

Which federal act regulates insider trading? A. Securities Act of 1933 B. Investment Company Act of 1940 C. Securities Exchange Act of 1934 D. Securities Investor Protection Act of 1970

Securities Exchange Act of 1934. Since the question is not about new issues, investment companies, or the SIPC (Securities Investor Protection Corporation), the trading must be covered under the 1934 Act. Also, remember that the Securities Act of 1933 always covers new issues, so 1934 covers secondary trading

Interest rates have been declining for the last 14 months. What would the Fed do to influence the declining rates from dropping too much more?

Sell Treasury bills. The best answer in this case is the odd one out. Buying Treasury bills and increasing bond prices both drive interest rates lower. Opening the discount market means more nonmember banks will be borrowing, which increases the amount of money in the market. More available money acts to drive interest rates down. By selling Treasury bills, the Fed will take money out of circulation and thus increase, or stabilize, interest rates. Also remember that if the Fed is buying Treasuries, it is driving the prices of these securities up; therefore, interest rates would decrease.

When trying to influence the market, the Fed uses which of the following instruments? A. Sell Treasury bills B. Sell repurchase agreements C. Sell reverse repurchase agreements D. Purchase municipal securities

Sell repurchase agreements. Selling or buying Treasury bills makes the interest rate decrease or increase respectively. However, the Fed buys and sells repos on a daily basis, which influences the market much more and much faster. Since the Fed determines the amount of purchases and sales in the weekly auctions, it can influence the economy relatively quickly. However, since repos are daily transactions, they are the biggest tool the Fed uses. In addition, there is no waiting to change the discount rate monthly or quarterly. The Fed never deals in municipal securities.

An investor has purchased a stock that has been fluctuating between 56 and 72. What type of order would be entered to sell the stock when it breaks through the support level? A. Sell limit B. Buy limit C. Sell stop D. Sell stop limit

Sell stop. A sell stop is entered below the market and most often, below a support level. It is used to protect against a loss when the stock is first purchased, and once a gain is achieved, it is used to lock in the gain. [Module 13, NYSE, Section 3.5]

An investor who owns a variable annuity has been dissatisfied with the latest performance results of the variable annuity. The investor wishes to change investments, but is concerned about the possible tax consequences. As the investor's registered representative, what can you suggest regarding a 1035 Exchange? A. Sell the variable annuity and purchase a mutual fund of the same issuing company B. Sell the variable annuity and purchase another variable annuity or a fixed annuity C. Sell the variable annuity and purchase a variable life annuity D. Sell the variable annuity and purchase stock in the insurance company

Sell the variable annuity and purchase another variable annuity or a fixed annuity. The Internal Revenue Code Section 1035 allows an investor to transfer the value of the tax basis between insurance products such as variable annuities, fixed annuities, and life insurance. Of the choices presented, only selling the variable annuity and purchasing another variable annuity or a fixed annuity is allowed under this rule. However, other exchanges or "replacements" that can take place are between fixed annuities and variable annuities, and one life insurance policy for another life insurance policy. Purchasing shares of the insurance company is a sale of one type of asset and the purchase of another type of asset, and the same with a mutual fund. Only insurance products can be exchanged using a 1035 Exchange

An investor who owns a variable annuity has been dissatisfied with the latest performance results of the variable annuity. The investor wishes to change investments, but is concerned about the possible tax consequences. As the investor's registered representative, what can you suggest regarding a 1035 Exchange?

Sell the variable annuity and purchase another variable annuity or a fixed annuity. The Internal Revenue Code Section 1035 allows an investor to transfer the value of the tax basis between insurance products such as variable annuities, fixed annuities, and life insurance. Of the choices presented, only selling the variable annuity and purchasing another variable annuity or a fixed annuity is allowed under this rule. However, other exchanges or "replacements" that can take place are between fixed annuities and variable annuities, and one life insurance policy for another life insurance policy. Purchasing shares of the insurance company is a sale of one type of asset and the purchase of another type of asset, and the same with a mutual fund. Only insurance products can be exchanged using a 1035 Exchange.

You have a customer who has been with you for two years and has both a cash and a margin account. He has $550,000 invested through you. He calls you one day and wants to purchase some puts to protect his stock. When should you send the Options Disclosure Document to the customer?

Send the Options Disclosure Document prior to the BOM approving the account. Remember, you want to pick the earliest time possible. Since the client is introducing the subject of options, the rep would not know to send the ODD prior to the call from the client. Therefore, the next earliest time given is prior to the account being approved by the BOM. The other answers are after the account is opened, which is long past due.

A branch manager for a large firm has received a proposed advertisement from a registered representative (RR) in his office. The RR would like to send the advertisement to some prospects who he'd also like to cold call to solicit a particular family of mutual funds. What does the branch manager need to do? A. Nothing, because he is qualified to approve all advertising for his office B. Send the advertisement to his firm's compliance department C. Send the advertisement to the SEC D. Send the advertisement to FINRA

Send the advertisement to his firm's compliance department. Before the advertisement can be used, a registered principal must approve it. Most firms do not allow reps to send advertisements directly to FINRA, so the ad would first be sent to the firm's compliance department for review and approval prior to being sent to FINRA. The advertisement must eventually be sent to the FINRA Advertising Regulation within 10 days of use. [

The individual must notify both members of his relationship with the other

Several years after opening a brokerage account at a FINRA member firm, an individual joins another FINRA member as a research analyst. Regarding his responsibilities, which of the following is TRUE?

An individual is part of her employer's retirement plan. She is earning $65,000 at the company and wants to increase her retirement with an IRA. Which of the following is true of her contribution to the IRA? A. She can make a full tax-deductible contribution to her traditional IRA. B. She can make a tax-deductible contribution to her traditional IRA. C. She can only make a non-tax-deductible contribution to her traditional IRA. D. She cannot have a Roth IRA.

She can make a tax-deductible contribution to her traditional IRA. Since she is part of her employer's retirement plan and her income is in-between the lower and higher thresholds, she can make a "partial" tax-deductible contribution. On the test, the answer does not include "partial" -- it is just a "tax-deductible" contribution in contrast to a "full tax-deductible" contribution

An individual is part of her employer's retirement plan. She is earning $65,000 at the company and wants to increase her retirement with an IRA. Which of the following is true of her contribution to the IRA? A. She can make a full tax-deductible contribution to her traditional IRA. B. She can make a tax-deductible contribution to her traditional IRA. C. She can only make a non-tax-deductible contribution to her traditional IRA. D. She cannot have a Roth IRA.

She can make a tax-deductible contribution to her traditional IRA. Since she is part of her employer's retirement plan and her income is in-between the lower and higher thresholds, she can make a "partial" tax-deductible contribution. On the test, the answer does not include "partial" -- it is just a "tax-deductible" contribution in contrast to a "full tax-deductible" contribution.

A 55-year-old client has been with you for the last 15 years. She has been putting money into your firm's nonqualified variable annuity and has deposited $70,000 over that time. The account is now worth $280,000. As a school teacher, she has decided it is time to retire at the end of the year. She asks you to look into her variable annuity and tell her what her income and taxes will be if she decides to withdraw the money as a retirement plan. You tell her that: A. She is under 59 1/2, so all appreciation will be taken first and subject to income tax and a penalty tax. B. She is taking the value as a retirement package and will have a percentage of each payment taxed as ordinary income and a percentage returned tax-free. C. She is under 59 1/2, so all appreciation will be taken first and subject to income tax, but no penalty tax will be incurred. D. She is taking the value as a retirement package and will have a percentage of each payment taxed as ordinary income with a penalty until 59 1/2, and a percentage tax-free.

She is taking the value as a retirement package and will have a percentage of each payment taxed as ordinary income and a percentage returned tax-free. Since this is a nonqualified annuity, she will pay ordinary income tax on the part of each payment that represents appreciation, and no taxes on the part of each payment that represents contributions since she paid taxes on the money before it was contributed. Because she is taking the money as a retirement plan, she will not pay a 10% penalty for taking the money out before age 59 1/2; had she taken it as a lump sum, she would be subject to a 10% penalty on the portion that represents appreciation. [Module 8, Retirement Plans, Section 7.1]

You have a 30-year-old customer who has $60,000 to invest in her two children's (ages 3 and 5) college education. She does not want the kids to benefit from the money if they do not use it for school. What do you suggest to the customer?

She should invest in a growth mutual fund. By doing so she retains ownership and control of the money. The Coverdell ESP account has a $2,000 annual contribution limit, and since she presumably would like to fund each account with $30,000, the Coverdell is not a good option. If she places the money into an UGMA/UTMA, it becomes the children's money when they reach the age of majority, meaning the children could use the money for something other than education. In a 529 plan she is penalized if the children do not use the money for school. [Module 8, Retirement Plans, Sections 13.0 & 14.0; Module 11, Customer Accounts, Section 4.5]

Which of the following securities is not considered funded debt? A. Mortgage bond B. Equipment trust certificate C. Debenture bond D. Short-term note that had to be refunded with a long-term bond

Short-term note that had to be refunded with a long-term bond. Short-term notes are not funded debt. The long-term bond that refunded the short-term note is funded debt, but the note is not. All of the other securities listed are considered funded debt

When interest rates are fluctuating up or down:

Short-term rates fluctuate more quickly than long-term rates. When interest is fluctuating, short-term bond prices fluctuate more quickly than long-term bond prices because there is less time to maturity. Long-term bonds have a longer period of time to amortize the change, but will change more in price when they do change. Short-term instruments react immediately.

In 2010 the city of San Francisco, CA issued a 15-year general obligation bond. Starting in 2015, the city will be calling in some of the bonds in installments. What type of call is this?

Sinking fund call. Since the call is in installments, it must be a sinking fund call. If the call were part or all of the issue and at a premium, it would most likely be an optional or "in-whole" call. A catastrophe call occurs when the project has a major breakdown and is unable to continue to function. The call would be at par. The extraordinary call is usually in the first few years because not all of the money is being spent. The call is usually at par.

Which of the following would be good delivery of 600 shares of stock? A. Eight 50-share certificates; five 40-share certificates B. 10 60-share certificates C. Six 30-share certificates; six 70-share certificates D. Four 100-share certificates; six 30-share certificates; one 20-share certificate

Six 30-share certificates; six 70-share certificates. To find the answer, add the size of the certificates together. In this case, 30 + 70 = 100; therefore, this is good delivery. None of the others added together equal 100. [Module 14, FINRA, Section 5.0]

The statute of limitation for bringing an action under arbitration procedures is:

Six years. The statute of limitation for taking action through arbitration is six years. After that, no action can be take

Noncompetitive bids

What types of bids are always filled in US treasury securities auctions?

Which of the following is considered negative information about a municipality when it is issuing a new bond? A. Insuring the outstanding debt in a young, increasing population B. Stable debt, decreasing population C. Increasing debt and increasing population D. Decreasing debt and an increasing population

Stable debt, decreasing population. This is the only negative direction. Decreasing debt and decreasing population is fine, as there are less people to pay for the debt. Insuring the outstanding debt is always beneficial. Decreasing debt and increasing population is outstanding as there are more people to help pay

Public utility.

Stock of which of the following companies trades on equity?

negotiation.

Stock prices in the over-the-counter market are determined by:

the Chicago Stock Exchange

Stocks that are listed on the NYSE can also be listed on

Which of the following is the underlying financial backing for Sallie Mae securities? A. PAC CMOs B. TAC CMOs C. Plain vanilla CMOs D. Student loans

Student loans. Sallie Mae, the Student Loan Marketing Association (SLMA), issues securities that allow student loans to be sold to investors. Corporate securities, government-issued securities, and CMOs do not back Sallie Mae securities. Since CMOs are backed by government agency loans, they are similar to Sallie Mae securities; however, CMOs represent mortgage-backed securities.

Interest on U.S. government bonds is:

Subject to federal income tax but exempt from state income tax. This is due to a Supreme Court ruling saying the Feds cannot tax state-issued interest and states cannot tax federal government-issued interest

The limited partners in an oil and gas direct participation program are unhappy with the general partners who are managing the assets, and the direction the program is taking. Which of the following actions can be taken by the limited partners to remove the general partners and not lose their limited liability status? A. Fire the general partners and hire new ones B. Vote the general partners out and hire new ones C. Sue in a court of law to remove the general partners and have the court appoint a new general partner if one is needed D. Hire a new general partner and have that general partner remove the existing general partners

Sue in a court of law to remove the general partners and have the court appoint a new general partner if one is needed. If the limited partners take any of the actions in the other choices, they become general partners. The only way they can keep their limited liability status is to go to court and sue for the removal and, if needed, the appointment of a new general partner

The economic theory that says the government should use good government fiscal policy and tax cuts to influence the business cycle and make a healthy economy is called:

Supply side. Supply-side economics supports tax cuts and reduced government spending, while -Keynesian supports increases in taxes and government spending. - Monetarists believe that the Federal Reserve should take an active role in changing the money supply. -Macroeconomics is the study of the broad state of economic affairs

Many economic theories indicate in which direction the economy will move. Which of these following theories best describes the impact on the economy if there is a decrease in taxation and a hands-off policy of government spending? A. Supply-side economics B. Monetarist economic theory C. Macroeconomics theory D. Keynesian economic theory

Supply-side economics. Supply-side theory holds that lowering taxes can stimulate the economy by putting more money in the hands of people who will then spend it. Supply-side theorists also feel that a hands-off approach is important to strengthen the economy. Keynesian theory focuses on increasing government taxation and increasing government spending. Monetarists believe in influencing the money supply through changing interest rates, the daily actions of the Fed in the repo market, and the issuance of T-bills. Macroeconomics is the study of the behavior of the overall economy.

The economic theory advocating that the government should use good fiscal policy and tax cuts to influence the business cycle and make a healthy economy is called:

Supply-side. This is the definition of supply-side economic theory. Supply side advocates lower taxes and spending, while the Keynesian theory advocates higher taxes and spending. Monetarists believe in changing interest rates to influence the economy. Macroeconomics is not an economic theory-- it is the broad study of the economy. [Module 4, Economics & the Markets, Section 4.3]

A member firm is participating in the underwriting of a municipal bond. After completing the order period, which of the following documents does the syndicate member firm review to determine the amount of concession it will receive? A. Syndicate contract B. Account letter C. Syndicate settlement letter D. Agreement among underwriters

Syndicate settlement letter. The syndicate settlement letter includes the final syndicate agreement with all the information as to securities sold and amount of concession to be received. The other three choices are all referring to the same document -- the account letter, also known as the agreement among underwriters, the SYNDICATE letter, the syndicate contract, and the syndicate agreement, is distributed before the order period and contains details about which firms are responsible for how much of the offering, the type of account (Eastern or Western), and how the allocation priorities will be assigned if the issue is oversold.

FALSE

T or F: Member firms must grant permission in writing prior to any outside business activity on the part of the associated person.

Trader A works for ABD, Inc. a broker/dealer firm. Trader A receives an order to purchase 2,000 shares of NANOS at the market. Trader A calls up the firm making a market in NANOS and asks for a quote on 2,000 shares. Trader B gives Trader A a quote of 5.25 at .75, firm for an hour. A short time later, Trader C calls Trader B and asks for a quote on all 3,000 shares that Trader B owns. What does Trader B do?

Tell Trader C that 2,000 shares are out firm for an hour, but that Trader C can have 1,000 shares immediately. Watch out for this question, as there is no "five-minute recall." The quote is firm for the hour, and the trader has that whole hour. The stock can be shown, but must be stated they "are out firm until ." to anyone who wants them. If the five-minute recall had been mentioned in the question, the answer would be choice B, but no trade can be confirmed to Trader C until Trader B decides yes or no to the purchase.

principal OTC trades

The 5% markup applies to what

primary market transaction.

The 5% markup policy would apply to all of the following equity transactions EXCEPT:

Trading authorization.

Which of the following documents must an existing customer sign to establish a discretionary account?

You have a customer who thinks that real estate is an excellent investment. He decides to invest in a 6% CMO with a 10-year amortization. Which of the following is the best description of this investment? A. The CMO typically pays a 6% annual guaranteed return paid monthly with an ending period in 10 years. B. The CMO typically pays a 6% annual guaranteed return paid monthly over a 10-year period, unless the bonds are paid off early. C. The CMO typically pays a 6% annual return paid monthly over a 10- year period, unless the bonds are paid off early. D. The CMO typically pays a 6% annual return paid monthly with an ending period in 10 years.

The CMO typically pays a 6% annual return paid monthly over a 10-year period, unless the bonds are paid off early. Be careful on this. It is not a guaranteed return. It is paid monthly, though it could be quarterly or semiannually. The time period is 10 years, provided that the securities are not paid off early and thus called prior to the time the amortization period is up

Which of the following entities establishes Regulation T requirements for the purchase of securities on margin through a broker/dealer that is a member of FINRA and the NYSE? A. The SEC B. The FRB C. FINRA D. The NYSE

The FRB. The FRB (Federal Reserve Board) establishes the initial margin requirement for all securities purchased through a broker/dealer. Regulation T of the Securities Exchange Act of 1934 established the authority of the FRB to establish initial margin requirements. [Module 10, Margin, Section 1.0]

the national debt.

The Federal Open Market Committee's (FOMC) open market operations affect all of the following EXCEPT

income tax rate.

The Federal Reserve Board can manage the money supply with all the following tools EXCEPT

Which of the following is used to keep track of interest rates for, and disseminate information on, Eurodollar bonds? A. The Euromarket system B. The Interbank System C. The Intermarket system D. The Fed

The Interbank System. This is the system for keeping track of interest rates put out by foreign banks for Eurodollar bonds worldwide.

Random selection.

The Options Clearing Corporation uses which of the following methods to assign exercise notices?

Which of the following statements is true about the planned amortization class (PAC) companion securities? A. The PAC companion securities increase the prepayment risk to the holders of that tranche. B. The PAC companion securities reduce the prepayment risk to the holders of that tranche. C. The PAC companion securities eliminate the prepayment risk to holders of that tranche. D. The PAC companion securities have the same prepayment risk as the PAC tranches.

The PAC companion securities reduce the prepayment risk to the holders of that tranche. The object of the companion securities is to reduce the risk of prepayments. That is what makes the PACs and the TACs so much more attractive than the plain vanilla CMOs and the regular pass-through mortgage-backed securities. [Module 3, Government Securities, Section 9.2]

Official statement.

Which of the following documents would include information about the issuer's financial condition?

A registered representative (RR) who is registered with her broker/dealer in only one state receives a call from a customer in another state. The RR's firm is recommending a security that the RR would like to recommend to the customer. Which of the following is true regarding the recommendation? A. The RR cannot recommend the security because it is against blue-sky laws. B. The RR can recommend the security if she discloses she is not in the same state as the customer. C. The RR can sell the securities to the customer if she is registered in the customer's state, even if her broker/dealer is not registered in that same state. D. The RR can sell the securities to the customer if she registers in the customer's state and the broker/dealer is also registered in that state.

The RR can sell the securities to the customer if she registers in the customer's state and the broker/dealer is also registered in that state. To sell securities to a customer, the issue, the broker/dealer, and the registered representative must all be registered in the customer's state. An RR cannot register in a state unless her broker/dealer is also registered in the same state. Disclosure of registration status is not a requirement, but the RR must be registered to recommend a security.

Suzi Quay tells her RR to buy 300 shares of TUF, Inc. stock in her personal account. The RR enters the order and the order is executed. As the confirmation is sent, the RR notices that 800 shares of stock were purchased instead of only 300 shares. What must the RR do? A. The customer will have to send a notice in writing that she only wanted 300 shares, not 800 shares. B. The customer will have to take all 800 shares, since that is how the trade was executed. C. The RR will have to cancel the trade and then re-enter the trade with 300 shares. D. The RR is to notify the manager immediately and have the correct amount of shares credited to the account.

The RR must notify the manager immediately and have the correct amount of shares credited to the account. Then the back office will work out the problem. It is not up to the RR to make the changes. The customer is not to be involved with it in any way unless absolutely necessary. [Module 14, FINRA, Section 2.18]

Sam Jones calls and tells his registered representative (RR) to buy 500 shares of APL stock for his personal account. The RR enters the order and it is executed. The shares are purchased for the account, but as the confirmation is being sent out, the RR notices that the wrong account number is listed on the confirmation. What must be done?

The RR must tell her manager and get the manager's signature to change the account number. The RR must notify the manager of any errors. The back office of the broker/dealer should only act upon the instructions of managers who are approved to submit correction requests. Neither the representative nor the customer can make any account corrections.

the U.S. government.

The Securities Act of 1933 requires securities issued by all of the following to register and be subject to prospectus provisions EXCEPT:

regulates trading im the secondary market, created the SEC

The Securities Exchange Act of 1934

John Maynard Keynes.

Which of the following economists supports demand-side economics?

An investor wants to purchase a CMO for his granddaughter's college education. In explaining these to the investor, you want to differentiate between the two different types of CMOs. Which statement is true regarding the liability of the PAC and TAC CMOs? A. The PAC has greater liability because of the prepayment risk. B. The TAC has greater liability because of the prepayment risk. C. The PAC has greater liability because of the extension risk. D. The TAC has greater liability because of the extension risk.

The TAC has greater liability because of the extension risk. The PAC and the TAC both have prepayment risk companion tranches, diminishing the prepayment risk in both the PAC and the TAC. Only the PAC has extension risk protection, leaving the TAC with exposure to extension risk liability. [Module 18, Scenario-Based & Financial Statement Questions, Sections 3.1 - 3..3; Module 3, Government Securities, Section 9.2]

a sale of an issue of $5 billion worth of Treasury bonds maturing in 2025. The Trust Indenture Act of 1939 requires all corporate debt issues of $50 million or more sold interstate to have a trust indenture; U.S. governments are exempt.

The Trust Indenture Act of 1939 covers all of the following securities transactions EXCEPT:

An RR is looking at the balance sheet of ABC, Inc. He wants to know the book value of the company. What does this tell him?

The actual value of the company for each shareholder. The book value shows the actual value of the company based on what it owns and what it owes. It is the shareholder's true valuation. The liquidity of the company is found through the current ratio and the acid test ratio. The acid test is more stringent, but they both show liquidity. The capitalization of the company is found in the debt/equity ratio. The amount the company will make in the coming year can only be found directly from the company and its earnings.

Which of the following statements about technical theories is correct? A. An analyst is bullish when volume is heavy in a declining market and bearish when volume is light in an advancing market. B. Reduced short interest tends to make for a technically weak market. C. The advance-decline theory is a good indicator of the strength of a bull or bear market. D. The odd-lot theory states that the small investor is usually right.

The advance-decline theory is a good indicator of the strength of a bull or bear market. All the other statements are just the opposite of the way they should read: Odd-lotters are wrong, not right; bearish is when volume is heavy in a decline, and a reduced short interest indicates that stocks may increase in price, which indicates a technically strong market.

A customer of yours is 65 years old. She has made regular contributions to a Keogh for 25 years. She is now retiring. Which of the following is true regarding her withdrawals? A. The amount can be withdrawn tax-free. B. The amount can be withdrawn but is fully taxable as ordinary income. C. The amount can be withdrawn but is partially taxable as ordinary income and partially taxable as a capital gain. D. The amount can be withdrawn but is fully taxable as ordinary income plus a 10% tax penalty.

The amount can be withdrawn but is fully taxable as ordinary income. Keogh plans, similar to other retirement plans, are made up of pretax contributions and appreciation. The full amount is taxable upon withdrawal as ordinary income. Pension plans do not produce capital gains and since she is over 59 1/2, no tax penalty is applied, whether the withdrawals are taken in a lump sum or at regular intervals. [

Working capital

The amount of money a corporation has available to work with if it liquidates its current assets and pays off all of its current liabilities

In the paid-in surplus.

The amount paid in excess of par value on the sale of common shares by an issuer is reflected in which of the following accounts on the corporate financial records?

Holding Company Depository Receipts (HOLDRS)

Trade on exchanges in round lots. investments that will distribute the cash dividends of the companies held in the portfolio directly to the investor

Which of the following statements is true regarding a variable life annuity with 10-year certain? A. The beneficiary receives payments for 10 years after the annuitant dies. B. The annuitant has to pay for 10 years. C. The annuitant and beneficiary are guaranteed to receive payments for 10 years. D. The annuitant receives payments for 10 years.

The annuitant and beneficiary are guaranteed to receive payments for 10 years. This is the definition of a 10-year certain life annuity

An investor has contributed to a variable annuity for the last 15 years. He is about to retire and selects a payout option in his variable annuity for a life annuity 20-year certain. Which of the following is true? A. Only the annuitant will receive payments for 20 years, and if he dies before 20 years has passed, his beneficiary will not receive any payments. B. The annuitant or the beneficiary will receive payments for 20 years. C. The annuitant or the beneficiary can renegotiate the contract in 20 years. D. The annuitant will get payments for life, and after he dies, the beneficiary will get payments for another 20 years.

The annuitant or the beneficiary will receive payments for 20 years. A life annuity 20-year certain is an investment that provides for annuity payments to the annuitant for at least 20 years. If the annuitant lives beyond the 20 years, he will continue receiving payments until he dies. If the annuitant dies before this time, his beneficiary will receive payments until the 20 years has expired. Neither the annuitant nor the beneficiary can renegotiate the contract in 20 years. [Module 8, Retirement Plans, Section 4.2]

Which of the following are found in the Bond Buyer 40 Bond Index? A. The average yield of 20-year general obligation bonds on a particular day B. The average yield of 20-year general obligation bonds in a particular week C. The average yield of 20-year general obligation in a particular month D. The average yield of 30-year revenue bonds on a particular day

The average yield of 20-year general obligation bonds on a particular day. The Bond Buyer 40 Bond Index gives investors interested in bonds the average yield on 40 municipal bonds with 20 years to maturity as of a certain day, which happens to be Thursday. Average yields in a particular week or month are incorrect choices. In addition, the index for revenue bonds is the 30-year index. Be careful to watch for one-word differences in the answer. You need to remember four indexes: The three GO 20-year indices -- the 11-bond, 20-bond, and 40-bond -- as well as the Revenue (REV) 30 Bond Index.

U.S. government bonds.

Which of the following interdealer trades does NOT settle in clearinghouse funds?

if written notification is given to the firm and the representative receives written approval.

Under FINRA rules, a registered representative is permitted to borrow money from a customer:

When an annuitant dies, the remaining value of the separate account goes to which of the following in a life income only annuity?

The insurance company. This is a life annuity; therefore, when the person dies, only the insurance company gets the money. That was the bet. [Module 8, Retirement Plans, Section 4.2]

Industrial development revenue bonds.

The interest from which of the following bonds might be included in the alternative minimum tax calculation?

Collateralized mortgage obligations.

The interest on which of the following instruments is subject to taxation at the federal, state, and local levels?

long-term interest rates are high and beginning to decline.

The best time for an investor seeking returns to purchase long-term, fixed-interest-rate bonds is when

Who decides for a company if some of the outstanding shares of stock are bought back and made into treasury stock?

The board of directors. The board of directors determines if stock is to be issued, called back in and retired, or treated as treasury stock. The SEC and the state securities department have no say in what a company can and cannot do regarding its stock. They just register the securities before they are issued. The stockholders do have some say in the issuance of more shares, but not the calling in of stock.

Hammers and Sutcliff, a law firm with a specialty in writing legal opinions on municipal bonds, has written a qualified legal opinion. By issuing a qualified opinion, it is stating:

The bond counsel has some concerns about the tax-exempt status of the bond. If there are any situations that could cause the municipal bond to lose its tax-exempt status, they must be included in the opinion. When these situations are named, the opinion becomes a qualified legal opinion

The city of San Rafael is issuing serial bonds that are redeemable prior to maturity. How would they determine the order in which bonds in a sinking fund are selected for redemption? A. The bonds would be picked by random lot. B. The bonds would be picked by decreasing chronological order. C. The bonds would be picked by increasing numerical order. D. The bonds would be picked by the ones with the highest coupon rate.

The bonds would be picked by decreasing chronological order. In this way, the municipality will pay off the debt sooner. Since longer maturities generally have higher interest costs than shorter maturities, the city will decrease its costs as well. If the bonds in this question were term bonds, they would be selected for redemption via the random method. [Module 5, Municipal Securities, Section 8.8]

A technical analyst compares advances to declines to measure which of the following?

The breadth of the market. The advancing issues and the declining issues mark the breadth of the market

Your customer asks you to help him invest in a REIT. The REIT investment could be in any of the following types of investments, except: A. The construction of apartment buildings and hotels B. The building of a toll road C. The building of a new shopping center D. The building of high class hotels

The building of a toll road. REITs invest in real property, not land. They build professional offices, hotels, motels, and shopping centers. They are looking for income. Toll roads are usually built with municipal bonds because they are for the benefit of the general population

par and the call price.

The call premium on a municipal bond trading above par is best described as the difference between:

bond resolution.

The call provisions of a municipal issue would be detailed most completely in the:

Which of the following is used in evaluating a real estate limited partnership? A. The yields of similar programs B. The work experience of the general partner C. The projected deductions that will be generated D. The cash flow analysis

The cash flow analysis. The intent to make money is first and foremost the answer. You may disagree personally, but this is the answer that the test considers correct. [Module 16, DPP, Section 2.0]

An ARS has a reset period for a $2 million share of ARS. There are enough bids received, and the ARS "Dutch auction" is completed. The rate at which all of the bids are accepted is called:

The clearing rate. The clearing rate is the rate that all of the accepted bids will receive. When there are enough bids, the highest bid of those accepted is used and is called the clearing rate. If there are too many bids, the lowest bids that equal the offering are accepted and are called the minimum bid. If not enough bids are received, no bids are taken and all investors will continue to hold their ARS but will now receive the maximum bid, which is set by the issuer for the "failed auction."

Which of the following statements is true regarding foreign currency options with American settlement? A. The exercise settlement is in U.S. dollars only. B. The exercise can only be on the expiration date. C. The seller must deliver foreign securities. D. The contract size is fixed.

The contract size is fixed. The contract size on all options is fixed. Equities have 100 shares, and each foreign currency option has a fixed amount of currency. Remember that American settlement is any time before expiration, while European settlement is only on the expiration date. [Module 9, Options, Section 17.3]

120.

Under FINRA rules, a registered representative must complete the regulatory element of continuing education within how many days of a registration anniversary date?

Which of the following statements is true regarding foreign currency options with American settlement? A. The exercise settlement is in U.S. dollars only. B. The exercise can only be on the expiration date. C. The seller must deliver foreign securities. D. The contract size is fixed.

The contract size is fixed. The contract size on all options is fixed. Equities have 100 shares, and each foreign currency option has a fixed amount of currency. Remember that American settlement is any time before expiration, while European settlement is only on the expiration date. [Module 9, Options, Section 17.3]

Nominal Yield

The coupon on a bond can be described as its:

An investor writes a two-month call for a large premium. What is the maximum amount the investor can lose?

The current market price of the stock, less the premium and the strike price. Remember that the investor will be called, but will also be paid the strike price and has already been paid the premium. [Module 9, Options, Section 2.0]

as a percentage of its market value.

The current yield of a callable bond selling at a premium is calculated:

You have a customer with a cash account. He tells you that he understands he can make money when stock goes down by selling the stock short. He asks what happens to the outstanding stock that he sells short. You tell him which of the following?

The customer can borrow the stock for any length of time, but must return the stock when the broker/dealer demands its return. The customer can have the stock for any length of time, but upon the B/D's demand for the return, the customer must enter a buy order to replace the stock that was borrowed. There is no time frame -- in fact, Tutorial 17 tells of a person who was short Lionel Train stock for two years and finally covered the short position for a $23 profit. It was still a short-term gain, but the customer was short that whole time. [Module 4, Economics & the Markets, Section 9.0]

One of your customers calls and tells you to buy 700 shares of EGG at 39. EGG is an exchange-traded stock. You enter the order and the trading department is able to get 400 shares at 39 and 200 shares at 38.85. After that, no more trades are available at 39 or better. What should you do regarding the customer?

The customer gets the 600 shares at the executed prices and the remainder of the order is outstanding until the end of the day. The firm is not obligated to fill the order at the price requested. If the firm was a market maker in the stock and it was an OTC stock, the firm could fill the order at that price but is not required to do so. The order was not an FOK, so the trading department does not cancel the order; they just fill as much as they can and leave the rest as an order to be filled that day. If not filled that day, the order is then cancelled. Only if the order was entered GTC would it stay outstanding until executed or cancelled by the customer. This is not an IOC order, so whatever is not filled is cancelled. [Module 13, NYSE, Section 3.2]

A new customer of yours calls and purchases 400 shares of MRD stock at $46 per share. Two days later the stock drops to $40 per share. The customer calls and tells you he does not want the stock. Which of the following statements are true? A. The customer can refuse to pay and the firm will own the stock. B. The customer can sell the stock back to the firm at the same price. C. The registered rep can buy the stock from the customer to stop the customer from having a loss. D. The customer owns the stock and must pay at least the margin amount by the fifth business day.

The customer owns the stock and must pay at least the margin by the fifth business day. The customer cannot just elect not to pay for the stock. The customer made a verbal, legal agreement and must abide by it. If the customer does not pay, the account will be frozen for 90 calendar days and all trades will be on a cash first basis. The RR cannot protect the customer from loss and the firm has no obligation to buy the stock back if the customer does not want it. In this case, the firm can sell the stock in the open market and the customer will have to absorb the loss.

Efficient Market Theory

The investment theory that assumes the market adjusts automatically as new information becomes known or relevant about a particular security, thereby resulting in a fair market price at all times, is known as:

A customer of yours purchased 1,000 shares of General Motors stock six months ago at $51 per share. The stock had risen to $68 per share and lately has dropped back to $65. It does not seem likely that it will rise in price again, so she calls and tells you to sell the stock at $65. The order is entered immediately; 700 shares are sold, but the price drops below $65. There are no more trades at 65 that day. What should you do regarding the order for the customer? A. Since you did not sell all of the order, you must cancel the entire order. B. The B/D must buy the rest of the order for the firm's inventory. C. The customer sells the 700 shares at the price at which they were executed; you must cancel the remainder of the order at the end of the day, and notify the customer immediately after trading has ended. D. The customer sells the 700 shares at the price at which they were executed; the rest of the order remains outstanding, and you must notify the customer immediately after trading has ended.

The customer sells the 700 shares at the price at which they were executed; you must cancel the remainder of the order at the end of the day, and notify the customer immediately after trading has ended. The firm is not obligated to buy the rest of the shares at the price requested. If the firm was a market maker in the stock and it was an OTC stock, it could buy the stock at that price, but is not required to do so. The order is not an FOK, so the trading department does not cancel the order; they sell as many shares as trading allows, leaving the rest to be sold, if possible, that day. If the shares are not sold by the end of the day, the remainder of the order is cancelled and the customer is notified immediately to make a decision for the remaining shares

Which of the following describes how a sell limit order is used if a customer is monitoring a stock by following technical indicators? A. The customer will enter a sell limit order with a price that is below a support level for the stock. B. The customer will enter a sell limit order with a price that is at or above a resistance level for the stock. C. The customer will enter a sell limit order with a price that is at or above a support level for the stock. D. The customer will enter a sell limit order with a price that is at the support level for the stock.

The customer will enter a sell limit order with a price that is at or above a resistance level for the stock. A limit order to sell means the customer wants to sell the stock when the price is increasing and has reached a specific point -- but before it declines. A resistance level indicates that the stock may not rise higher. A support level indicates that the stock may rise above this level. In this scenario, the customer intends to sell the security and is trying to determine when the stock will be at its highest and is only interested in the resistance level. An investor may want to buy at the support level, which indicates that the stock should rise. [Module 4, Economics & the Markets, Section 6.2]

A customer of yours is purchasing a corporate bond. She has purchased 50M IBM 10s 16 for a price of 106.55 "and interest." What is meant by the term "and interest"?

The customer will pay a price of 106.55% on the $50,000 plus accrued interest. The term "and interest" means that the accrued interest will be added to the price of the bond. All of the other answers mean nothing.

A customer places an order to buy 800 shares of VFH stock. Two days later the contra broker sends a DK notice stating the trade was entered for 500 shares -- not for 800 shares. Which of the following statements is true? A. The customer has to take only the 500 shares. B. The trade must be cancelled and reentered for 800 shares. C. The customer will receive the first 500 shares at one price and the subsequent 300 shares at a different price. D. The trade stands and the firm has to fill the full order at the same price as the original 500.

The customer will receive the first 500 shares at one price and the subsequent 300 shares at a different price. The answer, "the trade stands and the firm has to fill the full order at the same price as the original 500" would be true if this were a buy limit order. If this were a limit order, and there were other trades at the same price, the firm would be required to fill the order at the price of the original trade, or better. If a higher price is paid, the firm must make up the difference. However, if there were no other trades at that price, then the firm would not be required to fill the full order of 800 shares. In this question, the customer is not at fault or responsible for correcting the error; however, since the trade was a market order, the customer must pay the price of the trades. The firm has the option to make up any extra costs incurred in the trade, although it is not required. [

Spread

The difference between the syndicate bid and reoffering price on a competitive bid

A customer of your invests in mutual funds and has all dividends and capital gains reinvested in the fund. The investor purchased 5,000 shares six-months ago. The fund issues a capital gain distribution of $3,000. How is this treated for tax purposes?

The distribution must be claimed that year as a long-term capital gain. All capital gains distributions to the holder of an investment company must claim the gains that year and they are ALWAYS taxed as a long-term capital gain. [Module 17, Taxation, Section 6.6]

The issuers. They do however regulate dealers, quotes and sales representatives

Who does the MSRB not regulate?

The time value of money refers to: A. Present dollars stated in terms of a designated rate in return B. The dollar value of future returns, stated in relationship to the value of present dollars C. The dollar value of future returns, stated as a percentage of investment D. Present dollars stated in terms of yesterday's dollars

The dollar value of future returns, stated in relationship to the value of present dollars. This is the definition of time value of money

An investor buys an OEX call option. Later he exercises the call. How does he settle?

The dollar value of the index option times the multiplier. Since the OEX is an index option, and indexes are just numbers, the only thing that can be delivered is money. The option is in the money the amount that is delivered. Always choose an answer that includes the phrase "times the multiplier." [Module 9, Options, Section 15.0]

Which of the following should be the primary concern of an investor considering an investment in a direct participation program? A. The conflicts of interest on the part of the syndicator B. The tax benefits of the program C. The economic feasibility of the program D. The ability to control the syndicator

The economic feasibility of the program. Actually, an investor should be concerned about all of these things. However, since the question asked for the primary concern, economic feasibility is correct. This consideration determines if the partnership will make money. Actually, this is the IRS's primary concern; however, it is interpreted as an investor's primary concern.

Stagnation

The economy has experienced several months of slow economic growth and rising unemployment. Market analysts would describe this period as ______

A corporate bond analyst is least concerned with: A. The estimated earnings of the corporation B. The fixed charges coverage C. The debt/equity ratio D. Current interest rates

The estimated earnings of a corporation. A corporate bond analyst is concerned with earnings, but less so than with fixed charge coverage, debt/equity ratio, and current interest rates. The bond analyst is concerned about the company's ability to pay interest and principal to bondholders. Bondholders will receive interest and principal payments before an investor in the company's stock will receive any benefit from earnings. Bond interest is paid before taxes are paid as well. Earnings are the amount that remains after taxes have been paid. The fixed charges coverage shows how much will be available to pay off the bonds. The debt/equity ratio shows how much the company has borrowed or how leveraged the company is. Current interest rates affect how marketable the bonds are in the secondary market -- if the company needs to sell additional bonds, interest rates will affect the fixed charges coverage for the corporation.

NYSE

The ex-date for NYSE-listed issues is set by

the NYSE.

The ex-date for NYSE-listed issues is set by:

The federal funds rate is likely to go down, and bonds prices will likely go up

The fed makes purchases in the open market, what does this do?

heavy open market purchases by the Fed.

The federal funds rate has been decreasing. A likely cause would be:

that banks charge each other for overnight loans and is subject to daily change.

The federal funds rate is the rate:

exchange traded transactions

The finra 5% rule does not apply to what?

A customer calls you in the morning and tells you to enter an order to purchase 200 shares of ITEC stock at the market. The order is executed as requested. Later in the day, she calls and tells you she did not really want that stock. What happens?

The firm sells out the customer and freezes her account for 90 days. The customer must pay for the securities or the firm has the right to sell her out and freeze her account. She is given the choice to pay for them or let the firm sell them; however, the registered reps try their best to get the customer to pay. If the customer does not pay, the account is frozen

fluctuates based on the values of other currencies.

The floating exchange rate for a nation's currency:

In the past three months, the U.S. dollar has been revalued. What is happening to foreign currencies in the open market?

The foreign currencies are being devalued. As one currency goes up, others go down in relation to it. This is one of the "odd man out" types of questions -- which one is different? The currency getting stronger and the revaluing of the currency are the same thing. Staying in proportion is not possible.

Which of the following best describes how Sallie Mae bonds are backed? A. The full faith and credit of the federal government B. Interests in pools of student loans C. The full faith and credit of the Student Loan Marketing Association (SLMA) D. A pool of mortgages

The full faith and credit of Student Loan Marketing Association (SLMA). Since the SLMA issues the bonds, the entity backs the bonds. Many people believe that the government ultimately will back the bonds, but only if Congress votes to do so. The U.S. government does not guarantee these bonds

Treasury bills are backed by:

The full faith and credit of the U.S. government The full faith and credit of the U.S. government. T-bills are backed by the full faith and credit of the U.S. government

I and II.

The function of a broker's broker in the municipal bond business is to do which of the following?I. Help sell municipal bonds that a syndicate has been unable to sell. II. Protect the identity of the firm on whose behalf the broker's broker is acting. III. Help prepare bids for an underwriting syndicate. IV. Serve as a wholesaler, offering bonds at a discount from the current bid and offer.

The number of bonds involved in the transaction, yield to maturity

The initial confirmation of a when-issued municipal bond contains which of the following

Confirms the bonds to the member that did not sell its share.

The syndicate manager takes which of the following actions in a divided municipal syndicate that does not sell out?

A 40-year-old married individual with a nonworking spouse is making $200,000. The individual wants to contribute to a Roth IRA. Which of the following is true? A. The investor cannot contribute to a Roth IRA because of her income. B. The investor can contribute $5,500 to a Roth IRA, but it is taxed prior to the contribution. C. The investor can make a $5,500 tax-deductible contribution to a Roth IRA. D. The investor can make a non-tax-deductible contribution for both herself and her husband for $5,500 each.

The investor cannot contribute to a Roth IRA because since her income exceeds the maximum Roth IRA limit for a married individual filing jointly. Refer to Table 5 of the Current Tax Tables found under Module 00. She could contribute to a traditional IRA if she had no retirement plan through her employer. [Module 8, Retirement Plans, Section 12.0]

An investor sells short 100 shares of TA stock on the day before the ex-dividend date. TA has declared a $0.25 dividend on the stock. How does this affect the investor's account?

The investor's account is debited $25. Since the investor is borrowing the stock, he must pay the dividend to the person from whom he borrowed the stock. Two people are entitled to the dividend because there are two owners, but the company only pays one person. The investor who is short is not hurt because the price of the stock drops that much anyway. [Module 11, Customer Accounts, Section 5.1]

An investor wants to know about the new type of securities that are being traded -- ETNs. What do you tell him about the investment risk of ETNs?

The issuer could go bankrupt prior to the maturity of the ETN. Even though an extremely strong bank or broker/dealer issues them, there could be actions in the marketplace that cause the issuer to go out of business, such as bad investments (like Barings Bank in the 1990s that went bankrupt due to a speculative trader employed at the bank). Other investors have nothing to do with the issuer, and the SEC does not investigate securities.

foreign branch of a domestic bank.

The issuer of an ADR is a

foreign branch of a domestic bank.

The issuer of an ADR is a:

Sarbanes-Oxley.

The legislation that required SRO's to establish research analyst conflict of interest rules for its members is:

Which of the following is required in a partnership agreement? A. The limited partner's signature B. The limited partner's contribution C. The general partner's contribution D. The signature of the state administrator

The limited partner's signature. A partnership agreement is an agreement between the limited partners and one or more general partners in a general partnership. Both the limited partners and the general partner(s) must sign the agreement. This agreement provides for the general partner(s) to have limited powers of attorney-in-fact over the limited partner's investment in the partnership. The partnership agreement is filed with the state administrator, but the administrator does not sign the agreement. [Module 16, DPP, Section 1.3]

A registered representative is looking at the balance sheet of ABC, a new company. He determines the current ratio for the company. What does this figure estimate?

The liquidity of the company. The current ratio and the acid-test ratio are used to estimate the liquidity of a company. These ratios can also show how fast a company can come up with cash if needed. The value of a company can only be estimated using various accounting measurements, because the true value can only be calculated if the company is purchased. Book value is an estimate of the company's value based on the net assets of the company. Debt/equity ratio is an estimate of a company's capitalization structure and portrays how much money the company has borrowed in relation to the amount of equity available to common shareholders

A limited tax general obligation bond is limited in terms of what? A. Other debt issues B. The tax rate C. The number of taxpayers subject to those taxes D. The number of purchasers

The tax rate. The city can only raise taxes up to that limit to pay for the bonds

An investor is reviewing a direct participation program as an investment. This particular investment is a real estate program investing in raw land. What is the main advantage of this type of investment for this investor?

The long-term appreciation and the low costs of maintenance. The chief advantages with this direct participation program (limited partnership investment) is the long-term investment returns and the low cost of maintenance for the raw land. Raw land does not have any depreciation expenses and the taxes are minimal. The only benefit of raw land is the possibility of appreciation.

the operating expense fund

The management fees paid by an investment company are part of

the operating expense of the fund

The management fees paid by an investment company are part of

secondary market

The market in which exsisting securities are purchased and sold is the

A customer calls after the market closes and tells you to enter an order for the next day. He wants the order to be a market order at the opening. How does this order's priority compare to a marketable limit order?

The market order at the opening has priority over the limit order. This is because it is a market order; market orders take priority and are filled first. Then limit orders are filled if a willing buyer or seller, depending on the transaction, is willing to buy or sell at the limit price.

30% of the market value or $5 per share, whichever is greater.

The minimum maintenance requirement on short stock selling above $5 is:

A person who is a wholesaler for American Masters Funds comes to your office to encourage you and the other registered reps in the office to sell their funds. He tells all about the funds and their benefits. Which of the following must the wholesaler mention? A. The name of the portfolio manager B. The top 10 holdings C. P/E ratio for the last month D. The earnings per share

The name of the portfolio manager. This is a real toss-up, but since the investments are always in a state of change, the top 10 in the fund today may not be in the fund next week. It is the manager of the fund that is so important since this person makes the decisions for the fund. The P/E ratio is for stock, not mutual funds, and the earnings are not the dividends received, but the earnings of the company that pays the dividends. These latter two have nothing to do with mutual funds. [Module 7, Investment Companies, Section 3.2]

The purchase price of a no-load fund is determined by:

The net asset value as computed at the end of the business day. All investment companies must compute the NAV daily (some do it twice), so they usually do it at the end of the day. Since it is a no-load, no sales charge or commission is added; since it is a fund, or open-end investment company, there are no supply and demand restrictions on the price. Remember, a no-load fund has to be offered by an open-end investment company because such companies are the only ones that deal in loads (or lack thereof). Closed-end investment companies have commissions

interest rates fall abroad and the u.s. dollar strengthens

The net asset value of an international bond fund can be expected to increase if:

U.S. government transactions settle:

The next business day. U.S. government securities settle the next day in a regular way transaction -- memorize this. Other securities currently settle on the trade date plus three days, or T+3, and options settle the next day.

required to be delievered to purchasers at or before settlement date

The official statement is usually used to

specialist (designated market maker).

The opening quote for issues listed on the NYSE is set by the

specialist (designated market maker).

The opening quote for issues listed on the NYSE is set by the:

It insures against loss due to theft.

Under MSRB rules, which of the following statements regarding a fidelity bond is TRUE

For two years an investor has owned 800 shares of a stock for which he paid $57 a share. He then writes June 70 call options on the underlying stock for a premium of 7. The options expire. The investor subsequently sells his stock at $68. What is the investor's cost basis in the stock after the options expire? A. The cost of the stock less the premium received B. The original cost of the stock C. The cost of the stock plus the premium received D. The sale price of the stock

The original cost of the stock will always be the cost basis. Any options bought or sold against the underlying stock will have the premiums either added to the strike price (with a call) or subtracted from the strike price (with a put) in determining the proceeds or cost basis. The premium never gets included with the stock. In this case, since the options expire it becomes a short-term gain, and the subsequent sale of the stock will be a separate transaction and have its own gain (or loss if the price had been lower). The options have no bearing on the stock position for tax purposes. [Module 17, Taxation, Sections 3.0 & 6.1]

Feds fund rate

The overnight repo interest rate is usually lower than bank rates and just below or comparable to the

A customer of yours has invested $30,000 in a 529 plan for his children's education, which is now worth $50,000. The children are three years apart, and when the first one is in her last year of college, the second child starts her first year. There is still plenty of money for the second child. Which of the following statements is true regarding the proceeds in the 529 plan? A. The money can only be used for the one child for whom it was invested. B. The excess money can be used by the second child, but there will be a penalty tax. C. The parent can withdraw the excess money and use it tax-free for the second child's college education. D. The parent can withdraw the excess money and use it tax-free for his own use

The parent can withdraw the excess money and use it tax-free for the second child's college education. The money may be used for any child in the family, not just the designated child. If the designated child does not use all of the money, any money that child takes for anything other than education will be subject to income tax plus a penalty on the amount that is appreciation from the investment. The contribution will be income and penalty tax-free, but the contribution is ALWAYS used first.

The payments in a variable annuity are closely tied to:

The performance of the securities in the separate account. The amount of annuity units that will be paid out each month, multiplied by the annuity unit value at the time of each payment, determines the actual dollar amount of each payment made out of a variable annuity. The performance of the management company affects the account but does not determine the payments. The Dow Jones Industrial Average is often used to measure how stocks are generally doing, but it has no direct effect on variable annuity payments.

offered for sale in the market that week.

The placement ratio in "The Bond Buyer" indicates the relationship for a particular week between the number of bonds sold and the number of bonds:

the total takedown.

The portion of a municipal bond underwriting spread that remains after the syndicate manager subtracts the management fee is:

buy a security in a falling market and buy it in a rising market

The practice of dollar cost averaging requires the investor to:

525.

The premium on the XYZ Jan 30 calls is 3 - 3.15, while the premium on the XYZ Jan 30 puts is quoted at 2.25 - 2.35. A customer establishing a short straddle receives total premiums of:

State governments

Who gets the least amount of their revenues from property taxes?

One broker firm tells another broker firm that the price of QPR stock is "$56.30 net." What does this mean?

The price of the stock does not include the commission. Even though the term "net" normally indicates a markup has been included when dealing with a customer, the net from broker to broker does not include markup. The term "net" refers to the total cost to the purchasing broker. Be careful with this type of question. Make sure you understand the difference between customer and broker-to-broker transactions. A transaction between firms does not include a markup or commission.

Under normal economic conditions, which of the following comparisons of different interest rates is true? A. The discount rate is above the prime rate. B. The fed funds rate is above the prime rate. C. The prime rate is above the broker loan rate. D. The discount rate is above the broker loan rate.

The prime rate is above the broker loan rate. The prime rate is above the discount rate offered between banks, and the prime rate is used as a benchmark for lending to corporations and individuals. Banks generally lend to individuals at a rate above prime. Broker/dealers use the broker loan rate for lending to margin customers. Broker/dealers are financial institutions and margin loans are highly regulated. The fed funds rate is the lowest rate of all the rates given. PRIME BL RATE DISCOUNT FEDERAL

Which of the following is true of adjustable rate U.S. government bonds? A. The interest rate on the bonds will change when the discount rate changes. B. The interest rate on the bonds will change with inflation and deflation. C. The principal amount of the bonds will change with interest rates. D. The principal amount will change with inflation and deflation.

The principal amount will change with inflation and deflation. Adjustable rate U.S. government bonds are really called TIPS, or Treasury Inflation Protected Securities. The principal adjusts with inflation and deflation, but will never decrease below the value at the time it was purchased. Since the principal amount changes, not the interest on the bonds, each payment the investor receives can increase or decrease.

The city of San Francisco is selling a general obligation bond. Which of the following statements is true after the bond is issued? A. The city can use the funds to get its finances in order. B. The ratings for the bond do not change with the sale of the bonds. C. The rating for the bond changes due to the larger amount of debt the city has incurred. D. The property taxes on the city will not change.

The ratings for the bond do not change with the sale of the bonds. The ratings for the bond are set prior to the issue being sold and are determined with the assumption that the whole issue will be sold. Therefore, there is no change in the debt amount of the city compared to when the bond was rated. The funds can only be used for the purpose the bond was issued, but property taxes will increase

2nd business day preceding the record date

The regular way ex-dividend date for cash dividends is the:

within 120 days of the 2nd registration anniversary and every 3 years thereafter.

The regulatory element of continuing education must be completed:

A registered rep purchases two tickets to a college football game with the intention of taking a client to the game. The tickets are worth $75 each, but because rep is a former football player and part of the college's alumni association, he gets them for free. The day before the game, the rep is called out of town on a family emergency and cannot attend the game. Which of the following is true? A. The rep cannot give either ticket to the client since he's not able to attend the game with the client. B. The rep can give both tickets to the client because the price he paid for both tickets is less than the gift limit. C. The rep cannot give both tickets to the client because the combined face value is more than the gift limit. D. The rep can give both tickets to the client because there is no gift limit when giving tickets to a game.

The rep cannot give both tickets to the client because the combined face value is more than the gift limit. The face value of the tickets, not the price paid, determines if the rep can give the tickets to his client. Since the face value of one ticket is $75, the rep can give one of the tickets to the client. However, even though the rep did not pay anything for the pair of tickets, he cannot give both tickets to the client since the combined face value exceeds the gift limit

A registered rep has been licensed for five years. At the end of the second year, the representative completes the regulatory education as required. At the end of the fifth year, the representative has not completed the next phase of the regulatory education in the time required. Which of the following is true? A. The representative cannot conduct business until his regulatory education has been completed. B. The representative is fined and sanctioned. C. The representative must retake the qualifying exam to have his license reinstated before conducting any business. D. The representative may continue to contact existing and prospective clients, but may not execute trades until his regulatory education has been completed.

The representative cannot conduct business until his regulatory education has been completed; this includes contacting clients or prospects. FINRA does not fine or sanction the rep, or require that he retake the qualifying exam.

private sector organizations only.

The requirements of ERISA apply to pension plans established by:

4

The secondary trading od securities is comprised of how many markets?

Two market makers who are members of the National Market System make a trade. They are required to report their trades on Network A. Which of the following statements is true? A. The seller must report the trade in 30 seconds. B. The buyer must report the trade in 30 seconds. C. Both buyer and seller must report the trade in 30 seconds. D. Both buyer and seller must report the trade at the end of the day.

The seller must report the trade in 30 seconds. FINRA requires all trades to be reported in 30 seconds. The answer for the test should be that the seller reports. The rule actually states that "between designated reporting members, the seller reports." However, when the trade is between a nondesignated reporting member and a designated reporting member, the designated reporting member must report, whether he is the buyer or seller. For the test, always say that the seller reports.

Which of the following is true regarding statutory voting for the directors of a company by the common shareholders? A. If the shareholders do not vote, their vote carries over to the next election. B. The minority shareholders get a chance to elect a director. C. The shareholders must vote for more than one director. D. The shareholders only get to vote one vote per share of stock owned for each director being elected.

The shareholders only get to vote one vote per share of stock owned for each director being elected. In statutory voting, the shareholders can only vote as many times for each director being elected as they have shares of stock. This is different from cumulative voting. In statutory voting, if a person has 300 shares and four directors are being elected, they can vote three hundred shares for each of four directors that are running for election. If it had been cumulative, all 1,200 votes could have gone to one director.

difference between the bid and production (the price at which the bonds are reoffered to the public).

The spread in a municipal competitive bid is the:

Which of the following affects the value of the Euro? A. The strength of the European economy B. Its relationship to the U.S. dollar C. The U.S. fed funds rate D. The corollary coefficiency of all other currencies

The strength of the European economy. All currency valuations are affected by the economies in their respective country. The Euro relationship with the dollar only affects those traveling between the U.S. and the European nations using the Euro. The fed funds rate does not affect the Euro, nor does the Euro affect the fed funds rate. However, changing the discount rate by the Fed can have an effect, as it can have an effect on any currency. However, the fed funds rate is not affected.

Which of the following is a violation of federal laws regarding tender offers? A. The tender of stock from a cash account B. The tender of stock from a long margin account C. The tender of a minor's stock from a custodian account D. The tender of stock that has been borrowed by a customer in a short margin account

The tender of stock that has been borrowed by a customer in a short margin account. To tender stock is to sell stock to an individual or firm that has made an offer. To tender stock from a short account is not possible, and is a violation, because it is borrowed stock. The customer does not own the stock he is tendering

straight line amortization.

The term for the annual reduction of a municipal bond's cost basis purchased at a premium is:

series.

The term that describes options of the same exercise price and expiration date for the same underlying security is:

interest and principal on a U.S. government issued bond.

The terminology "guaranteed full faith and credit" is most applicable to:

either competitive bid or negotiated underwritings.

The terms of municipal general obligation (GO) and revenue bond offerings may be set by the issuer as:

its premium.

The time value of an option that is at-the-money equals:

The trading department of your firm calls a market maker in ACCOB stock to purchase 400 shares of the stock and is given a quote of 23.25 - .50, good for one hour with a five-minute recall. Which of the following statements is true regarding the quote?

The trading department has control of ACCOB stock for one hour, but within that hour the market maker can call trading back with a five-minute fill or kill. When given a five-minute recall, the trader giving the quote can call back and give the other trader five minutes to decide (although the other traders don't really get five minutes). Just remember that this is called "a five-minute fill or kill." [Module 12, OTC, Section 5.2]

An investor buys a security that is in book-entry form. Which of the following statements best describes the transaction? A. The transaction is the transferring of securities and the entry by the issuer or transfer agent. B. The transaction is the transferring of securities between the buying and selling brokers. C. The transaction is an order that is entered on the books of the clearing agent. D. The transaction is an order that is entered in the books of the broker/dealer.

The transaction is the transferring of securities and the entry by the issuer or transfer agent. Book-entry form means there is no certificate; therefore, the issuer, or its transfer agent, holds the names of the owners of the securities. Any sale between parties is just a notation on the books of the issuer or transfer agent

redeem shares and handles name changes of mutual fund ownership

The transfer agent of a mutual fund. I. redeems shares. II. sells shares. III. holds custody of fund securities. IV. handles name changes of mutual fund ownership.

bond covenants.

The trust indenture of a revenue bond includes a statement explaining rates will be maintained at a level sufficient to cover the debt service and operating expenses. This statement would be found in that part of the indenture dealing with the:

Which of the following is true regarding the effect of a change in interest rates? A. The value of preferred stock will move in the same direction as the change in interest rates. B. The value of preferred stock will move in the inverse direction of the change in interest rates. C. The value of preferred stock will not move in a relationship to the change in interest rates. D. The change in interest rates will have no effect on preferred stock.

The value of preferred stock will move in the inverse direction of the change in interest rates. Preferred stocks are quasi-debt securities in that they have a stated value. Therefore, with a set dividend rate they act like bonds, moving inversely to the movement in interest rates. This is because as interest rates rise, the yield on the preferred stock will decrease, and vice versa.

An investor has purchased a variable rate government bond. After owning the bond for some time, the interest rates on newly issued bonds start to rise. How will the change in rates affect the owner of the variable rate bond? A. The value of the bond will increase. B. The value of the bond will decrease. C. The rate of the variable rate bond will adjust to the new rates. D. The rate of the variable rate bond will be inversely affected.

The value of the bond will decrease . The name "variable rate" on a government bond is really a misnomer. These are actually called Treasury Inflation-Indexed Securities, or "TIPS," and their interest rate does not adjust, but the principal returned at maturity adjusts with inflation and deflation and will never be less than the par value when it was purchased. Therefore, as with all bonds, as interest rates rise, the price of the bond declines. Be careful, and do not fall into the obvious trap of thinking that the interest rates will adjust with the changing interest rates. [Module 3, Government Securities, Section 1.1]

An investor has purchased a variable rate government bond. After owning the bond for some time, the interest rates on newly issued bonds start to rise. How will the change in rates affect the owner of the variable rate bond? A. The value of the bond will increase. B. The value of the bond will decrease. C. The rate of the variable rate bond will adjust to the new rates. D. The rate of the variable rate bond will be inversely affected.

The value of the bond will decrease. The name "variable rate" on a government bond is really a misnomer. These are actually called Treasury Inflation-Indexed Securities, or "TIPS," and their interest rate does not adjust, but the principal returned at maturity adjusts with inflation and deflation and will never be less than the par value when it was purchased. Therefore, as with all bonds, as interest rates rise, the price of the bond declines. Be careful, and do not fall into the obvious trap of thinking that the interest rates will adjust with the changing interest rates. [Module 3, Government Securities, Section 1.1]

The fluctuations in the value of a variable annuity correspond with the fluctuations in

The value of the securities held in the separate account of the annuity. The fluctuations in the value of a variable annuity correspond with the fluctuations in the value of the securities held in the separate account of the annuity. The individual values of the securities held in the separate account fluctuate with general market fluctuations

stable.

The writer of a combination expects the market to be:

A writer of puts has been assigned by his brokerage firm on the day before the ex-dividend date of a $.12 dividend. Which of the following is true regarding the dividend? A. The buyer of the puts will receive the dividend. B. The writer of the puts will receive the dividend. C. Neither the buyer nor the seller of the puts will receive the dividend. D. The buyer of the puts will receive a due bill.

The writer of the puts will receive the dividend. The writer of the puts will be buying the stock since he has been assigned -- meaning the option has been exercised. Since he is buying before the ex-dividend date, the writer is buying in time to receive the dividend. Since notice is given the day before the ex-dividend date, the writer of the puts will not be able to sell the stock prior to the ex-dividend date. Therefore, the writer of the put will receive the dividend. Remember, if you buy before the ex-dividend date, you receive the dividend.

under no circumstances

Under ERISA, a plan trustee wishing to write uncovered calls may do so:

The city of Wichita hires an attorney to give an opinion on the city's new $20 million bond. The bond counsel sends back an unqualified legal opinion on the issue. Which of the following statements is true? A. The attorney is not qualified to give the legal opinion. B. The issue is not qualified to be issued under the tax-exempt status. C. There are no qualifying conditions on the opinion. D. The bond is not qualified in terms of certain areas of the opinion.

There are no qualifying conditions on the opinion. The statement is almost the opposite of what you may think. But consider it this way: It is a sunny day outside if the sun is shining. That is a qualified statement since a condition is attached. The same goes for a qualified legal opinion. [Module 5, Municipal Securities, Section 5.5]

If a company issues more preferred stock and bonds, what is the net effect?

There is increased leverage in the capital structure. The bonds are leveraged because interest is owed every year on the borrowed money. Preferred stock, although it is stock, is required to have paid a set amount of dividends if the common are to receive any dividend. Therefore, preferred stock is considered to be quasi-debt securities or leveraged securities.

If shares of one of the stocks in Standard & Poor's 100 Index (OEX) split 2-for-1, what happens to the OEX options? A. There is no change. B. The index decreases. C. The index increases. D. The stock is dropped from the index.

There is no change. When a stock has a stock split, the split reduces the price of the stock; however, since there are more shares outstanding, the total value remains the same. For this reason, there is no change to any of the indexes. [Module 1, Equities, Section 5.1]

penny stocks

These are exempt from the requirement to prepare a suitability statement but are not exempt from the disclosure rules

Series EE

These bonds are purchased at face value and earn a fixed rate of return

Treasury bills have all of the following characteristics, except: A. They are quoted in percentages on a yield to maturity basis. B. The bid will be lower than the offer. C. They are exempt from taxation by states. D. They mature from one to five years after they are issued.

They mature in one to five years after they are issued. This is not true — T bills mature in three, six, or 12 months, not in one to five years. Treasury notes mature in one to 10 years, and Treasury bonds mature in 10 to 30 years. Remember: Bills (up to one year); notes (one to 10 years); and bonds (10 to 30 years). However, all the other choices are true — Treasury bills are quoted in percentages on a YTM basis, the bid will be lower than the offer, and they are exempt from taxation by states.

An investor has just inherited $18,000. She invests it in a mutual fund and is informed of a sales breakpoint at $20,000. If she signs the letter of intent to meet the $20,000 investment for the reduced sales charge, how long does she have to meet the requirement?

Thirteen months. To meet a letter of intent from a mutual fund, investors have a maximum term of 13 months to bring their investment up to the sales breakpoint. If they have made previous investments in the same family of funds, they can backdate the letter three months. [Module 15, Federal & State Regulations, Section 14.3]

OCC exercise limit rules

Three family members each hold sizable call option positions with the same underlying equity security in their individual accounts. Over the course of 3 days, Monday through Wednesday, each of the customers calls your broker/dealer and gives instructions to exercise all of their call options in that security. You recognize this as a potential violation of

A registered representative of a firm holds a seminar on options. He tapes the session and has the compliance options principal check the tape. The registered representative then sends copies of the tape to interested investors. How long does the firm have to keep copies of the tape?

Three years. All advertisements must be kept for three years, and they must be readily accessible for the first two years.

A broker/dealer has just run an advertisement announcing to the public that the firm is now able to execute trades on the Internet. How long must the firm keep its advertisements and sales literature available for inspection?

Three years. All firms must keep all advertising and sales literature for three years on microfiche or on computer discs in the office, so that FINRA can check on the ads if a problem arises.

A corporation declares a $0.33 dividend on March 1. The record date is Monday, March 8. The ex-dividend date is:

Thursday, March 4. The ex-dividend date for a cash dividend is two business days before record date. March 8 is a Monday. The first business day before Monday, March 8, is the previous Friday, which would count as one business day. Thursday is the second business day, or the ex-dividend date.

All of the following are affected when a cash dividend is declared, except: A. Total assets B. Total liabilities C. Working capital D. Stockholder's equity

Total assets. The assets are not affected at all when a cash dividend is declared, only when it is paid. [Module 18, Scenario-Based & Financial Statement Questions, Section 1.1]

The Federal Reserve Board is having its monthly meeting. The chairman has noted that the economy is starting to come under pressure from inflation. What would the board do to combat the possibility of inflation getting too high? A. Decrease the discount rate B. Buy treasury securities C. Loosen money D. Tighten money

Tighten money. All the other answers would lead to loosening the money supply and inflation would take off. The Fed tightened money in 2000, and it slowed the market and even put the economy in a spin. Nevertheless, the runaway inflation of the late 1970s was avoided by tightening the money supply. [Module 4, Economics & the Markets, Section 1.6]

accreted.

To calculate a capital gain or loss on the sale of an original issue discount municipal bond, the discount must be:

buy the near expiration, sell the distant expiration

To create a credit calendar spread, an investor should

An ADR has been issued by a bank. For which of the following is the ADR developed? A. To facilitate the trading of U.S. securities on foreign exchanges B. To facilitate the trading of foreign securities on U.S. exchanges C. To facilitate the trading of U.S. securities on U.S. exchanges D. To facilitate the trading foreign securities on foreign exchanges

To facilitate the trading of foreign securities on U.S. exchanges. ADRs represent foreign stock of companies that trade in the U.S. on the various exchanges or over-the-counter markets.

Churning

Trading in a customer's account considered excessive, and for which no discernible investment purpose is detected

For which of the following is an ADR issued?

Trading of foreign securities on U.S. exchanges. An American depositary receipt (ADR) is issued by a branch of a U.S. bank located in a foreign country. The bank buys shares of the foreign stock and reissues them in ADRs, based on the exchange rate between the currencies. One ADR could equal anywhere from one to several shares of the underlying stock. ADRs are traded in U.S. markets, including the exchanges, Nasdaq, and the Pink Sheets. ADRs are used solely for foreign stocks that are to be traded on U.S. exchanges and markets.

An issuer in which of the following industries would most likely issue an equipment trust certificate? A. Automotive B. Tobacco C. Durable goods D. Transportation

Transportation. Any industry that moves people or things will have equipment trust certificates. Here is a practical example: In 1999, American Airlines advertised that it was going to put one new airplane into service each month for a year. The company issued a large bond, and divided up the cost of the airplanes into equipment trust certificates with each airplane represented by one certificate. If the airplanes were grounded, there would be no debt service being made to that particular certificate. Risky? Yes. [Module 2, Corporate Debt Securities, Section 5.1]

I.Semiannual adjustments to principal based on the CPI., III.The interest payments will keep pace with inflation.

Treasury Inflation Protection Securities (TIPS) offer which of the following benefits to an investor?

they give semiannual adjustments to principal based on the CPI and interest payments keep pace with inflation

Treasury Inflation Protection Securities (TIPS) offer which of the following benefits to an investor?

yield to maturity

Treasury STRIPS and Treasury receipts are quoted based on

A quote of 8.7 bid, 8.55 ask would most likely be a quote for a:

Treasury bill. Remember that with most securities price quotes, the bid is lower than the ask. *With a Treasury bill quote, just the opposite is true: *The ask price is lower than the bid price of the bill, which means investors can purchase the T-bill for more than they will get for it if they sell it back to a dealer. This is because T-bills sell at a discount. The other three answer choices -- a U.S. government bond, Treasury note, and debenture -- are all quoted in the same way as other securities with the ask greater than the bid. [Module 3, Government Securities, Section 1.1]

Treasury STRIPS are derived from which of the following securities? A. Treasury bills B. All U.S. government issues C. Treasury stock D. Treasury bonds

Treasury bonds. Treasury STRIPS (Separately Trading of Registered Interest and Principal Securities) are derived from bonds and notes. The interest portion of the bond or note is stripped and then sold and traded separately. STRIPS are initially sold at a discount and the full amount of interest is paid to the investor upon maturity

An investor pays 105 plus accrued interest. Which of the following securities has the investor purchased? A. Treasury bills B. Banker's acceptances C. Negotiable certificates of deposit D. Treasury notes

Treasury notes. Both Treasury notes and negotiable CDs trade with accrued interest; negotiable CDs are quoted in terms of yield, not price. Banker's acceptances and Treasury bills are purchased at a discount and mature at par. [Module 2, Corporate Debt Securities, Sections 14.2 & 14.3; Module 3, Government Securities, Section 1.1]

Which of the following government securities are issued for two to 10 years? A. Treasury bills B. Treasury stock C. Treasury bonds D. Treasury notes

Treasury notes. Notes are issued from more than one and up to 10 years. Bonds are issued for 10 to 30 years. Treasury bills are issued for 1, 3, 6, and 12 months. Remember that treasury stock is stock that has been issued by a company and then repurchased by the company.

The federal government guarantees all of the following, except: A. Treasury bills B. Treasury receipts C. Treasury notes D. Treasury bonds

Treasury receipts. The U.S. government issues all Treasury bills, notes, and bonds. Treasury receipts are issued by broker/dealers that have purchased a Treasury bill or Treasury note and are now using that as backing for the Treasury receipt, a form of a zero-coupon bond. The B/D strips the interest payments off the Treasury bond/note, and sells the interest separately. The customer purchases the interest coupons as zero-coupon bonds, receiving no interest until the receipt matures. The customer gets all of the interest at once, although they pay taxes on the "phantom."

Which interest-bearing security is traded "without interest"? A. T-notes B. Treasury strips C. T-bills D. Treasury stock

Treasury strips. These are Treasury securities that have had the interest coupons taken off and are being sold "without accrued interest." They are (supposedly) an interest-bearing security that has lost its coupons, which are selling separately.

A client of yours is 65 years old and has been trading through you for the last 15 years. He has three grandchildren and wants to set up an account for each of them. He would like to manage these accounts and their investments. Which of the following accounts would be most suitable for the client? A. 529 plan B. Coverdell plan C. Trust account D. UGMA/UTMA

Trust account. Since Gramps wants to have control over the account, and the time span is not mentioned, we will assume it is until he dies. The 529 and Coverdell are out, since he would have no control. The UGMA/UTMA is out since the securities are turned over to the grandkids when they reach age of majority. That leaves the trust as the only logical choice. If a time span was mentioned, such as the kids are old enough to manage their account or go to school, then choose UGMA/UTMA. If the question had stated until his death, the trust would be the only answer. The question gives neither of these statements, so you have to assume until Gramps dies. Another good answer could be TOD, transfer on death, which would be the best answer given the little bit of information. [Module 11, Customer Accounts, Section 4.4]

For which of the following are nonregistered foreign securities not a suitable investment? A. Insurance companies B. Mutual funds C. Individual investors D. Trust accounts

Trust accounts. This is a very difficult choice, because a fiduciary is in charge of the trust account and must make trades in securities that are in the best interest of the trust's beneficiaries. Unregistered foreign securities are simply too risky for a trust account. As far as the individual investor is concerned, foreign securities may be too risky but that is up to the individual. Mutual funds invest in foreign securities, so the foreign securities that are registered with the SEC as an ADR are OK, and the same with insurance companies. You must interpret this question on your own. There are no specific rules, but you must know the customer and what is best for that customer. You have to use your best judgment on this type of question.

A customer purchases securities in a cash account on Tuesday, February 6 but does not pay for the securities in the required time. According to the Federal Reserve Board's Regulation T, by which date could an extension of time be requested?

Tuesday, February 13. Regulation T requires payment within five business days. A Regulation T extension must be requested on that date. February 6 is a Tuesday, so the fifth business day will be the following Tuesday, February 13, because weekend days are not counted as business days. A regular way settlement would require payment by Friday, February 9. Regulation T allows two additional business days to settle a trade, but the broker/dealer can notify the customer in the meantime that the trade could be cancelled

A customer buys 300 shares of U.S. Steel stock on Wednesday, June 20. How many days after regular way delivery has expired must he pay for the stock?

Two days. Regular way for stock is three business days. However, the question asked how many days after the regular way delivery expired must the customer pay. This is the way the exam may ask for the Regulation T date of delivery. Regulation T requires all payments by the fifth business day, or the second business day after regular way delivery expires.

The deceased sibling's interest in the account will pass to the surviving sibling in accordance with the joint tenants with rights of survivorship account registration.

Two siblings have an account with your broker/dealer registered as joint tenants with rights of survivorship. Both live in a state that recognizes community property as an ownership designation. If one of the siblings dies, which of the following will occur?

5 days

Under regulation T payment of a cash account must be made in within within when?

8.5% of the offering price

Under the Conduct Rules, the maximum sales charge on any transaction involving an open-end investment company share is:

8.5% of the offering price.

Under the Conduct Rules, the maximum sales charge on any transaction involving an open-end investment company share is:

7.

Under the Investment Company Act of 1940, redemption payments for mutual fund shares must be made within how many days?

The custodian may exercise or sell the rights as he deems prudent.

Under the Uniform Gifts to Minors Act, how may stock subscription rights or warrants be handled in a custodial account?

What is the maximum amount that can be contributed to an UGMA/UTMA account for a minor? A. $3,000 B. $5,500 C. $14,000 D. Unlimited amount

Unlimited amount. Yes, any amount can be contributed, but only $14,000 is gift-tax free. The question did not say gift-tax free, but most people automatically assume that. Do not read anything into the question that you do not see. [Module 17, Taxation, Section 6.8]

The purchase of long securities on margin, the short sale of securities

Which of the following events will cause the special memorandum account to decrease?

all parties involved

Who is binded by the Code of Arbitration?

0.01

Under the provisions of Rule 144, what percentage of outstanding stock may a control person sell every 90 days?

0.8.

Under the provisions of Rule 147, what percentage of an issuer's gross business revenues must be derived from sales within the company's home state?

a bank officer.

Under the rules governing the activities of broker/dealer firms, prior notification to the employing firm would be required in order to open a cash account for all of the following EXCEPT:

A variable contract separate account invested in one or various mutual funds must be registered as a:

Unit investment trust. Every variable annuity separate account must be registered with the SEC as either a unit investment trust or an open-end management company. The separate account is established by the insurance company and makes up the variable component of a variable annuity. It is completely separate from the general account of the insurance company, which is why it is called the separate account. If the insurance company manages the portfolio of securities purchased with the money in this separate account, the separate account must be registered as an open-end management company. If the insurance company takes the money and invests it in one or more mutual funds, the separate account must be registered as a unit investment trust. This question states that the variable contract separate account has been invested in one or more mutual funds, so it must be registered as a unit investment trust.

591/2

What is the distribution age that someone can take money out of a Keogh plan without penalty

interest

What is the main objective of a money market fund

The Yellow Sheets list which of the following? A. All bonds sold OTC B. Listed bonds sold OTC C. Unlisted bonds sold OTC D. All bonds except government and exempt bonds sold OTC

Unlisted bonds sold OTC. The lists are always for unlisted stock and bonds, and the Yellow Sheets are specifically for bonds. [Module 2, Corporate Debt Securities, Section 9.0]

The Yellow Sheets publish which of the following? A. All bonds sold OTC B. Listed bonds sold OTC C. Unlisted bonds sold OTC D. All bonds sold OTC except government and exempt bonds

Unlisted bonds sold OTC. The Yellow Sheets publish unlisted bonds. Listed bonds are traded on the exchanges or OTC and can be located through the exchanges. Since there is no other means of knowing who has unlisted bonds, broker/dealers looking to buy or sell unlisted bonds go to the Yellow Sheets.

hedging.

Using listed options to reduce the market risk in a stock position by taking an opposing position in the options that represent an equivalent number of shares is known as

Which of the following industry's stock is considered defensive stock? A. Aerospace B. Utilities C. Automotive manufacturing D. High-tech

Utilities. Utilities are defensive issues. Aerospace is a defense industry stock and automotive manufacturing is considered a cyclical stock. A defensive stock is issued by a company that does well in both good and bad times. For instance, grocery store stocks may be considered defensive stocks since grocery stores may perform better during slow economic times than during good economic times -- fewer people frequent restaurants or spend money on entertainment when times are bad. High-tech stocks are generally associated with volatile businesses that are prone to wide fluctuations in value.

.25

What is the maintenance requirement on a long margin account

Which of the following industries are affected most by rising interest rates? A. Insurance B. Railroads C. Steel manufacturing D. Utilities

Utilities. Utilities are most affected by rising interest rates because of their heavy reliance on debt for raising capital for operations and purchasing resources. Utility companies are highly leveraged and pay higher interest rates on new bond issues, which translate into fewer dividends for the common stockholders. Utility company stock prices are also negatively affected by rising interest rates.

Repurchase agreements

Utilized by financial institutions to raise short term financing. Include the repurchase price and maturity date that may be fixed or left open

You have a new customer who wants to make a lump sum investment of $150,000. The customer wants to retain her market value of the principal investment, but also wants growth in her portfolio. As her registered representative, which of the following do you suggest to this customer?

Variable annuities (Variable annuities could lose money, but there is usually some guarantee built in that assures the investor that the amount of the contribution is maintained.)

the state insurance commission and the SEC

Variable annuities must be registered with:

As a retirement vehicle, which of the following would probably provide the greatest protection against the loss of purchasing power? A. Fixed annuities B. Corporate bonds C. Variable annuities D. Series HH bonds

Variable annuities. Theoretically, variable annuities would provide the greatest protection against the loss of purchasing power among the investments listed. Historically, the underlying equity securities in most variable annuities have kept up with inflation. Equity securities are generally regarded as the most appropriate investment to protect against the loss of purchasing power in the long term. Fixed annuities, corporate bonds, and Series HH bonds all represent fixed payments, and provide very little, if any, protection against inflation. Any fixed income securities are generally considered poor hedges against inflation.

As a retirement vehicle, which of the following would probably provide the greatest protection against the loss of purchasing power? A. Fixed annuities B. Corporate bonds C. Variable annuities D. Series HH bonds

Variable annuities. Theoretically, variable annuities would provide the greatest protection against the loss of purchasing power among the investments listed. Historically, the underlying equity securities in most variable annuities have kept up with inflation. Equity securities are generally regarded as the most appropriate investment to protect against the loss of purchasing power in the long term. Fixed annuities, corporate bonds, and Series HH bonds all represent fixed payments, and provide very little, if any, protection against inflation. Any fixed income securities are generally considered poor hedges against inflation

Which of the following statements is true concerning variable annuities? A. The investment risk is borne by the insurance company, the same as in a fixed annuity. B. Payments of a variable annuity can be decreased because of an increase in the expenses of insurance company. C. Salesmen selling variable annuities are not required to register with FINRA. D. Variable annuity nontaxable separate accounts are registered under the Investment Company of 1940

Variable annuity nontaxable separate accounts are registered under the Investment Company Act of 1940. The separate account component of a variable annuity is registered under the Investment Company Act of 1940. However, in a variable annuity, the investment risk (market value fluctuation) is borne by the annuity owner, not the insurance company. In a fixed annuity, the investment risk is borne by the insurance company. Payments cannot be decreased due to increases in expenses because the expense ratio is guaranteed. All salespeople are required to register with FINRA.

Which of the following statements is true concerning variable annuities? A. The investment risk is borne by the insurance company, the same as in a fixed annuity. B. Payments of a variable annuity can be decreased because of an increase in the expenses of insurance company. C. Salesmen selling variable annuities are not required to register with FINRA. D. Variable annuity nontaxable separate accounts are registered under the Investment Company of 1940.

Variable annuity nontaxable separate accounts are registered under the Investment Company Act of 1940. The separate account component of a variable annuity is registered under the Investment Company Act of 1940. However, in a variable annuity, the investment risk (market value fluctuation) is borne by the annuity owner, not the insurance company. In a fixed annuity, the investment risk is borne by the insurance company. Payments cannot be decreased due to increases in expenses because the expense ratio is guaranteed. All salespeople are required to register with FINRA.

.30

What is the maintenance requirement on a short margin account

10% of total money raised

What is the maximum underwriting fee for a LP

Which of the following could be used as a hedge against inflation? A. Face-amount certificate B. Fixed annuity C. Municipal bond D. Variable annuity

Variable annuity. Of the answers given, only the variable annuity could be used as a hedge against inflation. A face-amount certificate is based on real estate mortgages and fixed payments, which could not be expected to keep pace with inflation. Fixed annuities also have fixed payment amounts and are not good during inflationary times. Since municipal bonds also have fixed interest payments and the investor will only receive the principal back at maturity, they would not provide a good hedge against inflation either.

Which of the following could be used as a hedge against inflation? A. Face-amount certificate B. Fixed annuity C. Municipal bond D. Variable annuity

Variable annuity. Of the answers given, only the variable annuity could be used as a hedge against inflation. A face-amount certificate is based on real estate mortgages and fixed payments, which could not be expected to keep pace with inflation. Fixed annuities also have fixed payment amounts and are not good during inflationary times. Since municipal bonds also have fixed interest payments and the investor will only receive the principal back at maturity, they would not provide a good hedge against inflation either.

Beta is a measure of which of the following? A. Risk B. Leverage C. Liquidity D. Volatility

Volatility. Beta is a measure of volatility based on the movement of a stock price against the market index. Beta can be with one stock, a basket of stocks, and/or with a portfolio. Alpha is a measure of risk independent of the market, comparing the makeup of the company to how the company is progressing. Liquidity is the ability to dispose of the securities when necessary without losing too much money. Leverage is borrowing money to make money. [Module 4, Economics & the Markets,

30% of the current market value

What is the minimum maintenance in a short margin account?

100,000

What is the minimum net worth of for a company hoping to make a public offering?

75000 contract

What is the position limit in a combined position

A covered call writing program.

What options trading program would be most appropriate for a retired customer with a portfolio of low cost basis blue-chip stocks who is seeking income from his portfolio?

Writing naked calls

What type of option strategy would be most risky in a bull market

The board of directors of Tandem, Inc. declared a 50% stock dividend to stockholders of record on Friday, April 4, payable on April 30. The stock will be selling ex-dividend on Thursday, May 1, so all trades after which of the following dates will no longer require the delivery of a due bill?

Wednesday, April 30. This question involves a stock dividend that is greater than 25%, which means that the ex-date will be the day after the payable date. Therefore the date that due bills no longer need to be attached to the delivery of stock is the ex-date. Yes, the ex-date is after the record date for all stock dividends of 25% and greater. Since the question asks for after which date the due bills are no longer required, the answer is April 30 -- the day before the ex-date

against any other security

What can losses of a municipal bond be applied against

NEW ISSUES

What does the act of 1934 not apply to?

the account must be turned over to the donee

What happens to UGMA accounts when the minor reaches the age of majority

it is increased

What happens to leverage when a company issues additional bonds

Some securities are mispriced and value can be captured through security selection.

What is a basic assumption in an active management investment style?

maturity, coupon, issuer. NOT THE PRINCIPAL AMOUNT

What is considered for a tax swap in a wash sale

The day after the trade date

What is regular way for US government bonds?

Confirmation showing the purchase price and the official statement

What is required to be given to a retail customer at settlement of a municipal new issue

The time value is zero.

When XYZ stock trades at 40 and an XYZ Oct 35 call trades at 5, which of the following is TRUE?

The option is at parity.

When XYZ stock trades at 40 and an XYZ Oct 35 call trades at 5, which of the following statements is TRUE?

whether the transfer agent has accepted the securities.

When a customer enters a sell order and the customer is in possession of the certificates, a broker/dealer must determine all of the following EXCEPT:

customer's name and deliver the securities to the customer.

When a customer instructs a registered representative to transfer and ship, the representative instructs the margin department to transfer ownership into the:

All gains and losses are recognized on the transfer date

When a customer transfers the proceeds of a sale from one fund to another within the same family of funds, what are the tax consequences?

To which of the following transactions does the FINRA 5% Policy not apply? A. Listed stock traded in the OTC market B. When a firm is the market maker in a stock sold to a customer C. A principal transaction D. When a firm acts as an agent to another member firm

When a firm acts as an agent to another member firm, the 5% policy does not apply. The 5% policy does not apply to securities that are sold by prospectus, or to transactions between member firms. When a broker/dealer buys or sells for a client in the OTC market or on an exchange &mdash whether acting as a broker and effecting a transaction, or acting as a dealer and trading for its own account &mdash the trade is covered by the FINRA 5% Policy. This is true whether the trade is for listed stock traded on an exchange, in the OTC market, or for unlisted stock trading in the secondary market OTC

pay operations and maintenance.

When a municipality is allocating funds from a revenue-producing facility under a net revenue pledge, the first priority is to:

At what point is a person subject to FINRA rules and disciplinary actions? A. When the person submits an application to work for a member firm B. When the person submits an application to take a test given by FINRA C. When a principal of the firm signs the person's Form U-4 D. When the person passes an exam given by FINRA

When a principal of the firm signs the person's Form U-4. It is at this point that the employee is subject to the firm's policies and procedures, as well as FINRA rules, regulations, and disciplinary action. Simply submitting an application to work for a firm or taking a test given by FINRA does not subject a person to FINRA's rules, regulations, and disciplinary actions, nor does passing a FINRA exam. [Module 14, FINRA, Section 1.1]

II, I, IV, III.

When a registered representative opens a new options account, in which order must the following actions take place?I. Obtain approval from the branch manager. II. Obtain essential facts from the customer. III. Obtain a signed options agreement. IV. Enter the initial order.

securitization.

When assets are pooled into financial instruments such as Collateralized Mortgage Obligations (CMOs) to better facilitate selling them to the general public, the process is known as:

any-time

When can American-Style Options be Exercised

Rule 144.

Which of the following exemption provisions of the Act of 1933 may NOT be used for an initial offering of securities?

An increase in current market value.

Which of the following factors will NOT increase SMA in a customer's short margin account?

Receipt of a cash dividend.

Which of the following increases SMA?

Registrar.

Who is responsible for ensuring that a corporation does not have more shares of stock outstanding than it has been authorized to issue?

Long calls and short puts, Short calls and long puts.

When determining a position limit, a member firm aggregates which of the following customer positions?

$175 per share.

When disseminating information about transactions of OTC equity securities, 1 share equals 1 round lot for stocks trading at or above:

Immediately

When do IRA's become vested?

last day of trading

When do yield based options expire?

before the first order

When does a customer have to receive the Options Disclosure Document?

two days after the ex-date

When is the record date

When each purchase is made.

When is the sales charge deducted from purchases of mutual fund shares made under a letter of intent?

Within 5 business days in a margin AND cash account

When must a Regulation T call be met?

At or before the time the account receives approval for trading

When must a new options customer receive the Options Clearing Corporation's current disclosure document?

within 15 days of account approval

When must the options agreement be signed

private label

When private institutions invest in CMOs they are called what

do not have the same potential tax consequences as mutual funds, such as making capital gains distributions annually

When speaking to a customer about exchange-traded funds (ETFs), a registered representative could CORRECTLY state that these funds

When is a person's investment considered to be accepted into an oil and gas direct participation program?

When the general partner signs the completed subscription agreement submitted by the investor. Signing the completed subscription agreement indicates that the general partner (GP) accepts the prospective investor as a limited partner. After signing the subscription agreement, the investor signs the partnership agreement that gives the GP power of attorney for the limited partner and officially confirms the investor as an LP. The certificate should already have been filed, and it is amended to include the new limited partner within 30 days of the person being accepted into the partnership.

A registered rep has approached an investor about a real estate direct participation program. The investor looks over the important information and the offering memorandum and decides to purchase $30,000 of the program. When is the investor officially subscribed to the real estate program? A. When the certificate of limited partnership is recorded with the state B. When the investor signs the subscription agreement for the program C. When the investor signs the limited partnership agreement D. When the general partners accept the limited partner in the program

When the investor signs the limited partnership agreement. Watch out for this question. The question is asking for when the investor is "officially subscribed" to the program. When the general partners sign the subscription agreement, they are accepting the limited partner into the program. Now the new LP will receive the partnership agreement, which must be signed by both the GP and LP, and upon signing the partnership agreement, the LP is "officially subscribed" into the program. The certificate is recorded after all of the investors are part of the program. It is actually filed first and then amended when the limited partners are added. The investor may sign the subscription agreement, which is when the LP is accepted. But it takes signing the partnership agreement to officially be accepted into the partnership by the GP. [Module 16, DPP, Section 1.4]

None of these

Which of the following individuals could open an account at another member firm without his employer's knowledge? I.A registered representative of a member. II.A government security trader employed by a member. III.A purchases and sales clerk of a member. IV.An insurance agent who sells variable annuities.

registered rep has approached an investor about a real estate direct participation program. The investor looks over the important information and the offering memorandum and decides to purchase $30,000 of the program. When is the investor officially subscribed to the real estate program? A. When the certificate of limited partnership is recorded with the state B. When the investor signs the subscription agreement for the program C. When the investor signs the limited partnership agreement D. When the general partners accept the limited partner in the program

When the investor signs the limited partnership agreement. Watch out for this question. The question is asking for when the investor is "officially subscribed" to the program. When the general partners sign the subscription agreement, they are accepting the limited partner into the program. Now the new LP will receive the partnership agreement, which must be signed by both the GP and LP, and upon signing the partnership agreement, the LP is "officially subscribed" into the program. The certificate is recorded after all of the investors are part of the program. It is actually filed first and then amended when the limited partners are added. The investor may sign the subscription agreement, which is when the LP is accepted. But it takes signing the partnership agreement to officially be accepted into the partnership by the GP. [Module 16, DPP, Section 1.4]

It is based on the net risks associated with all the securities in a portfolio and are lower than the margin requirement calculated conventionally

When using customer portfolio margining (CPM) to calculate margin, the requirements are I.Based on the risk associated with the security having the highest volatility in a portfolio. II.Based on the net risks associated with all the securities in a portfolio. III.Higher than the margin requirements calculated conventionally. IV.Lower than the margin requirements calculated conventionally.

At auction

Where are T-bills sold?

Regular Reports

Where would a firm's financial status need to be in regard to SEC requirements

CD with no call feature

Which of the following CDs, all maturing in ten years, would be most suitable for an investor wanting long-term consistent income without incurring much risk?

retail and institutional transactions and trades done by a broker's broker

Which of the following activities can occur in the municipal bond secondary market?

An increase in the reserve requirements.

Which of the following activities of the FRB would cause interest rates to increase?

Short 1 XYZ call and long 1 XYZ put.

Which of the following are on the same side of the market?

Money market mutual funds. Money market funds are part of M2, but not M1. M2 includes everything in M1, plus time deposits and money market funds.

Which of the following are part of M2, but NOT M1?

Labor, fuel, or drilling rig rental.

Which of the following best describes an intangible drilling cost?

The SRO has authority to censure, fine, or expel the registered representative.

Which of the following best describes the disciplinary actions a self regulatory organization (SRO) such as FINRA or an exchange can impose upon a registered representative who has violated its rules?

High inflation risk; low default risk.

Which of the following best describes the investment characteristics of a high-quality long-term municipal bond?

Section 8.

Which of the following bonds is issued to finance the construction of subsidized housing and is backed by rents and the taxing authority of the U.S. government?

Purchasing more bonds, each having smaller face values, Purchasing bonds with maturity dates spread over periods of time

Which of the following characteristics are associated with bond laddering?

Long stock and stock rights

Which of the following covers a short call? I.Long stock II.Short stock III.Long put IV.Stock rights

The difference between the total dollar amount received from the issuance of common stock and the stock's aggregate par value.

Which of the following describes additional paid-in capital?

National Public Finance Guarantee Corp. and AMBAC.

Which of the following insures general obligation bonds?

Discount rate. The discount rate is set by vote of the Federal Reserve Board. The remaining interest rates are directly or indirectly set by their markets.

Which of the following interest rates is NOT market driven?

Money market funds.

Which of the following investments is the most liquid?

An investor wishing to be able to get into and out of the investment easily

Which of the following investor profiles would a principal protected equity-linked exchange-traded note best be suited for?

Whether the piece will be distributed in written form or via electronic media

Which of the following is NOT a factor when a communication to be distributed to the public is either being reviewed or approved within the broker/dealer?

The call has a time value beyond an intrinsic value that gradually dissipates

Which of the following is NOT an advantage of buying listed call options as compared to buying the underlying stock?

Community property applies to property owned individually before a marriage once the marriage has occurred.

Which of the following is NOT true in jurisdictions that recognize the marital property designation known as community property?

It is based in London.

Which of the following is NOT true of the Interbank market?

Interest income is taxed at maturity.

Which of the following is NOT true regarding Treasury Receipts?

Private lending institutions approved by GNMA originate eligible loans and sell the mortgage-backed securities to investors.

Which of the following is TRUE regarding GNMA?

They tend to have lower premiums than standard contracts or long-term equity anticipation (LEAP) contracts

Which of the following is TRUE regarding option contracts that expire weekly?

A variable, semi annually adjusted inflation rate, based on changes in the CPI.

Which of the following is a characteristic associated with I bonds?

Maintenance requirement.

Which of the following is always affected by a change in the market value of securities in a long margin account?

Defeasement.

Which of the following is an effect of advance refunding on a municipal bond issue?

The principal

Which of the following is required to sign a new account form for a cash account?

Exclusive use of index funds.

Which of the following is the best example of a passive investment management style?

Total takedown

Which of the following is the largest component of a municipal underwriting spread?

they are registered as open ended companies. They have operating costs and expenses that are lower than mutual funds

Which of the following is true regarding exchange traded funds (ETFs)?

Insider Trading and Securities Fraud Enforcement Act of 1988

Which of the following mandates the creation of an information barrier to thwart the flow of sensitive or non public information between departments within a broker dealer?

Nasdaq Global Market, Nasdaq Capital Market.

Which of the following market tiers are included in Nasdaq?

shareholder's equity, total liabilities

Which of the following may be affected when a company declares a cash dividend?

Issuing new bonds with an equal lien on the project's revenues.

Which of the following may only be accomplished after applying the additional bonds test for a revenue bond?

Broker Call Loan Rate, Prime Rate.

Which of the following money rates are set by banks in competition with one another for borrowers?

Broker Call loan Rate, PRime rate

Which of the following money rates are set by banks in competition with one another for borrowers?

The discount rate

Which of the following most quickly reflects changes in FRB policy?

The discount rate.

Which of the following most quickly reflects changes in FRB policy?

A) School bonds. B) Limited tax bonds. C)All of these municipal issues. D)Industrial development bonds. All of them

Which of the following municipal securities could have been sold in a negotiated underwriting?

Short put.

Which of the following option strategies besides going long a call can be used to purchase stock below its current market value?

Limit order to buy

Which of the following orders is reduced on the order book on the ex-dividend date for a cash dividend?

Limit order to buy.

Which of the following orders is reduced on the order book on the ex-dividend date for a cash dividend?

A buy stop limit and A buy limit

Which of the following orders may be used to acquire a security at a specific price or better?

buy stops and sell limits

Which of the following orders would be executed in a rising market?

Syndicator

Which of the following registers the securities and packages the program for an LP

Negotiable certificate of deposit

Which of the following securities trades with accrued interest?

Treasury securities.

Which of the following securities underlies a yield-based option?

Treasury securities. Yield-based interest rate options are based on the yields of Treasury bills, notes, and bonds.

Which of the following securities underlies a yield-based option?

Custodian

Who signs a stock or bond power to effect good delivery of securities sold from an account under the uniform transfer minors act

An official or officials of the issuer

Who signs an official statement for a new municipal issue?

They are used primarily to finance imports and exports.

Which of the following statements best describes banker's acceptances?

Money is raised without a specific property being stated, and the GP selects the investments.

Which of the following statements describes an oil and gas blind pool offering?

They generally have higher yields than direct US Obligations and FNMA is a publicly trade company

Which of the following statements regarding U.S. government agency obligations are TRUE?

Stock sold through a 144 sale is considered registered stock after the sale, after holding the stock for 6 months, nonaffiliated may sell unrestricted

Which of the following statements regarding a Rule 144 sale of restricted stock are TRUE?

Securities in a discretionary account may not be rehypothecated.

Which of the following statements regarding a discretionary account is NOT true?

Interest payments are tied to the movements of specified interest rate, the coupon rate of the bond changes and the price stable

Which of the following statements regarding a municipal variable rate demand obligation are TRUE?

The issuing bank guarantees them, they are traded in the secondary market

Which of the following statements regarding negotiable CDs are TRUE?

the issuing bank guarantees them, they can be traded in the secondary market

Which of the following statements regarding negotiable CDs are TRUE?

an investor who expects no change in a stock price and sells a straddle, an investor who expects substantial's fluctuations in a stocks price and is unsure as to direction buys a straddle

Which of the following statements regarding straddles are TRUE?

1 to 2 % and is usually returned to the syndicate

Which of the following statements regarding the good faith deposit submitted by interested bidders are TRUE?I. It is usually 1% to 2% of the total par value of the bonds offered. II. It is usually 10% of the total par value of the bonds offered. III. If the bid is unsuccessful, it is returned to the underwriting syndicate. IV. If the bid is unsuccessful, it is retained by the issuer.

It is composed of listed securities traded OTC

Which of the following statements regarding the third market is TRUE?

II and IV

Which of the following statements regarding transactions in the different securities markets are TRUE?I. Transactions in listed securities occur mainly in the OTC market. II. Transactions in unlisted securities occur in the OTC market. III. Transactions in listed securities that occur in the OTC market are said to take place in the fourth market. IV. Transactions in listed securities that occur directly between institutions without the use of broker/dealers as intermediaries are said to take place in the fourth market.

A bond confirmation will show YTC if the bond has been called under an in-whole call provision

Which of the following statements regarding yield shown on a bond confirmation for a bond that has been called is TRUE?

Long straddle

Which of the following strategies is intended to be profitable with either a significant upside or significant downside move in the underlying stock?

Redeemable.

Which of the following terms or phrases does NOT apply to REITs?

Engaging in open market operations.

Which of the following tools is most often used by the Federal Reserve Board to control the money supply?

buy a put or sell short

Which of the following transactions in the same security will affect the holding period of a security held for 12 months or less?

Sell stop.

Which of the following would accelerate a decline in a bear market?

A bull vertical spread.

Which of the following would best describe, "Bought 1 Jan 55 call at 3 and sold 1 Jan 60 call at 1"?

the premium the investor must pay for the contract.

Which of the following would not be a concern for an investor writing a naked option?

Investors demand higher interest when lending their money for longer periods.

Which of the statements below best describes why a normal yield curve is positively sloped?

The registered representative may not effect transactions excessive in size or frequency in view of the customer's resource.

Which of the statements regarding a member firm's handling of a discretionary account is TRUE?

The registered representative may not effect transactions excessive in size or frequency in view of the customer's resources.

Which of the statements regarding a member firm's handling of a discretionary account is TRUE?

Unsecured corporate debt

Which one of the following best describes a debenture?

Congress and the President.

Which organization or governmental unit sets fiscal policy?

Wrap account.

Which types of accounts are billed a single fee annually for a group of services?

number of annuity units.

With regard to a variable annuity, all of the following may vary EXCEPT:

current assets - current liabilities.

Working capital is:

When a corporation purchases equipment for cash, what is affected on the company's balance sheet?

Working capital. Purchasing equipment takes dollars from the current assets to the fixed assets. This reduces the left side of the balance sheet, but only on the top. Remember, the top is working capital. When you affect one side or the other (not both), you affect working capital. [Module 18, Scenario-Based & Financial Statement Questions, Section 1.1]

Microsoft, Inc. has announced the declaration of a $0.12 dividend payable to stockholders of record on Monday, December 1. An owner of Microsoft who sells the stock regular way on Friday, November 28:

Would be entitled to receive the dividend. In this scenario, the stock was sold when it was selling ex-dividend (without the dividend). This entitles the present owner, the seller of the stock, to receive the dividend. The ex-dividend date is two business days before the record date -- in this case on November 27 (remember, there is a weekend here). The stock then trades ex-dividend for two more days -- November 28, the day after the ex-dividend date, and December 1, the record date. Since the seller of the stock is still the owner of the stock on the record date, the seller is entitled to the dividend. Only buyers receive due bills; sellers never receive due bills

A portfolio manager of a pension fund desires income but wants to protect the portfolio. Which of the following options positions would he make?

Write calls against the market

An investor notices that the market is going down and is experiencing a big sell-off. The investor is expecting a period of consolidation before the market starts to rise again. What would the investor do to take advantage of the situation?

Write covered calls. When investors think the market is stabilizing, they want to write options. In this situation, the option writers would probably be able to keep the premiums. [Module 9, Options, Section 7.1; Module 18, Scenario-Based & Financial Statement Questions, Section 3.1]

A municipal securities broker/dealer who underwrites new municipal issues has been asked to make a contribution to a political candidate in the municipal area of the firm. If the municipal securities dealer makes the contribution, which of the following statements is true? A. No, contributions are not permitted. B. Yes, contributions of $100 are permitted. C. Yes, contributions of $300 are permitted. D. Yes, contributions are permitted regardless of the amount.

Yes, contributions of $100 are permitted. In actuality, contributions up to $250 are allowed, provided the person making the contribution is eligible to vote for the political candidate. [Module 15, Federal & State Regulations, Section 10.2]

A self-employed individual is 45 years old and has a Keogh retirement plan. He has contributed $150,000 to the plan, but the plan is now worth $200,000. He plans on taking a partial withdrawal. Can he make this withdrawal from the Keogh plan?

Yes. The full amount of the withdrawal will be taxed as ordinary income plus a 10% penalty. The contributions to the Keogh were tax-deferred; therefore, funds are taxed as ordinary income when they are withdrawn. Because the individual is younger than 59 1/2, he will have to pay 10% of the withdrawn amount as a tax penalty.

tranche's expected life

Yield quotes on CMOs are based on the

One of your customers calls and tells you he wants to invest in some shares of a new company. He wants a company that is doing well and is in a bullish movement. Under which of the following circumstances can you recommend an IPO of your firm's shares to the customer?

You can recommend your firm's shares if you find that the customer's objectives and suitability are met by the purchase of the shares of your firm. The securities must be suitable for the customer. In addition, prior to or at the same time the transaction is completed, written disclosure of the relationship of the firm must be given.

day order.

You receive a not-held order from a customer who wants you to buy 1,000 shares of ABC when the price is right. Under NYSE rules, this order is a:

Two brothers have a tenants in common account with your firm. One of the brothers places a trade to sell $5,000 worth of stock. He asks you to send him the check in his name only. Which of the following do you tell him? A. You will be glad to send the check on the settlement date. B. You will be glad to send the check on the settlement date, but it will be in both names. C. You can't send the check, since it takes both tenants to request the check. D. You can't do the trade until the other brother agrees to it.

You will be glad to send the check on the settlement date, but it will be in both names. This is a tenant in common account and both names must be on the check when it is issued. Either party can make the trade, but the check is always made payable to both parties. [Module 11, Customer Accounts, Section 4.2]

XYZ Corp. has contacted Bill Banks, a registered representative with ARP Securities. XYZ would like to begin offering a retirement plan benefit to its employees. In trying to decide whether to recommend a defined benefit or defined contribution plan to the company, the registered representative inquires about the age demographics of XYZ employees. Upon receiving this information, the registered representative recommends a defined contribution plan. From this recommendation, you could deduce that a majority of the employees of this corporation are:

Younger employees. From this recommendation of a defined contribution plan, you can deduce that a majority of the employees of this corporation are younger employees. A defined contribution plan tends to favor younger employees since they have a greater number of years to contribute to the plan prior to retirement. A defined benefit plan favors older employees over younger employees, since the limit on the amount that can be contributed per employee is higher for defined benefit plans than for defined contribution plans

2800.

Your 65-year-old client owns a nonqualified variable annuity. He originally invested $29,000 4 years ago; it now has a value of $39,000. If your client, who is in the 28% tax bracket, makes a lump-sum withdrawal of $15,000, what tax liability results from the withdrawal?

The broker/dealers clearing agent.

Your broker/dealer is not self-clearing but instead is an introducing broker/dealer. Therefore all extension requests made to your broker/dealers SRO on behalf of customers would be made by:

soft dollar arrangement

Your broker/dealer provides occasional research reports to an institutional trading desk in exchange for that institution doing executions for its various fund portfolios through your broker. This is known as

cannot enter a discretionary order.

Your client informs you that a signed discretionary account form is in the mail. Before receiving the form and unable to contact the client, you notice that one of his stocks is dropping sharply on adverse news. You:

$950. All of the dividends received must be reported. Reinvesting any or all of the money in TXR stock does not reduce the client's tax liability on dividends received.

Your client owns stock in the TXR Corporation and has received dividends of $950 this year. The client has taken $450 of this and used it to purchase additional shares of TXR. For tax purposes, your client must report:

the value of the assets the company holds, both tangible and monetary, in relation to its debt obligations

Your customer asks you to help evaluate several companies she is considering adding to her portfolio. One of the tools you are using is the asset coverage ratio to assess

Sovereign debt securities

Your customer has listed income and safety of principal as his primary investment objectives. Which of the following might be the least suitable recommendation?

Selling calls against restricted (Rule 144) shares is prohibited because the restricted shares could not be delivered if the calls were exercised by the buyer.

Your customer informs you that he has shares of stock restricted under Rule 144. He suggests that he wants to sell covered calls against the shares he owns to bring additional income into his account. Which of the following should you advise?

with guaranteed minimum withdrawal benefits (GMWBs) a lifetime of periodic payments is guaranteed

Your customer is interested in a variable annuity but is unclear on some of the details regarding different specifications and riders that can be attached to the contract. He makes the following four statements, all of which are true EXCEPT

Over-allotment (Green Shoe Provision)

Your customer is interested in purchasing shares of a new issue where the demand for shares has already exceeded the number of shares the issuer intends to offer. Which of the following options might you look for that could allow your customer to receive shares?

The MSRB's RTRS (Real Time Transaction Reporting System).

Your customer is interested in up-to-the-minute price information and transparency of municipal securities transactions. This information is available through third-party data vendors with pricing information captured by:

One pound equals $1.4847.

Your customer tells you that she sees the exchange rate for the British pound in the spot market is listed at 148.47. What do you tell her when she asks you what this means?

indicate an interest in the offering.

Your customer wishes to purchase shares of an IPO. During the cooling- off period, the customer can:

A young man is in a low tax bracket. He has $10,000 to invest and wishes to make the best investment for his young children's college education. As his registered representative, you advise him to invest in which of the following? A. Zero-coupon bonds B. T-bills C. Certificates of deposit D. Tax-exempt bonds

Zero-coupon bonds. Since the investment is for young children, they have a long time to wait. The zero-coupon bonds will be long-term and they are safe. Certificates of deposit are with banks and do not have as much interest. Children do not need tax-exempt issues

Prime account

a customer contracts with one broker, the prime broker, to provide a list of support services, such as clearing and settlement of transactions, while contracting with numerous other brokers for executions services.

Buy Stop Order

a customer with an unrealized gain on a short stock position wishes to protect his profit he should take what action

STRIPS

a debt instrument that pays no periodic interest

Over-allotment option for a new issue

allows underwriters to sell up to 15% more than the original number of shares offered and is a way for underwriters to address demand exceeding the number of shares originally offered

A Municipal Finance Professional (MFP)

an associate of a broker dealer engaged in municipal securities representative activities, other than retail sales. -an employee of a broker/dealer engaged in municipal security representative activities other than retail sales or who solicits municipal securities business for the broker/dealer

decrease them and create a multiplyer effect

an increase in the FEDS reserve requirement would do what to bank deposit

All of the following statements are true regarding both PAC mortgage obligations and TAC mortgage obligations, except: A. As interest rates fall, the companion tranches have an incidence of high prepayments. B. The main tranches of both the PACs and the TACs have prepayment protection. C. PAC and TAC mortgage obligations can be purchased in $1,000 increments. D. Both PAC and TAC mortgage obligations have protection against late payments.

answer (false statement): Both PAC and TAC mortgage obligations have protection against late payments. Only PAC mortgages have late payment protection. Both PACs and TACs have early payment protection

A client, Mr. Armstrong, is interested in investing in Nissan Corporation. However, Nissan stock only trades on the Tokyo stock exchange. Mr. Armstrong would like to understand how to invest in Nissan using ADRs. With this in mind, which of the following statements is not true about ADRs? A. ADR investors receive dividends in U.S. dollars. B. ADR investors do not have voting rights, as do stockholders. C. ADR investors do not have preemptive rights, as do stockholders. D. The dividends for ADRs are declared in U.S. dollars.

answer (false statement): The dividends for ADRs are declared in U.S. dollars. Actually, dividends are declared in the currency of the country where the stock trades -- in this case, the Japanese yen. However, because the ADRs trade in the U.S., investors receive any dividends in U.S. dollars. They also have all of the other rights of a regular stockholder with two exceptions: ADR investors do not have voting rights or preemptive rights, as do stockholders.

The Fed uses open market operations for all the following, except: A. To contribute to economic growth B. To influence the money supply C. To make up Treasury deficits D. To control interest rates

answer (false statement): To make up Treasury deficits. The Fed cannot buy and sell Treasury securities and borrow and lend money to change the debt. To change the debt, they have to pay off the debt.

full-power of attorney

anyone other than the account owner, entering trades and withdrawing assets requires this

the requirements for using the customer portfolio margining

are based on the net risks associated with all the securities in a portfolio and have lower than the margin requirement calculated conventionally

Lagging Indicators

are factors that change after the economy has begun a new trend but serve as confirmation of the new trend. Help to differentiate long trends form short ones

Stop Limit Order

are placed below the current market and will be elected when the stock trades or through (lower than) the stop price. Once elected, the order becomes a limited or to sell at the strike price or better

Ginnie Maes

are quoted in 1/32nds and are traded with an accrued interest computed on a 30/360 basis

Leading Indicators

are spot checks of business activity that reliably predict trends in the economy. Positive changes in these indicators predict economic improvement. Negative predict economic contraction

Good-Till-Canceled (GTC)

automatically canceled last day of April and last business day of October

4th narket

block trades are made in which market

Private Label CMOS

can use government issued securities as collateral and carry greater risk than agency issued CMOS

529 plan

contributions are considered gift under law and earnings are generated tax deferred

Lagging Indicators

corporate profits, average duration of unemployment, labor cost per unit of output. ration inventory sales, commercial and industrial loans, ration of consumer credit to personal income

improve a return on a portfolio

covered calls are used to what

Not Held (NH)

customer agrees not to hold the floor broker to a particular time and price of execution

front-end sales load

difference between the public offering price and the net asset value of a mutual fund share

4th market

direct trade between insittuions are made in which market

NASDAQ Level 1

displays the inside market; highest bid and lowest ask

mutual fund shares- undivided interest

each investor owns a proportional part of every security in the portfolio

Which of the following statements is not true regarding CMOs? A. Holders of CMOs get monthly payments. B. Investors of CMOs can buy them in increments of $1,000. C. The credit rating of the CMO is the same as the credit rating of the underlying pass-throughs that are being held in trust. D. A buyer of a CMO is subject to greater risk of prepayment than a holder of the underlying pass-through securities.

false statement): A buyer of a CMO is subject to greater risk of prepayment than a holder of the underlying pass-through securities. This is not true -- the CMOs would have less risk of prepayment because they have companion tranches that will take the prepayment risk instead of the actual tranche or pass-through. Some CMOs pay on a basis other than monthly, but most of them pay monthly. The fact that investors can buy CMOs in $1,000 increments makes these more available to lower-income investors. [Module 3, Government Securities, Sections 8.1, 9.1 & 9.2]

An exchange-traded fund (ETF) is an investment company that invests in all the following, except: A. Shares of companies that mimic a major market index B. Shares of companies that are only in one country C. Shares of companies that have a certain market capitalization D. Shares of companies that are all on the stock exchanges and periodically change the investments

false statement): Shares of companies that are all on the stock exchanges and periodically change the investments. ETFs do not change their shares, unless there is a change in the index they are mimicking, or some other event would cause the change. Investment companies that buy and sell shares of companies are managed investment companies. ETFs are not managed.

A registered rep sends out numerous informational pieces to her customers. If the information is considered advertising, it must be approved by her manager. All of the following are considered advertising, and thus subject to approval by her manager, except: A. A summary of the official statement of a new municipal offering B. A market write-up about the firm that is found on the firm's Web site C. A preliminary prospectus of a new corporate offering D. A research paper written by the firm's research department regarding a stock she is showing her customers

false statement): A preliminary prospectus of a new corporate offering. The preliminary prospectus and even the prospectus are not considered advertisements, nor is the official statement. However, a summary of the prospectus or OS is an advertisement and subject to approval. The write-up, although found on the Web site, is still advertising. Of those listed here, only the actual prospectus or official statement is not advertising

All of the following statements are true regarding an investment in a unit investment trust, except: A. The investor who purchases units of a unit investment trust has the ability to redeem the units to the trust, which may sell these units to other investors. B. A unit trust is an open-end investment company under the 1940 Investment Company Act. C. Unit investment trusts make a one-time offering of units that are invested in specific stocks and/or bonds that are kept for the life of the trust. D. A unit investment trust does not have a management team nor does it have a board of directors.

false statement): A unit trust is an open-end investment company under the 1940 Investment Company Act. A UTI is a trust, not a management company. It is neither open-end nor closed-end, but it is registered as an investment company under the 1940 Act. It does make a one-time offering of units, not shares, and makes a provision for the holder to redeem these to the trust. It can also trade on an exchange or in the OTC market. Today, many are known as ETFs. Instead of a management team, the trustees of the trust decide on some specific investments and then hold them until a specific time, and if they are debt securities, until they mature or are called.

All of the following are exempt from the registration provisions of the Securities Act of 1933, except: A. U.S. government and municipal securities B. Private placements C. American deposit receipts D. The sale of securities incorporated in a state that is being sold only to residents within the state

false statement): American deposit receipts, or ADRs ADRs are not exempt from registration ADRs have to be registered because they represent shares of stock of a foreign company, and when selling in the U.S. markets, the receipts have to be registered to be issued. Private placements (Rule 506) and intrastate offerings (Rule 147) are exempt from full registration; these only require that a short form registration statement be sent to the SEC. U.S. government and municipal securities do not have to be registered with the state

Regulation NMS was enacted to make trading of certain securities more transparent. All the following are subject to FINRA and Reg NMS rules, except: A. Stock of listed companies B. OTC stock C. Listed options D. Corporate bonds of listed companies

false statement): Corporate bonds of listed companies. Convertible corporate bonds are subject to Reg NMS, but not bonds that are not convertible. Since the question does not state that the bonds are convertible, do not assume they are or are not. Therefore, this is the best answer. Even listed options and OTC stock are subject to Reg NMS and FINRA rules.

The board of directors of a company determines all of the following dates when distributing a cash dividend to stockholders, except: A. Payment date B. Record date C. Ex-dividend date D. Declaration date

false statement): Ex-dividend date. The board of directors of a company does not determine the ex-dividend date (ex-date). Depending on where the stock trades, the exchange or FINRA determines the ex-dividend date. If an investor purchases stock on or after the ex-dividend date, the investor is not entitled to the dividend that is granted. The investor will not be the owner of record before the settlement date. The ex-dividend date is two business days before the record date, and regular way delivery is three business days after the trade date. The board of directors determines the payment date, the record date, and the declaration date.

Your customer is a 52-year-old conservative investor in a moderately high tax bracket. All of the following would be a suitable investment, except: A. U.S. government bonds B. Municipal bonds C. High-yield corporate bonds D. Money market investments

false statement): High-yield corporate bonds. High-yield corporate bonds involve risk that the corporation may not pay the interest and principal of the bonds. Since the customer is in a relatively high tax bracket, municipal bonds may be suitable, since these bonds pay interest that is exempt from federal taxes. The government bonds and the money market investments could be considered relatively safe. Now don't mix-up money market instruments with money market mutual funds. Money market mutual funds invest in money market instruments, as do many individuals, but money market mutual funds are used only for a short period of time for holding money. ****Money market instruments, such as T-bills, commercial paper, and banker's acceptances are excellent short-term investments.****

All of the following are standard maturity periods for a U.S. Treasury bill, except: A. Three months B. Six months C. Nine months D. 12 months

false statement): Nine months. Treasury bills are sold in one-month, three-month, six-month, and 12-month maturities, which are really 4-week, 13-week, 26-week, and 52-week periods. Just remember that there are no nine-month T-bills.

The Securities Exchange Act of 1934 does all of the following, except: A. Prohibits manipulation B. Regulates securities exchanges C. Requires the disclosure of information pertaining to listed securities D. Requires full disclosure to prevent fraud in the issuance of new securities

false statement): Requires full disclosure to prevent fraud in the issuance of new securities. This is not covered under the Securities Exchange Act of 1934 -- it is covered under the Securities Act of 1933. All the others are covered under the Securities Exchange Act of 1934. A good way to remember this is if the question is dealing with new issues, it is generally referring to the Securities Act of 1933; if the question deals with secondary issues, it is generally referring to the Securities Exchange Act of 1934. The answer, "Requires for the disclosure of information pertaining to listed securities" is a bit tricky, but since it says "listed securities," it is referring to stock trading on an exchange.

Position- Trading Firm

firm trading form its own account

more than 60 days

for dividends to be taxed as qualified how long must they be held for

ADRS

foreign exchange in a US market. Falls under the oversight of the SEC, dividends are in US currency and all transaction

Conduit Theory

funds are not taxed on earnings, if it distributes provided distributions equal to 90% or more of net investment income and Dividends and Interest is passed through to the investor without being taxed

The City of San Rafael has issued a limited tax bond. Which of the following types of bonds is this? A. A revenue bond B. A general obligation bond C. A moral obligation bond D. A special assessment bond

general obligation bond. A limited tax bond is a full faith and credit bond, which means it's a general obligation bond. It is backed by a special tax, group of taxes, or a limited tax on real estate. Because it is backed by the full faith and credit of the municipality, it is a general obligation bond. It has some of the attributes of a special assessment bond because it is backed by some taxes, but it is still an obligation of the whole municipality. Therefore it is classified as a GO bond. It is not a revenue bond or a special assessment bond.

Taxed. All contributions are pretaxed

how are qualified retirement plans distributed?

normal requests are up to three months but longer periods can be granted for acceptable reasons

how long can a broker hold mail?

25

how many days do you have to make an appeal

liabilities

if assets stay the same but owners equity declined, what increased?

neutral

if you write a call and a put on the same security what type of market attitude do you have

The issuer

in a new municipal offering who is responsible for hiring the bond counsel

the inside market

in the interdealer market, the quote that is the basis for the markup or markdown of a transaction

progressive

income taxes and estate taxes are what kind of tax

borrowed money

leveraged by-out

Shorting Against the Box

is a strategy that is used to lock in a capital gain that will be deferred into a later tax period.

Thomson's Muni News or Muni Market Monitor

is an automated system of delivering information relating to the market of municipal securities

systematic risk

is the possibility that an overall decline in the market will cause a loss in investment.

Dark pools of liquidity

less the transparency of the overall market as volume, quote and price information, and the market participant is unknown

dark pools of liquidity

lessen the transparency of the overall market as volume, quote and price information, and market participant identity is unknown

Third Friday of the month of Expiration

listed stocks options cease to trade when

The best time for an investor seeking returns to purchase long-term, fixed-interest-rate bonds is when:

long-term interest rates are high and beginning to decline

money market

market place for short term debt

Penny Stock

market price is less than 5 dollars a share and is a non-nasdaq otc stock

capital market

marketplace for long term debt

Leading Indicators

money supply, building permits (housing starts), Average weekly initial claims for unemployment compensation. average work week of manufacturing, new orders for customer goods, machine tool orders, changes in inventories of durable goods, changes in sensitive materials prices, stock prices, changes in business and customer borrowing

Fill-Or-Kill (FOK)

must be made in its entirety and in one attempt

All or None (AON)

must be made in their entirety or not at all. do not have to be made immediately

Options sales literature

must be preceded or accompanied by an OCC disclosure booklet and can include recommendations

Coincident Indicators

number of hours worked, employment levels, nonagricultural employment, personal income, industrial production, GDP, manufacturing and trade sales

Immediate or Cancel (IOC)

orders must be made right away but can be made partially

GDP

personal consumption, private investment, foreign investment, net exports and all government purchases

yield-to-call

prerefuned bonds should be listed how

retained earnings

represent income that has not been paid out to its shareholders

TRACE Trade Reporting and Compliance Engine

requires both sides of a transaction to report a trade. is the reporting system for corporate bonds trading in the otc market

certain price or more

sell limit

either long or short

sell order tickets must be marked as

Break point sale

selling mutual fund shares immediately below the dollar amount stated in the prospectus that world qualify an investor to receive a reduced sales charge

PSA

shows historical data projections regarding mortgage payments

the option expires, or the option is sold

taxable gain or loss on a long call option transaction would be recognized when

.01 or more for the member firm and for the customer

the OCC automatically exercises an open equity option contract if, at expiration, the contract is in the money by:

disintermediation

the flow of money from traditional, low-yielding savings accounts to higher- yielding money market instruments

industrial stocks

the largest component of the S&P 500 index is made up of

7am -8 pm

the nadaq is open form when

Higher prices and lower yields

the result of declining inflation on outstanding bonds

Aaa

the tranche of a CMO obligation that is secured by a government agency pass through certificate is rated by moodys as

limited power of attorney

this enables a nonaccount owner to enter trades but not make withdraws

Brokered CD

trade at a market driven price that typically moves inversely to the movement of interest rates

close ended investment companies

traded in the secondary markets, including the exchanges and otc markets it is also traded by institutional investors

stock repurchased by the issuer

treasury stock

they are taxed as ordinary income

what happens to partial distributed annuities

firewall or barrier

used to prevent the passing of potentially sensitive material and sensitive nonpublic information between a firm's departments

theft

what do fidelity bonds protect against?

100k

what is the minimum face value of cd

8.5%

what is the what is the maximum sales charge of POP

the cost basis

what price you bought the stock at

1.50

what rate is associated with ARS

A wash sale

when an investor swaps identical issues of stock to establish a loss that is disallowed, the transaction is known as:

quarterly ; annually

when are 12b-1 shares charged and approved

when all are accredited investors

when may private placements be advertised.

owner of a put writer of a call

when will investors sell stock if an option is exercised

Coincident Indicators

where the economy is; are those measurable factors that vary directly with the business cycle

principal or agency trade

which of the following items appears on the confirmation statement for a when-issued trade of a municipal bond

FRB

which of the following organizations determines which OTC securities are eligible for purchase on market

cumulative

which type of voting benfits the small investor

only member firms, not the public or non-members

who can buy people POP

IRS and 1933 at

who governs the sale of a publicly offered direct DPP

FINRA, Securities Act of 1933

who governs the sale of sale of a publicly offered direct participation program

a self-regulatory organization

who grants regulation t extensions- self regulatory organizations

NYSE

who sets the ex-date for a close end fund listed for trading on the NYSE

An investor owns 1,000 shares of stock. The corporation declares a 3% stock dividend and an increase in its quarterly dividend of $0.15 to $0.17 per share. If the record date is June 10 and the payable date is June 30 for both the stock dividend and the cash dividend, the investor will receive:

$0.17 per share and 30 shares. .17 per share is the amount paid to current shareholders, plus the 3% stock dividend. The stock dividend is shares in the amount of 3% for each share owned -- 3% x 1,000 shares = 30 shares.

An investor buys 1,000 shares of stock on Tuesday, April 18. The corporation declares a 3% stock dividend payable to holders of record on April 20 and an increase in the quarterly dividend of $0.15 to $0.17 per share to holders of record on April 24. The payable date is May 15 for both. The investor receives:

$0.17 per share and no additional shares. The .15 has nothing to do with the amount of the cash dividend because that is the old dividend. The dividend just declared is .17 per share to holders of record on April 24, and the 3% dividend in the form of additional shares of stock to holders of record on April 20. To determine whether the investor will receive the stock dividend, make a calendar to diagram and identify the record date and the ex-dividend date. The trade date is Tuesday, April 18, and the ex-dividend date is April 18. The investor purchases the stock on the day the stock trades ex-dividend. Because the settlement date is after the record date for the stock, the buyer does not receive the 3% stock dividend. The investor will receive the cash dividend because the investor will own the stock on the record date for the cash dividend. Here's how it looks on a calendar: MON TUE WED THUR FRI 17 18t 19 20rs 21s t = trade day s = settlement day 24rc 25 26 27 28 rs = record stock dividend rc = record cash

An investor buys 1,000 shares of stock on Thursday, August 4. The corporation declares a 10% stock dividend to holders of record on August 10, and a quarterly dividend of $0.24 per share to holders of record on August 16. If the payable date is August 31 for both the stock dividend and cash dividend, the investor receives:

$0.24 per share and 100 shares. The cash dividend of .24 per share is paid to holders of record on August 16. The stock dividend of 10%, in the form of additional shares of stock, is paid to holders of record on August 10. Since the investor purchased the stock before the ex-dividend date, the investor receives both the cash dividend and the stock dividend. Here's how it looks on a calendar: MON TUE WED THUR FRI 1 2 3 4t 5 t = trade day s = settlement day 8 9s 10rs 11 12 rs = record stock dividend 15rc 16 17 18 19 rc = record cash Since the settlement date is before both record dates, the investor owned the security in time to get both dividends.

An investor has the following account: SMV $90,000 Cr $150,000 How much SMA is in the account?

$15,000. To determine SMA, you first must find the equity in the account. This is a short account, so subtract the SMV from the credit balance ($150,000 - $90,000) to get $60,000. Next, you need Reg T of the market value ($90,000 × 50%), which equals $45,000. Now, SMA is the difference between the equity and the market value: $60,000 - $45,000 = $15,000

A customer buys 100 LEAPS calls at 8 with 15 months to expiration. What is the loan value of the option?

$20,000. LEAPS of nine months can be borrowed against. The option must have a maturity in excess of nine months. If the option has a maturity in excess of nine months, the option has a loan value of 25%. To calculate this equation: 100 contracts x 100 shares x 8 premium = $80,000 x 25% = $20,000.

A corporation has a balance sheet that has the following items listed in the total liabilities and the net worth: Current liabilities = $22,000,000 Funded debt, 8% convertible bond = $80,000,000 4% preferred stock (par $100) = $15,000,000 Common stock (par $1) = $45,000,000 Paid-in surplus = $60,000,000 Retained earnings = $32,000,000 What is the capitalization of the company?

$200,000,000. Remember -- do not include current liabilities in a company's capitalization as they have to be paid immediately. The capitalization is made up of the money the company uses to stay in business -- all long-term debt, usually called funded debt, as well as all the net worth except the retained earnings. The retained earnings are used to expand the business and to keep the company going in the coming year and thus are not part of the capitalization. The paid-in surplus is the extra amount received over the par value when the stock, both preferred and common, was issued. That is part of the capitalization. Therefore, add the funded debt, preferred stock, common stock, and paid-in surplus: $80 million + $15 million + $45 million + $60 million = $200 million.

A syndicate member in a divided $8 million municipal offering agrees to sell $800,000 of the offering. Initially the firm only sells $600,000 of the bonds. After two weeks, $3 million of the offering still must be sold. The member has responsibility for selling what amount of the bonds?

$3 million. Selling responsibility is 100% until the offering closes down. It is the liability that is split up. In this case, $3 million is the selling responsibility at this point, but if it closed, only $200,000 would be the liability.

An investor has incurred margin expenses of $2,500 in a convertible bond account, $7,600 in a stock account, and $5,600 in a municipal bond account. According to IRS rules, the customer can deduct all of the expenses, except: A. $2,500 margin expense on the convertible bond B. $5,600 margin expense on the municipal bond C. $7,600 margin expense on the stock D. Only margin expenses for stocks and bonds can be deducted.

$5,600 margin expense on the municipal bond (This cannot be deducted. Because municipal bonds pay interest that is tax-free, the interest expense (or margin expense) that is incurred in purchasing municipal bonds cannot be deducted)

A customer has an account with the following balances: Long market value = $86,000 Debit balance = $35,000 SMA = $22,000 The customer tells you he is sending in a check to put into his account. The check arrives for $10,000. What is the new equity in the account?

$61,000. The account presently has $51,000 in equity ($86,000 - $35,000), and when the funds arrive, the whole amount is applied to the debit balance. This raises the equity AND the SMA. The question could have asked what does it do to the SMA, which is the same answer -- it increases. For all deposits of cash, the debit balance decreases; therefore, the equity AND the SMA increase by the amount of the deposit. If you picked $41,000, you included the $10,000 in the debit account and did not credit the equity. Remember, the deposit of money reduces the debit, thus increasing the equity. The deposit does not increase the debit balance.

An older customer purchases 1,000 shares of Dow Chemical, Inc. stock for $26 per share. After the customer has held the stock for over five years, he dies. At the time of death, the stock is selling for $62 per share. By the time the customer's son inherits the stock, it has risen to $64. Later, the son sells the stock for $69 per share. What is the cost basis of the stock upon the sale?

$62,000 (The cost basis of stock received in an inheritance is the value on the date of the death certificate. )

A 50-year old man has been contributing to a Keogh. He needs money for a vacation and withdraws $20,000 from the Keogh. He is in the 28% tax bracket. What is his tax?

$7,600. The person is under 59 1/2, so add the 10% penalty. If he were over 59 1/2, the 10% penalty would not be incurred. However, in either case he owes ordinary income tax since it is tax-deferred retirement plan. All withdrawals are fully taxable at the investor's tax rate.

A municipality has a new offering of a $10 million general obligation bond, $30 million of existing debt, and responsibility for 50% of a $40 million county debt. If the population is 500,000, what is the net direct debt per capita?

$80. To find net direct debt, divide municipal debt by population. In this case, $40 million ($30 million old and $10 million new) divided by the population of 500,000. Net debt would include overlapping debt. Note that net debt excludes the word direct and therefore includes the overlapping. The calculation is $40,000,000 divided by $500,000 = $80.

An investor who purchases a TAC mortgage-backed security knows that all of the following are true statements about the TAC, except: A. As interest rates rise, the possibility for an early call decreases on the TAC. B. The TAC tranche has prepayment-risk protection as well as extension-risk protection. C. The TAC companion tranche has a greater possibility of an early call than the TAC main tranche. D. The TACs are backed by mortgage-backed securities and are generally rated AA or better.

(False statement)The TAC tranche has prepayment-risk protection as well as extension-risk protection. . The TAC only has prepayment-risk protection. The PAC has extension-risk protection as well as the prepayment-risk protection. Most TACs and PACs are backed by GNMA and FNMA securities, which are mortgage-backed securities with at least an AA rating. Therefore, the TACs assume the rating of these securities. For both TACs and PACs, the possibility of calls rises and falls in relation to interest rates. This is because the CMOs are backed by mortgages, and mortgages are refinanced as lending rates drop.

Which of the following does not need to be considered when an underwriter is analyzing a new municipal revenue bond that will be for the expansion of the airport for that municipality? A. The feasibility study on the airport B. The per capita debt for the parcels in the municipality C. The increase in tourism that the expansion will encourage D. The increase in population over the next 15 years

(false statement): The per capita debt for the parcels in the municipality. This is a revenue bond. The per capita debt is only for a general obligation bond, and therefore has no effect on analyzing the new issue revenue bond. The most important aspect for a revenue bond is the ability of the users to pay for it. The feasibility study determines the usage of the airport, and thus the need for expansion. This is the main factor. The increase in population will increase usage of the airport and the increase in tourism is important for the airport as well.

Two brothers would like to set up an account with you, but are unsure whether to open a JTWROS or joint tenants in common account. As their registered rep you tell them that all of the following are true of both types of accounts, except: A. The proceeds must be split evenly. B. Either party can make a trade in the account. C. Neither party is required to sign the new account form. D. All checks written with the proceeds of a transaction will have both names on it.

(false statement): The proceeds must be split evenly. Only a JTWROS account must be split evenly. A tenants in common account is split according to each person's participation. In both account types either party can make a trade. FINRA does not require the signature of the customer on the new account form; however, many firms do require the signatures on the new account form so that they have a sample of each customer's signature on file -- but for the test, no signature needed. All checks issued from both types of accounts have the names of all parties on the check, and all securities are registered accordingly, unless valid written authorization is obtained first. [Module 11, Customer Accounts, Section 4.2]

According to SEC Rule 15-c-3-3, how many days past settlement can an investor deliver securities?

10 business days

Your customer is a single man in his mid-20s. He has enough cash for two month's living expenses. He wants to do some investing with you for his retirement. Which of the following would be the best investment strategy for him? A. 100% stock and 0% bonds B. 50% stock and 50% bonds C. 10% stock and 90% bonds D. 0% stock and 100% bonds

100% stock and 0% bonds The customer is young and will likely generate income for 40 plus years, and his objective is to save for retirement

A customer has a large amount of shares of a listed stock he would like to sell in a limit order. Up to how many shares can he send using SuperDot?

100,000 shares. Actually, it is "up to 100,000" shares, or 99,999 shares. Notice the question states "up to" how many shares. If these are not choices, then pick the 30,099. If you have the choice between 30,099 and 100,000, take the 100,000 shares. SOES is only 500 shares.

A corporate bond with a coupon rate of 15% has a yield to maturity of 17%. What is the equivalent yield on a municipal bond for an investor in the 28% tax bracket?

12.24%. To find the municipal equivalent, multiply the yield on the corporate by 100% minus the tax bracket. In this case, 17% x (100 - 28) = .17 x .72 = 12.24%. Remember to always use the yield, not the coupon rate. Also, multiply by the compliment of the tax bracket, not the tax bracket itself.

An insider owning 20,000 shares of EGG Corp. unregistered stock has filed a Form 144 notice of offering. The weekly volume of trading in EGG on all exchanges is: Week ending March 11 ..................20,000 March 18 ..................17,000 March 25 ..................17,000 April 1 ..?................18,000 April 8 ......................19,000 EGG has 1.84 million shares of stock outstanding. On April 12, the insider would like to sell a portion of his unregistered stock. What is the maximum amount of stock he can sell under Rule 144?

18,400 shares To calculate this number, there are two steps. First take 1% of 1.84 million (the outstanding shares) = 18,400. Second, add up the volume of shares for March 18th through April 8th (71,000) and divide by 4 weeks = 17,750. Since 18,400 is the larger amount, it is the amount that can be sold.

An investor has a margin account with the following balance: Long market value = $140,000 Dr = $60,000 Based on the present market value, what is the minimum maintenance that must be in the account?

35,000. To find the minimum maintenance in a long account, multiply the market value times 25%. Or, divide the market value by 4 and get the same answer (25% is 1/4 of 100%). Either way, $140,000 x 25% (or divided by 4) = $35,000.

An investor buys a 9% bond that is priced at 104.8. What is the current yield on the bond?

8.63%. This is a government bond as determined by the quote of 104.8. The decimal tells you this is 104 and 8/32, or 104.250% = 1042.50 per bond ($1,000). The interest is 9%, or $90 per year. Current yield is yearly interest divided by the market price. In this case, 90 divided by 1042.50 = .0863, or 8.63%.

Which of the following monetary disputes does NOT have to be settled through arbitration? A. A dispute between two broker/dealer firms B. A dispute between a registered rep and a broker/dealer C. A dispute between registered reps of two different firms D. A dispute between a registered rep and a customer

A dispute between a registered representative and a customer. A dispute between broker/dealers, between registered representatives, and between a broker/dealer and a registered rep must be settled through arbitration. Unless the customer agrees to and/or institutes arbitration, the customer does not have to be subjected to arbitration

Joint accounts are set up so that more than one person can benefit from transactions in an account. All of the following can have a joint account, except: A. Two business partners B. Two corporations C. A minor and a parent D. Two unrelated people

A minor and a parent. A minor and a parent CANNOT have a joint account. The parent must be a custodian for the minor and the parent cannot join the minor in the account. A custodial account and a trust account cannot be joint accounts. Any two adults, such as business partners, siblings, spouses, parents and adult children, and even unrelated people, may have a joint account. Even corporations can have a joint account, although it is unlikely. Remember, corporations are a legal entity, and as such, can have a joint account in much the same way as two unrelated people can.

A large investor has a tender offer outstanding for ABC stock. All of the following can tender their stock to the person with the tender offer, except: A. A custodian tendering the stock in an UGMA B. A person who has a long account with the stock, but the account is restricted C. A person with an account where the stock will be sold short, and the account has an SMA D. A registered representative who has discretionary authority over a customer's discretionary account

A person with an account where the stock will be sold short, and the account has an SMA To tender stock in a tender offering, the person tendering the stock must own the stock -- the stock cannot be borrowed for the tender.

A husband and wife have a JT TIC account. Which of the following statements is true regarding trades made in the account? A. All trades in the account must be made by both parties. B. A wife may make a trade in the account without the husband's approval. C. Only the husband can make a trade in the account without the spouse's approval. D. Either party may enter a trade in the account, but both parties must approve the trade with the representative.

A wife may make a trade in the account without the husband's approval. A JT TIC account (joint tenants, tenants in common account) is like any joint account. Only one of the parties needs to make the trade. It does not take approval of the other person, nor does it take both parties telling the rep to do the trade. In addition, neither the husband nor the wife has to have the other's permission or approval. Some states may require the husband's approval of trades made by the wife, for a joint account or even her own account, but this is a state law, not a federal law. In this case, the exam tests only what is true federally

Which of the following is considered an advertisement by an issuer? A. An offering in the Blue List B. An offering in a tombstone C. An offering in a notice of sale D. An offering in an Official Statement

An offering in a notice of sale. The notice of sale is an advertisement from an issuer to underwriters asking them to bid on the issue. The Blue List is a list of secondary offerings by municipal dealers with prices and yields in the secondary market. The tombstone is the advertisement the underwriters publish enticing investors to purchase the newly issued securities. The Official Statement has all the important information about the issuer.

As the bond counsel for a city, the law firm Snoop and File has issued a legal opinion. By issuing the legal opinion, the bond counsel has done which of the following? A. Attested to the validity of the underwriting spread B. Stated in their opinion that the offering price of the issue is of fair value C. Given assurance of the legality of the issue D. Attested to the validity of the issue as being municipal securities

Attested to the validity of the issue as being municipal securities The whole point of the bond counsel is to determine the validity of a municipal bond and whether the bond issue falls under the rules in the tax code regarding the taxation of the interest payments. If it is a municipal bond, then the bond's interest will be exempt from federal income taxes, and in most cases, that state's income taxes.

You have an investor with a short position in APR stock at 56. He feels that the stock will be dropping 15 points, and wants to cover his position when the stock hits 42. Which of the following orders would you suggest to your customer to enter to take advantage once the stock hits 42? A. Buy limit B. Sell limit C. Buy stop D. Sell stop

Buy limit. Buy limit orders are used to purchase stock when it hits a certain low point that the investor wants to take a position in or to close a short position. In this case, the investor is short the stock and wants to take a gain at 42, so he enters an order to buy at 42 (limit).

A customer of yours in his mid-40s is diversified in stocks and bonds. He would like to take part in the new offerings that your firm has, but he has limited money to periodically invest, although he has a well diversified portfolio. Which of the following investment would be most beneficial to him? A. Buy shares of a sector mutual fund offered by your company B. Buy shares of a growth mutual fund offered by your company C. Buy shares of a venture capital mutual fund offered by your company D. Buy shares of a high-yield mutual fund offered by your company

Buy shares of a venture mutual fund offered by your company. This person is looking for new issues. Therefore, the venture capital mutual fund is best for best for him. , he is qualified to participate in a venture mutual fund since it provides the same effect as investing in individual new issues, with less risk.

An investor has a short position of 1,000 shares of APPL stock at 66.20. The stock declines to 62.10 and the customer wants to protect the profit. What type of order on the stock would you suggest to the investor?

Buy stop. The investor is short the stock, which means he has sold the stock. Any new sales are just increasing the investor's position. Since the question is asking to PROTECT the position, the buy stop is entered ABOVE the present market. That would be the best order. The buy limit could not be executed until the market dropped down to the limit price.

One of your new customers is 64 years old. He has $50,000 to invest and wants to know what would be a good investment for him. He has growth funds from which he is drawing income, and an IRA that will start payments in a year, but he states he wants some other investments. He wants income and is not worried about capital appreciation or when the payment of principal will occur. Which of the following would be a good investment for him? A. Growth and income mutual funds B. Closed-end investment companies that mimic an index C. CMOs D. Variable annuities

CMOs CMOs are all in real estate mortgages, and only give income. There is no diversification or appreciation.

A registered rep is soliciting many of his current customers. Which of the following solicitations can the registered rep do? A. Call a customer who is looking for capital appreciation and ask the person to purchase 1,000 shares of a $4 stock B. Call a customer who has just told you that he has $20,000 to invest, and suggest he sell short a $60 stock that the registered rep thinks is going to decline in value C. Call a customer and solicit a $25 stock with the guarantee that if the stock ever goes below $20 the rep will repurchase the stock at $25 D. Call a customer who is looking for a capital gain and suggest that he sell short 1,000 shares of an $80 stock that the registered rep thinks is about to decline in value

Call a customer who is looking for a capital gain and suggest that he sell short 1,000 shares of an $80 stock that the registered rep thinks is about to decline in value. The other short sale was for a person with limited funds, so it is not a suitable investment for that customer. There are no limited funds with this investor, and this person is looking for a capital gain. A rep can never guarantee a price to a customer. Selling a $4 stock to a person looking for capital appreciation is not suitable. The cheap stock is not an investment that normally makes a lot of money.

Which of the following is not considered part of a company's capitalization if issued by the company? A. Long-term bonds B. Preferred stock C. Corporate notes of less than one year D. Zero-coupon corporate bonds

Corporate notes of less than one year Capitalization is defined as the company's total amount of issued securities that have maturities GREATER than one year. This includes bonds, notes over a year, preferred stock, and common stock.

Which of the following does not appear in the notice of sale? A. Coupons B. Good faith deposits C. The various maturities of the bonds D. The range of interest rates

Coupons The underwriters in their bid determine the coupon for the maturity if it is a term bond, or for each of the maturities in a serial bond. The notice of sale is from the issuers to underwriters asking for bids on the issue. The underwriters determine the coupons within the range of interest rates given by the issuer. If the bond is a serial bond, that is denoted, as is a term bond. The amount of good faith deposit is also listed so the underwriters know how much to send.

PN Corporation, an airline, needs to purchase two new airplanes. The company uses extra cash it has to purchase the planes. This purchase will affect which of the following items found on the company's balance sheet? I. Current assets II. Working capital III. Total liabilities IV. Shareholder's equity V. Total assets

Current assets Working capital The purchase costs money, so the current assets are affected, as well as the working capital.

Which of the following best describes working capital? A. Total assets minus net worth B. Current assets minus current liabilities C. Current assets minus total liabilities D. Total assets minus total liabilities

Current assets minus current liabilities. This is the formula for working capital. The total assets minus total liabilities equals net worth, and the total assets minus the net worth is the total liabilities. Choice (d) is made up.

An investor has purchased a corporate bond at a discount. Which of the following is not needed to calculate the compound accreted value (CAV)? A. Dated date B. Amount of principal originally paid for the bond C. Current yield D. Maturity date

Current yield. You don't need the current yield to calculate the CAV. Don't get confused by the wording. You do need the purchase date, or if it's a new issue, the dated date, as well as the maturity date, to determine how many years to accrete the discount. The current yield has nothing to do with it, and in fact, if it is a zero-coupon bond, it has no interest and therefore no current yield

Your company is offering a new municipal bond and a customer of yours is investing in it. Which of the following best describes the accrued interest that will be paid by the purchaser? A. Dated date to trade date B. Dated date to settlement date C. Settlement date to interest date D. Trade date to settlement date

Dated date to settlement date. Even though this is a new issue, the purchaser always pays accrued interest. Often, the final settlement date is a month or two after the dated date, due to the sales of the bond itself. The dated date is the date that interest starts to accrue, and thus the issuer pays interest from that date until the first interest payment date, which could be six, seven, eight, or even 10 months later. Consequently, the bond purchaser has to pay the issuer the interest from the dated date until the date the new bonds settle with the syndicate, called the settlement date. The purchaser receives, on the next interest date, an amount of interest from the dated date to that first interest payment date.

A revenue bond has a gross revenues pledge outlined in the indenture of the bond. The revenues from the bond will go to which of the funds first?

Debt service fund. This question is asking about a gross revenues pledge. This is a pledge to pay debt service out of gross revenues -- in essence, the first thing to be paid

A husband and wife have a joint tenant with rights of survivorship account. The wife dies unexpectedly and the husband comes in to modify the account. The account is a margin account, so he decides to liquidate the stock that has a loan outstanding against them and pay off the debit balance. What further action should the registered rep take for the customer?

Do as directed by the customer. This is a joint account. The only time you need to wait for the death certificate or the executor is if the person is going to take the securities in his name only or the account is a single account, not a joint account. The attorney rarely has anything to do with it. Of course, until the death certificate comes in, all checks are made payable to both parties.

What is the maximum that an individual can contribute tax-free to a 529 plan for one beneficiary in any one year?

Five times the gift tax limit. The maximum that can be contributed to the 529 plan per beneficiary is five times the gift-tax limit (the amount of the yearly gift-tax limit changes periodically). Technically, there is no maximum amount, but only the above-mentioned limits would be tax-free. If only these choices are offered, you have to choose the five times the gift-tax limit. The $2,000 limit is for a Coverdell plan.

You have a client who has received $50,000 from a bond that has just matured. She wants to do a lump sum into one investment. She previously has invested in stock and lost money, so she is not comfortable investing in stocks. She is retired and wants a stable amount of income from the investment. What would you suggest to the client?

Fixed annuity. She does not want to be in stock, the utility stock is not an option. The high-yield corporate bond could be too risky in price fluctuations and she is concerned with income. A variable annuity would change values and change income, while a fixed annuity is a guarantee of a certain amount of income each month. Choose the fixed annuity.

A husband and wife are both 42 years old and employed with a combined income of $140,000. The wife is part of her employer's retirement plan and the husband is not. They wish to contribute to a traditional IRA. Which of the following is true of their contributions? A. Each can make a full tax-deductible contribution to their own IRA. B. Each can make a tax-deductible contribution to their own IRA. C. Each can make a nondeductible contribution to their own IRA. D. He can make a full tax-deductible contribution to his IRA, and she can make a nondeductible contribution to her IRA.

He can make a full tax-deductible contribution to his IRA, and she can make a nondeductible contribution to her IRA. Since they are over the $95,000 income limit and she has the retirement plan, she cannot make a fully tax-deductible contribution. However, because they are still under the $178,000 limit, his is still tax-deductible.

Charlie buys an RFG 110 call for 11 and sells an RFG 120 call for 5. The best description of this spread is which two of the following? I. Bullish II. Bearish III. Debit spread IV. Credit spread

I. Bullish III. Debit spread This is a debit spread because Charlie paid $6 per share more than he received. It is a bullish spread because the investor makes the most money if the market price of the stock is at 120 or higher.

A mark to the market occurs in connection with which two of the following? I. Changes in market value of securities under a contract II. A request for additional collateral made when securities have been loaned III. The issuance of due bills IV. The computation of accrued interest

I. Changes in market value of securities under a contract II. A request for additional collateral made when securities have been loaned A mark to the market only occurs when securities have been borrowed or loaned, and the market moves against the borrower. Basically, this only occurs in margin accounts. Marking to the market has nothing to do with due bills -- that is only for paying dividends for trades close to the record date. Computing the amount of accrued interest has nothing to do with determining the value of the account -- it is only for trades in bonds.

Dealer A provides Dealer B with a bid quote for a block of bonds that is firm for one hour. Which two of the following describe the situation of Dealer A? I. Dealer A cannot change the price for the bonds for one hour. II. Dealer A has bought the bonds. III. Dealer A has made a nominal bid. IV. Dealer A has agreed to buy the bonds at a particular price.

I. Dealer A cannot change the price for the bonds for one hour. IV. Dealer A has agreed to buy the bonds at a particular price. Dealer A cannot change the price for the bond for one hour, and Dealer A has agreed to buy the bonds at a particular price. Dealer A has made a bid to purchase the block of bonds, and the price offered for the block is good for one hour. Dealer A is locked into the price quoted, but Dealer B can sell at any higher price. Dealer A is committed to a specific price, but Dealer B does not have to sell.

Which of the following actions can FINRA take against a registered representative for violating FINRA rules? I. Impose fines II. Expel the person from FINRA III. Suspend the person for a period of time IV. Terminate the registered representative from the firm

I. Impose fines II. Expel the person from FINRA III. Suspend the person for a period of time

A registered representative who is updating a customer's file must update which of the following? I. Name and address II. Marital status III. Financial circumstances IV. Investment objectives

I. Name and address II. Marital status III. Financial circumstances IV. Investment objectives

An insurance company is looking to purchase a large quantity of a mutual fund. Where could this trade take place? I. Primary market II. Secondary market OTC III. Third market IV. Fourth market

I. Primary market IV. Fourth market Since all mutual funds are new issues, the primary market must be one of the answers. The secondary market OTC and the third market are both secondary trading, and thus are not possible answers. An insurance company buying directly from a mutual fund is the definition of the fourth market.

You have a client who has $20 million in securities with your firm. He wants to do some short-term investing, and asks you for advice. You suggest Jumbo CDs. He asks you about them, and you would tell him which two of the following? I. The CDs are negotiable and can be sold if the money is needed immediately. II. The CDs are not negotiable and must be held until maturity. III. The CDs are part of M2. IV. The CDs are part of M3.

I. The CDs are negotiable and can be sold if the money is needed immediately. IV. The CDs are part of M3 They make up the difference between M2 and M3. They are also negotiable, which means they can be sold in the secondary market. That is the difference between Jumbo CDs and CDs that people purchase from banks in small amounts of $1,000 up to $100,000. The Jumbo CDs start at $100,000, and are excellent for large investors wanting short-term investments.

A general partner specializing in real estate is putting together some real estate properties as the assets for a REIT. The cash to purchase the properties will come from investors in the REIT. Which two of the following are true regarding the REIT? I. The REIT must generate at least 75% of its income from the real estate properties. II. The shares of the REIT cannot be traded on an exchange, but must be purchased and redeemed through the REIT representatives. III. The interest received from the REIT is taxable as ordinary income. IV. The REIT is an investment company under the Investment Company Act of 1940.

I. The REIT must generate at least 75% of its income from the real estate properties. III. The interest received from the REIT is taxable as ordinary income. . To be a regulated investment company for tax purposes, a REIT must derive at least 75% of its income from the real estate properties, and then must pass 90% of its profits (net investment income) to investors. Remember, "regulated" means regulated under the Internal Revenue Code as a "regulated investment company," not the Investment Company Act of 1940. REITs are not investment companies under the Investment Company Act of 1940, but they are investment companies as defined by the IRC. Once the shares are issued, they can trade on an exchange, but most trade OTC.

A new customer to a FINRA member firm opens an account to purchase $50,000 of municipal bonds. Which two of the following are needed by the firm to open this municipal bond account and execute the order? I. The signature of the municipal principal II. The signature of the customer III. The approximate age of the customer IV. The tax bracket of the customer

I. The signature of the municipal principal III. The approximate age of the customer There is no requirement to have the customer sign a new account form for any securities. The tax bracket answer choice is not complete -- income, expenses, and the financial situation should be included with tax bracket, and collectively they are known as tax status.

Amber Corporation has issued 40 million shares of common stock and 2 million shares of preferred stock. What rights do the common stockholders have? I. Vote for the board of directors II. Vote on the election of officers III. Vote on the decision to give cash dividends IV. Vote on the takeover of another company

I. Vote for the board of directors IV. Vote on the takeover of another company Common stockholders elect the board of directors and vote on any major issues, such as a takeover, changing or keeping the company's present course of action, or any other major issues that the owners should make. The board elects officers and declares dividends.

A municipal broker's broker will do which of the following trading activities? I. Execute trades for retail customers II. Execute trades for broker/dealers III. Execute trades for broker/dealers for an institution IV. Execute trades for the firm's own account to fill an order

II. Execute trades for broker/dealers III. Execute trades for broker/dealers for an institution A broker's broker acts as a broker, and cannot buy for the firm's own account when purchasing for another dealer or institutional investor. Broker's brokers do not execute trades for retail customers. They execute trades for other brokers and for institutions when done through another B/D, but not for their own account or for public customers.

A new client opens a cash account with you and your firm. Under FINRA rules, which two of the following are minimum requirements that you must obtain from the client when opening the account? I. Bank or other financial institution II. Marital status III. Tax status IV. Date of birth

II. Marital status III. Tax status Besides the client's name, address, and phone number, an RR must obtain other information before opening an account. The marital status is a minimum requirement, as well as the tax status. The tax bracket is part of the "tax status," which could include other information about the client's tax situation such as other tax-advantaged accounts. Under FINRA rules, the age is required, but not the actual date of birth. However, the Patriot Act requires the date of birth for customer identification purposes.

Which two of the following are not subject to the FINRA 5% Policy? I. Municipal bonds II. Mutual fund transactions III. Secondary market transactions as a broker IV. New issue transactions

II. Mutual fund transactions IV. New issue transactions Mutual fund transactions and new issue transactions are not subject to the FINRA 5% Policy because they have a built in sales charge. The FINRA 5% Policy only covers trades in the secondary OTC market, including municipal and U.S. government bonds. FINRA member firms must follow the policy regardless of whether the firm acts as an agent, broker, principal, or dealer.

An investor has asked you about purchasing and owning GNMA securities. Which of the following would you tell your customer? I. They must be purchased at $1,000 minimum. II. They must be purchased at $25,000 minimum. III. They are U.S. government-backed bonds. IV. The interest is not subject to state taxation.

II. They must be purchased at $25,000 minimum. III. They are U.S. government-backed bonds. While Freddie Mac has a minimum purchase of $1,000, GNMA and FNMA are $25,000 minimum purchases. The federal government backs GNMA securities, but not FNMA. All three securities -- GNMA, FNMA, and Freddie Mac -- are subject to both federal AND state taxation. [Module 3, Government Securities, Sections 2.1 & 5.0]

A customer of yours is self-employed. He has a Keogh plan for himself and his employees. He is making a contribution to the plan for all participants in an amount of $80,000 for the year. The investment would be made into which of the following securities? I. Unit investment trusts made up of municipal bonds II. Treasury bonds III. Mutual funds IV. Insurance company stock

II. Treasury bonds III. Mutual funds IV. Insurance company stock An investment into unit investment trusts that invests in municipal bonds is not appropriate for a retirement fund. Since municipal bonds have tax-free interest, there is no sense in investing in those. Yes, Treasury bonds and stock of insurance companies are allowed.

Trading stocks on an ECN is for which of the following? I. New issues II. Over-the-counter stock III. Individuals IV. Institutions A. II and IV only B. I and III only C. II and III only D. III and IV only

III. Individuals IV. Institutions ECNs, electronic communication networks, are for individuals and institutions in Nasdaq and listed stock trades. Although Nasdaq stock used to be considered OTC stock, it is now an exchange. The OTC stock mentioned here is not Nasdaq. The answer choice with OTC stock is too encompassing, and the only stock that might be called OTC that can be traded through an ECN are Nasdaq stocks. ECNs are never for new issues.

As a registered rep, where can you find the most detailed information about a new municipal issue so you can fully inform your customer of the safety and suitability of the investment?

In the Official Statement. This is where the most detailed information for municipal securities can be found. The Blue List is a list of municipal offerings in the secondary market, with little or no information. Both the notice of sale and the tombstone are advertisements. The notice of sale is an advertisement from the issuer to underwriters and the tombstone is an advertisement from underwriters to the public. The Official Statement is a legal document providing all the important information of the issuer, and it outlines all of the rights and duties of the issuer and bondholder.

An investor buys 500 shares of DOW on margin in his margin account to take advantage of an increase in price. Later, he sells the position at that increase in price. What effect does the sale have on the SMA in his account?

Increases the SMA. The investor has a margin account with securities. When the investor sells some of the securities, whether at a profit or a loss, the broker will keep 50% of the sales proceeds. The customer has the option to take the other 50% or leave the rest of the proceeds from the sale in the account and reduce the debit balance even further. If the latter course of action is taken (leaving all of the proceeds from the sale in the account), the amount the investor "could have taken, but did not" becomes a line of credit to the customer; this is called SMA.

Income from which of the following securities is added to the investor's income and requires the investor to calculate the alternative minimum tax in addition to his regular tax? A. Interest from general obligation bonds B. Interest from industrial development bonds C. Interest from corporate subordinated debentures D. Dividends from a corporation paid on preferred stock

Interest from industrial development bonds In the 1987 Tax Equity Act, IDRs became tax-free interest that had to be claimed as income under the alternative minimum tax. Corporate bonds are already taxed, and general obligation bonds are always exempt from taxation since they are voted on by the residents.

Which of the following is not a tax preference item when determining the amount for tax purposes? A. Interest received on an IDR bond issued for pollution control B. Accelerated depreciation on a K-1 resulting from a DPP program C. Percentage depletion from an oil income program D. Intangible drilling costs from a gas exploration drilling program

Interest received on an IDR bond issued for pollution control. Tax preference items are part of the alternative minimum tax that is added back into a person's income when determining their tax under the AMTI. Accelerated depreciation, percentage depletion, and intangible drilling costs (IDCs) are all excessive deductions and are added back into a person's AMTI. [Module 17, Taxation, Section 2.4]

A customer purchases 500 shares of stock regular way on Monday, July 2. The customer tells you on Tuesday that he needs some extra time to get his money. When must the customer request an extension if he is unable to pay for the stock trade by settlement date?

July 10. The extension can be requested on the fifth business day beyond the trade date. Actually, the rule allows two days beyond the settlement date for the Reg T settlement of a trade. Regular way is three business days and two more days for Reg T is five business days after the trade date (not including the Fourth of July, a holiday), plus the weekend, makes the Reg T settlement date July 10. On this date, the firm must request an extension if it believes that the trade will be settled. Don't count holidays in determining the settlement date, the ex-dividend date, or the latest date for an extension.

You have a customer who buys $10,000 worth of U.S. Treasury bonds on Wednesday, July 3. When does the customer have to pay for the bonds in a regular way transaction?

July 5th July 5. U.S. government securities settle the next business day in a regular way trade. Since the Fourth of July is a holiday, the next business day is July 5. Remember, if the trade is on Friday, the next business day is Monday.

Equity securities have been purchased by an investor and are being held in safekeeping for her. In which of the following accounts will stock be held in street name? A. Option account B. Cash account C. Custodial account D. Margin account

Margin account. All stocks in a margin account must be held in street name so the B/D firm can sell them when an investor has failed to meet a margin call

Honeywell's board of directors announces a dividend to stockholders on record as of Thursday, July 5, payable on Tuesday, July 30. The ex-dividend date would be:

Monday, July 2. Make sure you do a calendar to do this problem. Start with Thursday July 5, and go back the full week. Don't forget that July 4th is a holiday, so put an H over it. The ex-dividend date is two business days before the record date. Two business days before the record date is July 2, which is actually three days due to the holiday

A corporation has declared a dividend payable to stockholders of record on Wednesday, July 23. Which of the following statements is correct? A. Monday, July 21 is the last day an investor could buy the stock and receive the dividend. B. Tuesday, July 22 is the ex-dividend date. C. The phrase "dividend on" or "with the dividend attached" would apply from Monday, July 21 to Friday, July 25. D. Monday, July 21 is the first day that an investor could buy the stock and not receive the dividend.

Monday, July 21 is the first day that an investor could buy the stock and not receive the dividend. the first day that an investor could buy the stock and not receive the dividend. First, make a calendar for the days before July 21 and for the days after July 21 to find the record date. Monday, July 21 represents the ex-dividend date or two business days prior to the record date.

Which of the following influences the economy by changing the money supply? A. Supply-siders B. Macroeconomics C. Keynesians D. Monetarists

Monetarists. The monetarists change the money supply, the supply-siders use tax cuts and the Keynesians do the opposite of supply-siders. Macroeconomics is the study of economics around the world, not just in one area.

An investor in the 28% tax bracket buys a municipal bond in the secondary market with a nominal yield of 6.25% and a yield to maturity of 7.25%. The investor holds the bond until it matures. The investor's after-tax yield will be:

More than 6.25%, but less than 7.25% (This is because the investor receives the 6.25% nominal yield tax-free; however, the bond has an effective yield of 7.25% because it was bought at a discount. Since the bond is purchased at a discount in the secondary market, the gain is going to be taxed as ordinary income. The investor does not really receive the benefit of the full amount of the gain since some of the gain goes to taxes. )

An investor sells 500 shares of DOW short to take advantage of a decline in price. Later, she covers the short position at the lower price. What effect does the buy-back have on the SMA in her account?

No effect. This is because there is only cash in the account to pay for the stock. If a profit were made in a long account, the proceeds would increase the SMA if left to reduce the debit balance. In this case, there will be more cash in the account, but not SMA.

A municipal bond is purchased in the secondary market at a discount and is later sold prior to maturity at a premium. For tax purposes, how will this investor treat the appreciation on the bond?

Part ordinary income and part capital gain (Any bond, including a municipal bond, purchased in the secondary market at a discount is accreted (amortized up) and treated as ordinary income. If the bond is held to maturity, the total appreciation is ordinary income.In this case, the bond was sold at a premium; therefore, it is taxed as ordinary income up to the new cost basis and capital gain above the cost basis to the proceeds)

A credit analyst is looking at a municipality's books and records, and performing a general analysis. Which of the following is considered a good advantage in the analysis of the municipality? A. Property taxes in the municipality have decreased. B. The rising per capita debt levels in the municipality is recognized. C. People in the municipality are not fully paying their property tax bills. D. An approved tax cut was set by the state legislature.

Property taxes in the municipality have decreased. This is a real toss-up in that an approved tax cut could also be an answer. The fact that property taxes are decreasing is the better answer, since they will only decrease if there is less to pay for. A negative of this is that the property is going down, thus causing a drop in the tax collections, but that is not stated in the question. An approved tax cut could be good, also signifying that there is less debt, or it could be brought on by political pressure, which means there will be cuts in funding programs in the municipality. This is not necessarily a good advantage. If you see a question of this kind, look at the wording in the question for more direction. The other two choices are detrimental to a municipality.

A husband and wife have a combined income of $180,000, and neither of their employers has a retirement plan. He is 52 years old and she is 47 years old. They wish to contribute to a traditional IRA. Which of the following is true of their contributions? A. Each may contribute $5,500 to their own IRA, but the contributions are not tax-deductible. B. She may contribute $5,500 to her own IRA, he can contribute $6,500 to his own IRA, and the contributions are tax-deductible. C. She may contribute $5,500 to her own IRA, he can contribute $6,500 to his own IRA, and the contributions are not tax-deductible. D. No contributions can be made into the IRA.

She may contribute $5,500 to her own IRA, he can contribute $6,500 into his own IRA, and the contributions are tax-deductible. . If either person, or both, had a retirement plan through their employer, the contribution could be made but would not be tax-deductible. Since he is over 50 years of age, he can contribute $6,500 to his account.

A customer of yours owns 500 shares of MBC stock that was purchased at $45 per share. Six-months later, she sells two-month 50 calls on the stock for a premium of 7. Just before expiration, she closes the options and two days later sells more two-month 50 calls on the stock for a premium of 4, which will expire in the next tax year. Which of the following statements is true of your customer's transactions? A. She will have no tax consequences since she owns the stock. B. She will have a short-term gain for the year in which the options were closed. C. She will have no gain or loss since she reopened the transaction. D. She cannot take a loss, but if a gain were realized it would reduce the cost of the stock.

She will have a short-term gain for the year in which the options were closed. Each option is a separate security, so the wash sale rules do not apply in their sale and purchase. They do have an effect on the holding period of stock, or can affect the stock in a wash sale; however, selling, closing (purchase), and selling again, or purchasing, closing (sale), and purchasing again are not governed by the wash sale rules.

There is overlapping debt in all of the following municipal tax districts, except: A. School district B. Road district C. State tax district D. Water district

State tax district Overlapping debt must have two separate entities and a state is only one entity. The different states do not share taxing authority over any parcel.

A potential client comes into your office and requests you. She knows a friend of yours and tells you she wants to open an account with you. In getting her information in the new account form, you notice that she is still 17, but will turn 18 in two months. What do you do?

Tell her she cannot open an account with you until she turns 18 and to call you on that day. You must refuse to open the account. However, you can take down all of her information, and activate her account on the day she turns 18. She is a minor and thus cannot have a joint account (don't confuse this with an UGMA/UTMA account). FINRA and the firm cannot override the legal aspects of opening an account.

The Japanese government has been concerned with the movement of the U.S. dollar. Which of the following would affect the Japanese yen in a deflationary market? A. The Japanese government buying U.S. Treasury bills B. Japanese citizens buying U.S. Treasury bills C. U.S. citizens buying Japanese stock D. The U.S. government buying Japanese stock

The Japanese government buying U.S. Treasury bills. With the movement in the U.S. dollar, the Japanese government would buy or sell U.S. Treasury securities, generally T-bills since they mature in a shorter period of time. In fact, the Japanese actually did this very thing in 1998 when they felt that the yen was deflating against the dollar.

A customer of yours is 65 years old. She has made regular contributions to a Keogh plan for 25 years and is now retiring. Which of the following is true regarding her withdrawals? A. The amount will be withdrawn tax-free. B. The amount will be withdrawn but is fully taxable as ordinary income. C. The amount will be withdrawn but is partially taxable as ordinary income and partially taxable as a capital gain. D. The amount will be withdrawn but is fully taxable as ordinary income plus a 10% tax penalty.

The amount will be withdrawn but is fully taxable as ordinary income. This is a Keogh plan and all contributions to the plan are from the customer's ordinary income unless additional contributions have been made that are not tax-deductible. Since no mention is made of voluntary contributions but only the regular contributions, it is all ordinary income. There are never capital gains with pension plans and since the customer is over 59 1/2 (think over 60), there is no penalty tax, whether taken in lump sums or as a retirement.

The city of Spokane is preparing to issue a bond on a new shopping mall. Spokane hires a bond attorney to give an opinion on the bond. The bond counsel sends back a "qualified legal opinion" on the issue. Which of the following statements is true?

The bond has some qualifying areas that could cause the bond to lose its tax-exempt status.

You have a new investor who likes to purchase income bonds. Which of the following statements is true regarding this investor? A. The customer is realizing income through interest payments. B. The customer is not looking for income, but for safety of principal. C. The customer is looking for appreciation of the bond. D. The customer is looking for income from interest and appreciation from the bond principal.

The customer is looking for appreciation of the bond. Income bonds do not have interest income, unless the company gets back on its feet. Income bonds usually trade at a discount because they pay no interest, so the only thing this customer can get is appreciation. There will probably be many missed interest payments, which are not made up.

customer calls in the morning and tells you to enter an order to purchase 200 shares of ITEC stock at the market. The order is executed as requested. Later in the day, she calls and tells you she did not really want that stock. What will happen?

The firm will sell out the customer and freeze her account for 90 days. The customer must pay for the securities or the firm has the right to sell her out and freeze her account. She is given the choice to pay for them or let the firm sell them, and the RR will do her best to get the customer to pay. For this question, the only logical thing that WILL happen is to sell out the customer and freeze the account.

An issuer is subject to Subchapter M. Which of the following is true?

The issuer must pass-through 90% of its net investment income. This is for all investment companies and REITs. The Subchapter M is part of the Internal Revenue Code, and is the reason there is no taxation to the investment companies or REITs, PROVIDED they pass-through at least 90% of their net investment income. [

The limited partners in an oil and gas direct participation program are unhappy with the general partners who are managing the assets and the direction the program is taking. What action can be taken by the limited partners to remove the general partner and not lose their limited liability status?

The limited partners can sue in a court of law to remove the general partner and have the court appoint a new general partner if one is needed. If the limited partners take any of the other actions, they become a general partner. The only way they can keep their limited liability status is to sue for the removal of the general partner and, if needed, the appointment of a new general partner.

A customer opens an options account with your firm. Under FINRA rules, which of the following signatures is required on the new account form? A. The registered rep B. The customer C. The general securities principal D. The options principal

The options principal, or the BOM if it is given, must sign the new account form for an options account.

You have been given an order to buy 300 shares of PG&E, a NYSE-traded stock. The order ticket is written and the order is executed through the automated execution system, SuperDot. Which of the following is true? A. The order goes directly to the floor trader for execution. B. The order is sent by the trading desk to the floor of the exchange where the order is executed in the crowd. C. The order goes directly to the specialist for execution. D. The order is sent to the market maker with the best price for execution.

The order goes directly to the specialist for execution. SuperDot is a direct electronic communication network that is a link from the trading desk of firms tied into the system to the specialist. Market makers are in the OTC marketplace, specifically in Nasdaq and the Pink Sheets.

A registered rep has been licensed for five years. At the end of the second year, the representative completes the regulatory education as required. At the end of the fifth year, the representative has not completed the next phase of the regulatory education in the time required. Which of the following is true? A. The representative cannot conduct business until his regulatory education has been completed. B. The representative is fined and sanctioned. C. The representative must retake the qualifying exam to have his license reinstated before conducting any business. D. The representative may continue to contact existing and prospective clients, but may not execute trades until his regulatory education has been completed.

The representative cannot conduct business until his regulatory education has been completed. The representative cannot conduct business until his regulatory education has been completed; this includes contacting clients or prospects. FINRA does not fine or sanction the rep, or require that he retake the qualifying exam.

Your client instructs you to sell his shares of EGG "at the close." What does this mean?

The trade must be effected as close to the closing price as possible. This is the definition of an "at the close" order. There can never be a guarantee to get the closing price, as the floor broker does not know when the last market order to come in will be the last for the close. "At the opening" can be executed at the opening, but "at the close" is as "close to the close as possible."

An investor is involved in a municipal bond swap to accomplish all of the following for his municipal portfolio, except: A. To increase cash flow B. To incur a loss for tax purposes C. To change to a high-quality portfolio D. To decrease his cost basis

To decrease his cost basis There is no reason to participate in a municipal bond swap in order to change cost basis, because if the bond is purchased at anything except par, it will change yearly anyway.

A man in his late 40s is in a low tax bracket. He has $40,000 to invest and wishes to make the best investment for his 12- and 14-year-old children's college education. As his registered representative, you would advise him to invest in which of the following? A. Zero-coupon bonds B. Treasury notes C. Treasury bonds D. Municipal bonds

Treasury notes Since the oldest child is 14 years old, the money will likely be needed in four years. Although safe, the zero-coupon bonds and Treasury bonds are too long-term. The investor is in a low tax bracket and would not benefit by buying municipal bonds. Treasury notes have maturities of one to 10 years, making them the best choice of those given.

An investor uses the compound accreted value call feature. This applies to:

Zero-coupon bonds. All the other bonds are subject to the either the interest rate as set by the Fed or by the bonds themselves. Zero-coupon bonds are discounted, so the value they sell at is the compound accreted rate.


Ensembles d'études connexes

Quizzes-IP 181 REGULATION OF INSURANCE PRODUCTS

View Set

Small Business Management (Midterm)

View Set

Chapter 8: The Courtroom Work Group and the Criminal Trial

View Set

Foundations II: Review for Exam 1

View Set